Sei sulla pagina 1di 174

Publication Date: 02/26/2010

Business Environment & Concepts

Publication Date: 02/26/2010

Making things easier


Please fold the pages lengthwise with the questions on the left half of the
page and the answers on the right for instant feedback when you need it, but
not before.
This is copyrighted information for your personal use only. It is not cool to
forward or resell to others.

Business Environment & Concepts > Corporations > Dissolution

Dissolution
Question 1

Answer 1

In general, for a corporation to dissolve:

Correct answer is A

A A corporation's board of directors may propose


dissolution for submission to the shareholders.
B The officers of the corporation recommend
dissolution to the shareholders.
C The corporation must notify only shareholders
who are entitled to vote.
D Adoption of the proposal to dissolve requires the
approval of 2/3 of shareholders entitled to vote.

In general, for a corporation to dissolve, the board


of directors votes upon a resolution proposing
dissolution. All shareholders must be notified of the
resolution to dissolve. The resolution is voted
upon, and can be approved by a majority of a
quorum.

Question 2

Answer 2

Which statement is correct with respect to a court ordered


dissolution of a corporation? A corporation will be dissolved
whenever the attorney general of the state of incorporation
establishes that I. the corporation obtained its articles of
incorporation through fraud; II. the corporation has continued
to exceed or abuse the authority conferred upon it by law; III.
the directors are deadlocked in the management of the
corporate affairs and the shareholders are unable to break the
deadlock.

Correct answer is C

A I only is correct.
B II only is correct
C I and II only are correct
D I, II and III are correct.

The Model Business Corporation Act provides that


in a proceeding by the attorney general if it is
established that: (i) the corporation obtained its
articles of incorporation through fraud; or (ii) the
corporation has continued to exceed or abuse the
authority conferred upon it by law, an appropriate
court may order dissolution of the corporation. In a
proceeding by a shareholder if it is established that
the directors are deadlocked in the management of
the corporate affairs and the shareholders are
unable to break the deadlock, and irreparable
injury to the corporation is threatened, then judicial
dissolution may be warranted.

Business Environment & Concepts > Corporations > Articles of Incorporation

Articles of Incorporation
Question 1

Answer 1

Under the Model Business Corporation Act, under what


circumstances can a corporation's board of directors amend
the articles of incorporation without shareholder approval? I. To
increase the number of authorized shares of the class to the
extent necessary to permit the issuance of shares as a share
dividend. II. To change the corporate name to a name that
better reflects the nature of its business.

Correct answer is A

A I only
B II only
C I and II are both correct.
D Neither I nor II is correct.

Section 10.05 of the Model Business Corporation


Act provides: Unless the articles of incorporation
provide otherwise, a corporation's board of
directors may adopt amendments to the
corporation's articles of incorporation without
shareholder approval: . . . (4) if the corporation
has only one class of shares outstanding: (a) to
change each issued and unissued authorized
share of the class into a greater number of whole
shares of that class; or (b) to increase the number
of authorized shares of the class to the extent
necessary to permit the issuance of shares as a
share dividend; (5) to change the corporate name
by substituting the word "corporation,"
"incorporated," company," "limited," or the
abbreviation "corp.," "inc.," "co.," or "ltd.," for a
similar word or abbreviation in the name, or by
adding, deleting, or changing a geographical
attribution for the name.

Question 2

Answer 2

Under the Model Business Corporation Act, which is correct?


The articles of incorporation must set forth any classes of
shares and series of shares within a class, and must I.
authorize one or more classes or series of shares that together
have unlimited voting rights. II. authorize one or more classes
or series of shares that have special, conditional, or limited
voting rights.

Correct answer is A

A I only
B II only
C I and II are both correct.
D Neither I nor II is correct.

The articles of incorporation must authorize: (1)


one or more classes or series of shares that
together have unlimited voting rights, and (2) one
or more classes or series of shares (which may be
the same class or classes as those with voting
rights) that together are entitled to receive the net
assets of the corporation upon dissolution. The
articles of incorporation may authorize one or more
classes or series of shares that have special,
conditional, or limited voting rights, or no right to
vote.

Question 3

Answer 3

In reviewing the proposed Articles of Incorporation of Buena


Vista, Inc., to determine whether the incorporators have
included all necessary information, which information would
NOT be required?

Correct answer is C

A Name and address of the registered agent.


B Names of incorporators.
C Par value of stock authorized.
D Name of the corporation.

Articles of Incorporation must include the name of


the corporation, the name and address of a person
or entity who is authorized to accept service of
process and official notices on behalf of the
corporation (registered agent) and the names of
the incorporators. Articles of Incorporation may
also include names of the initial directors, the
purpose of the corporation, par value of stock as
well as other information.

Business Environment & Concepts > Corporations > Articles of Incorporation

Question 4

Answer 4

Corporations from time to time amend their articles of


incorporation. Under the Model Business Corporation Act,
which of the following is correct regarding such amendments?
I. A shareholder of the corporation does NOT have a vested
property right resulting from any provision in the articles of
incorporation, including provisions relating to management,
control or dividend entitlement. II. If a corporation has not yet
issued shares, its board of directors or its incorporates may
adopt one or more amendments to the corporations articles of
incorporation.

Correct answer is C

A I only is correct.
B II only is correct.
C Both I and II are correct.
D Neither I nor II is correct.

A corporation may amend its articles of


incorporation at any time to add or change a
provision that is required or permitted in the articles
of incorporation as of the effective date of the
amendment or to delete a provision that is not
required to be contained in the articles of
incorporation. A shareholder of the corporation
does not have a vested property right resulting
from any provision in the articles of incorporation,
including provisions relating to management,
control, capital structure, dividend entitlement, or
purpose or duration of the corporation. If a
corporation has not yet issued shares, its board of
directors, or its incorporates if it has no board of
directors, may adopt one or more amendments to
the corporations articles of incorporation.

Business Environment & Concepts > Corporations > General considerations

General considerations
Question 1

Answer 1

Pre-employment screening is a necessary hiring practice for


many employers. With regard to pre-employment screening,
which is correct?

Correct answer is D

A Drug testing is permissible.


B Psychometric testing is often utilized.
C Background checks are common.
D All of the above are true.

Drug testing is permissible, psychometric testing


(measurement of attitudes and personality traits)is
often utilized. Background checks are also routine
in many instances.

Question 2

Answer 2

Ms. Foster incorporated her business as Foster Designs, Inc.


Under the Model Business Corporations Act, which of the
following statements is correct?

Correct answer is C

A Foster cannot be the sole shareholder of the


corporation.
B The corporation will be unlawful if Foster
incorporated solely for the purpose of limiting her
liability.
C Foster Designs, Inc., must designate a registered
agent for the purpose of receiving notice on behalf of the
corporation.
D If Foster dies, Foster Designs, Inc., is
automatically terminated if Foster is the sole employee.

An individual may incorporate for the purpose of


limiting his/her personal liability, and may be the
sole employee and sole shareholder of the
corporation. The corporation, however, has
perpetual existence and does not automatically
cease with the death of its sole shareholder. (If the
annual corporation fees are not properly paid, the
corporation will be terminated.) A corporation must
name a registered agent to receive service of
process and other official notices on behalf of the
corporation.

Business Environment & Concepts > Corporations > Business judgment rule

Business judgment rule


Question 1

Answer 1

In an action brought by Kline, a minority shareholder in


TinType Corporation, Kline established that the corporation
negligently opened a satellite sales facility in a foreign country
which was at the time experiencing significant political and civil
unrest. The facility was ultimately closed, resulting in losses to
TinType Corporation of in excess of $25,000,000, which, in
turn, resulted in a severe decline in the value of Kline's stock.
With regard to the liability of the officers and directors of
TinType Corporation, the Business Judgment rule

Correct answer is C

A is a minority rule, not followed in most modern


courts.
B provides for liability of officers and directors who
fail to exercise reasonable business judgment.
C requires proof of fraud, gross negligence or bad
faith in order to recover.
D applies a strict liability standard for officers, but
not directors.

Under the Business Judgment rule, officers and


directors will not be liable for acts which involve
their business judgment as long as they acted in
good faith and were not grossly negligent.

Business Environment & Concepts > Corporations > Bylaws

Bylaws
Question 1

Answer 1

Under the Model Business Corporation Act, which of the


following is a correct statement?

Correct answer is B

A Only a corporation's directors may amend or


repeal the corporation's bylaws.
B Bylaws may be amended unless the articles of
incorporation reserve that power exclusively to the
shareholders.
C A corporation's bylaws canNOT be repealed.
D Bylaws may be amended by the corporation's
officer's.

A corporation's shareholders may amend or repeal


the corporation's bylaws. In addition, a
corporation's board of directors may amend or
repeal the corporation's bylaws, unless: (1) the
articles of incorporation or section 10.21 reserve
that power exclusively to the shareholders in whole
or part; or (2) the shareholders in amending,
repealing, or adopting a bylaw expressly provide
that the board of directors may not amend, repeal,
or reinstate that bylaw.

Business Environment & Concepts > Corporations > Stock

Stock
Question 1

Answer 1

Which if the following is/are correct under the Model Business


Corporation Act with regard to articles of incorporation? I.
shares of one class may be authorized to elect a specified
number of directors while shares of another class may be
authorized to elect the same or a different number of directors;
II. shares of one class may be entitled to different dividend
rights or rights on dissolution.

Correct answer is C
The Model Business Corporation Act permits the
authorization of stocks with varying degrees of
dividend rights, voting rights, and rights to assets
on dissolution.

A I only
B II only
C Both I and II are correct
D Neither I nor II is correct

Question 2
A stockholder of the Chaplin Corporation received each of the
following items. All of them are considered asset or capital
distributions, except:
A a stock split of 300 previously unissued shares of
the companys stock
B a distribution of three acres of land
C a dividend of $3,000 based on the profits of the
company
D a dividend of $2,000 in cash that is deemed to be
a liquidating dividend

Answer 2
Correct answer is A
A stock split is a type of stock dividend that neither
reduces the assets of the business nor increases
the stockholders percentage of the ownership. It
is not viewed as a distribution but rather as an
attempt by the company to reduce the market price
of the companys stock by increasing the supply. It
is not viewed as an asset or capital distribution.
The business is no worse off and the stockholder
is no better off.

Business Environment & Concepts > Corporations > Derivative suits

Derivative suits
Question 1

Answer 1

A Derivative proceeding means a civil suit in the right of a


domestic corporation. Under the Model Business Corporation
Act, which of the following is a correct statement regarding
derivative actions?

Correct answer is C

A A shareholder may commence a derivative


proceeding even though s/he was not a shareholder of
the corporation at the time of the act or omission
complained of.
B A shareholder may commence or maintain a
derivative proceeding regardless whether the
shareholder fairly represents the interests of the
corporation.
C Unless irreparable injury is involved, no
shareholder may commence a derivative proceeding
until a written demand has been made upon the
corporation to take suitable action.
D Before commencing a derivative suit, a
shareholder must wait six months after making demand
for action.

A shareholder may not commence or maintain a


derivative proceeding unless the shareholder: (1)
was a shareholder of the corporation at the time of
the act or omission complained of or became a
shareholder through transfer by operation of law
from one who was a shareholder at that time; and,
(2) fairly and adequately represents the interests of
the corporation in enforcing the right of the
corporation. No shareholder may commence a
derivative proceeding until: (1) a written demand
has been made upon the corporation to take
suitable action; and (2) 90 days have expired from
the date the demand was made unless the
shareholder has earlier been notified that the
demand has been rejected by the corporation or
unless irreparable injury to the corporation would
result by waiting for the expiration of the 90 day
period.

Business Environment & Concepts > Corporations > Extraordinary transactions

Extraordinary transactions
Question 1

Answer 1

The management of Bowden Corporation approached key


personnel of Mx, Incorporated for the purpose of exploring a
merger or consolidation of assets of the two corporations. After
extensive negotiations it was determined that Mx would merge
into and become a part of Bowden Corporation. With regard to
the merger

Correct answer is A

A majority approval of both companies shareholders


is required.
B if 95% of Bowden shareholders agree to the
merger, dissenting shareholders are without recourse.
C if the directors of Bowden Corporation approve,
approval by Bowdens shareholders will NOT be
required.
D Mx, Incorporated must file new articles of
incorporation if the merger occurs.

Extraordinary transactions between corporations


(mergers, consolidations, sale of all assets) involve
three steps. First, the board of directors of each
corporation meets and decides on the action, then
makes a recommendation to shareholders.
Second, shareholders vote, and unless the article
or bylaws of the corporation provide otherwise, the
recommended action must be approved by majority
vote of the shareholders. Finally, dissenting
shareholders have appraisal rights, meaning that
they may demand to be paid the value of their
shares immediately prior to the transaction.

Business Environment & Concepts > Corporations > Directors, shareholders

Directors, shareholders
Question 1

Answer 1

Phillips and Broaddus are each 50% shareholders in Angel


Corporation, a manufacturer of luxury sail craft. Broaddus is
president and runs the day to day affairs of the business.
Phillips considers herself a "silent partner" since her only
involvement in the business is that she provided financing at
the company's inception. Angel Corporation was duly
incorporated. Which statement is correct?

Correct answer is B

A Corporate governance is generally a matter of


federal law.
B Broaddus and Phillips an equal say in the
selection of directors.
C Phillips has the authority to fire Broaddus as
president.
D Broaddus cannot transfer his stock ownership
without first offering the stock to Phillips for its fair
market value.

Directors are selected by the shareholders and as


50% shareholders, Phillips and Broadus each has
an equal vote. Corporations and how they are
governed is a matter of state law, not federal law. A
right of first refusal is generally a matter of
shareholder agreement or of bylaws of the
corporation and, in the majority of jurisdictions, is
not imposed by law.

Question 2

Answer 2

Summers is the owner of 5 percent of the outstanding common


stock of Scher Corporation, holding a far greater share of the
company than any other individual or entity. Gregory, the
president of Scher, has convinced Scher's board of directors to
double Gregory's compensation for the upcoming fiscal year
even though Scher's profitability is flat. Summers is unhappy
with that action. Summers can

Correct answer is C

A terminate Gregory with or without cooperation of


the board of directors.
B cause the board to terminate Gregory by filing a
derivative suit.
C best exert his influence by voting for or against the
directors at the next annual meeting.
D force the board, through a derivative action, to
reduce Gregory's compensation to a reasonable
amount.

The power to set compensation is vested in


management, and overseen by the board of
directors. Even a shareholder who has a significant
stock holding has no power to hire or fire an officer
nor to directly vote upon officer compensation.
Shareholders may vote for or against directors at
annual shareholder meetings.

Business Environment & Concepts > Corporations > Meetings

Meetings
Question 1

Answer 1

Under the provisions of the Model Business Corporation Act:

Correct answer is C

A A shareholder may waive any notice required by


the Act, the articles of incorporation, or bylaws, but only
before the date and time stated in the meeting notice.
B A corporation must notify shareholders of the
date, time, and place of each annual and special
meeting no fewer than 30 nor more than 120 days
before the meeting date.
C Action required or permitted by the Corporation
Act to be taken at a shareholders' meeting may be taken
without a meeting if the action is consented to by all the
shareholders entitled to vote on the action.
D A corporation may hold a meeting of shareholders
annually, but is not required to do so.

A corporation must hold a meeting of shareholders


annually at a time stated in or fixed in accordance
with the bylaws. A corporation must notify
shareholders of the date, time, and place of each
annual and special shareholders' meeting no fewer
than 10 nor more than 60 days before the meeting
date. Unless the Act or the articles of incorporation
require otherwise, the corporation is required to
give notice only to shareholders entitled to vote at
the meeting. A shareholder may waive any notice
required by the Act, the articles of incorporation, or
bylaws before or after the date and time stated in
the notice. The waiver must be in writing, be
signed by the shareholder entitled to the notice,
and be delivered to the corporation for inclusion in
the minutes or filing with the corporate records.

Business Environment & Concepts > Corporations > Proxies

Proxies
Question 1

Answer 1

Under the Model Business Corporation Act, which of the


following statements regarding proxies is INCORRECT?

Correct answer is B

A A shareholder may vote his shares in person or


by proxy.
B An appointment of a proxy is effective at the time
it is executed.
C A proxy appointment is valid for 12 months.
D An appointment of a proxy is revocable unless the
appointment form or electronic transmission states that
it is irrevocable and the appointment is coupled with an
interest.

A shareholder may vote his shares in person or by


proxy. An appointment of a proxy is effective when
a signed appointment form or an electronic
transmission of the appointment is received by the
inspector of election or the officer or agent of the
corporation authorized to tabulate votes. An
appointment is valid for 11 months unless a longer
period is expressly provided in the appointment
form. An appointment of a proxy is revocable
unless the appointment form or electronic
transmission states that it is irrevocable and the
appointment is coupled with an interest.

Business Environment & Concepts > Corporations > Dividend Payments

Dividend Payments
Question 1

Answer 1

Marko Corp. is planning to declare a dividend. Which of the


following statements is correct?

Correct answer is D

A Marko Corp.'s president must approve the


dividend.
B Restrictions on dividends may NOT be included in
Marko Corp.'s Articles of Incorporation.
C A dividend may be declared by the directors or by
a vote of a majority of the shareholders.
D No distribution may be made if the distribution
would severely impede the corporation's ability to pay its
debts as they come due.

Dividends are declared by the directors. Articles of


incorporation may include restrictions on the
directors in this regard. In addition, no distribution
may be made if, after giving it effect: (1) the
corporation would not be able to pay its debts as
they become due in the usual course of business;
or (2) the corporation's total assets would be less
than the sum of its total liabilities plus (unless the
articles of incorporation permit otherwise) the
amount that would be needed, if the corporation
were to be dissolved at the time of the distribution,
to satisfy the preferential rights upon dissolution of
shareholders whose preferential rights are superior
to those receiving the distribution.

Question 2

Answer 2

A company issues 100,000 shares of common stock and


10,000 shares of preferred stock with a $2 cumulative
dividend. In the first year, no dividends are paid or declared.
Which of the following statements are true?

Correct answer is B

A The company should report a $20,000 liability on


its year-end balance sheet.
B All dividends in arrears on the preferred stock
must be paid before any dividends can be paid on
common stock.
C The preferred stockholders are guaranteed to
receive this $20,000 at some point in the future.
D By not paying the cumulative dividend, the
company will be forced into bankruptcy within 90 days of
the beginning of the new year.

A cumulative preferred stock simply means that all


dividends on the preferred stock must be paid as
required before any dividends can be distributed to
the owners of the common stock. The company is
not obligated in any other way. There is no
guarantee nor liability and failure to pay will not
automatically lead to bankruptcy. However, if the
board of directors ever formally declares these
dividends, the liability must be reported.

Business Environment & Concepts > Corporations > Officers

Officers
Question 1

Answer 1

Giles G. Powers, president of Powers Placement Services,


Inc., is also a company director. As the result of a dispute with
other members of the board of directors, Powers created Giles
Placement Services, Inc. Powers

Correct answer is B

A has NOT breached his fiduciary duty to Powers


Placement Services, Inc., even if Giles Placement
Services, Inc., competes for the same business as long
as proper notice was given.
B would have no fiduciary duty to Powers Placement
Services, Inc., if he resigned his position as president
and board member before organizing the new business.
C would only be restricted from competing with
Powers Placement Services, Inc., if a noncompete
agreement had been entered into.
D has a fiduciary duty to Powers Placement
Services, Inc., as long as his name remains part of the
company name.

Officers and directors of a corporation owe the


corporation certain duties, the most important of
which is the fiduciary duty. A fiduciary duty requires
that one act in the best interest of the company
and not in his or her own interest. Taking business
from Powers Placement Services, Inc., by
competing with it would constitute a breach of
Powers's fiduciary duty. If Powers first resigns from
Powers Placement Services, Inc., he would no
longer have a fiduciary duty to the company.

Business Environment & Concepts > Corporations > Piercing the veil

Piercing the veil


Question 1

Answer 1

Henry Wiston was president of Gravitas Enterprises, Inc.,


which engaged in consulting services in the processed foods
industry. With respect to debts of the corporation, Wiston will
be subject to personal liability if

Correct answer is B

A he incorporated for the sole purpose of limiting his


personal liability.
B he undercapitalized the corporation.
C he is the sole shareholder of Gravitas Enterprises,
Inc.
D the corporation has insufficient insurance to cover
the activities in which he ordinarily engages in his
capacity as president.

A creditor may "pierce the corporate veil" and hold


management of a corporation liable as partners (or
as a sole proprietor) if the corporation was not
sufficiently capitalized. Insufficient capital is a red
flag that a corporation was started as a sham.
There is no specific formula to ascertain whether a
company is adequately capitalized. Adequate
capitalization is measured by the nature and
magnitude of the corporate undertaking at the time
of the company's formation. Incorporating for the
sole purpose of limiting one's personal liability,
being a sole shareholder, or having insufficient
insurance coverage are not grounds for piercing
the limited liability protection which the corporation
provides its owners.

Question 2

Answer 2

Peter and Paul Maxwell are brothers and are president and
CFO, respectively, of Maxwell Corporation. Rocky Bank, a
creditor of Maxwell Corporation, seeks to pierce the corporate
veil of Maxwell Corporation and hold the Maxwell brothers
personally liable for the business's debt to the bank. Which of
the following would be least persuasive in convincing a court to
pierce the corporate veil?

Correct answer is A

A Peter and Paul Maxwell are sole shareholders of


Maxwell Corporation.
B Paul Maxwell used his position in the business to
commit fraud.
C Peter Maxwell exerted his control of the business
to violate a statutory duty.
D Maxwell Corporation has failed to maintain
corporate records or hold corporate meetings.

The fact that Peter and Paul Maxwell are sole


shareholders of Maxwell Corporation is relevant to
the inquiry of whether to pierce the corporate veil
and hold management individually liable, but it the
least important factor listed. To prevail a plaintiff
must prove that the defendant completely
dominated the business to the extent that the
corporation had no separate mind, will or existence
of its own. The court will also examine whether the
defendant used the control to commit fraud or
other wrong or to violate a statutory or other legal
duty. Piercing the corporate veil is also known as
the "alter ego" doctrine.

Business Environment & Concepts > Corporations > S Corporation

S Corporation
Question 1

Answer 1

A corporation may elect treatment as an S Corporation under


the Internal Revenue Code even if it has: I. more than 100
shareholders; II. more than one class of stock. Which is
correct?

Correct answer is D

A I only is permitted.
B II only is permitted.
C Both I and II are permitted.
D Neither I nor II is permitted.

An "eligible" corporation may elect treatment as an


S Corporation under the Internal Revenue Code
only if it is a domestic corporation which does not:
(A) have more than 100 shareholders, (B) have as
a shareholder a person (other than an estate or
certain trusts) who is not an individual, (C) have a
nonresident alien as a shareholder, and (D) have
more than one class of stock.

Business Environment & Concepts > Corporations > Stock, Par value

Stock, Par value


Question 1

Answer 1

Foy Corporation issued 500 shares of $100 par value stock to


Dale. Dale sold 300 shares to Ving for $15,000 who knew that
the price was same price which Dale had paid for the stock.
Three months later, Foy Corporation dissolved without
sufficient assets to pay creditors. County Bank, an unsecured
creditor of Foy Corporation, wishes to hold Dale or Ving liable
for the unpaid par value of their stock. County Bank may
recover

Correct answer is C

A nothing from Ving, but $25,000 from Dale.


B $25,000 from Ving and $25,000 from Dale.
C $15,000 from Ving.
D $50,000 from Dale.

A purchaser of stock which has been assigned a


par value must pay at least par, or be liable to
creditors for the difference. One who purchases
stock knowing that par has not been paid will
similarly be liable.

Business Environment & Concepts > Corporations > Corporations general requirements

Corporations general requirements


Question 1

Answer 1

Under the Model Business Corporation Act, all of the following


are true EXCEPT:

Correct answer is C

A A corporate name must contain the word


corporation, incorporated, company, or limited, or
an abbreviation of one of these.
B A corporations name must be distinguishable
from the fictitious name adopted by a foreign corporation
authorized to transact business in the state.
C A corporation may NOT change its registered
office or registered agent once it is filed with the
Secretary of the Corporation Commission.
D Articles of incorporation must authorize one or
more classes or series of shares that together have
unlimited voting rights.

Articles of incorporation must authorize: (1) one or


more classes or series of shares that together have
unlimited voting rights, and (2) one or more classes
or series of shares (which may be the same class
or classes as those with voting rights) that together
are entitled to receive the net assets of the
corporation upon dissolution. A corporation may
change its registered office or its registered agent
at any time, but must must properly file such
change with the Secretary of the Corporation
Commission.

Business Environment & Concepts > Corporations > Subscriptions before incorporating (6.20)

Subscriptions before incorporating (6.20)


Question 1

Answer 1

The Model Business Corporation Act sets forth parameters for


pre-incorporation stock subscriptions. The Act requires all of
the following EXCEPT:

Correct answer is C

A A subscription for shares entered into before


incorporation is irrevocable for six months unless the
subscription agreement provides a longer or shorter
period.
B The board of directors may determine the
payment terms of subscription for shares that were
entered into before incorporation unless the subscription
agreement specifies them.
C A call for payment by the board of directors need
not be uniform as to all shares of the same class.
D If a subscriber defaults in payment of money or
property under a subscription agreement, the
corporation may collect the amount owed as any other
debt.

Section 6.20 of the Model Business Corporation


Act provides (in pertinent part): (a) A subscription
for shares entered into before incorporation is
irrevocable for six months unless the subscription
agreement provides a longer or shorter period or
all the subscribers agree to revocation. (b) The
board of directors may determine the payment
terms of subscription for shares that were entered
into before incorporation, unless the subscription
agreement specifies them. A call for payment by
the board of directors must be uniform so far as
practicable as to all shares of the same class or
series, unless the subscription agreement
specifies otherwise. If a subscriber defaults in
payment of money or property under a subscription
agreement, the corporation may collect the amount
owed as any other debt.

Business Environment & Concepts > Microeconomics > Supply and Demand

Supply and Demand


Question 1

Answer 1

Which of the following causes a reduction in the quantity


demanded of product X?

Correct answer is C

A Consumers see product X as less desirable.


B Consumers buy more of a substitute good for X.
C The price of X increases.
D Consumers have higher income and X is an
inferior good.

The question requires a distinction between


movement along a demand curve (change in
quantity demanded) and a shift of the demand
curve (a change in demand). The Law of Demand
is based on a downward sloping demand curve
where quantity demanded and price are inversely
related, everything else equal. In this case a
reduction in the quantity demanded is a movement
along the demand curve due to a higher price.
Answers A, B and D cause a shift in the demand
curve (change in demand) rather than movement
along the demand curve. According to the law of
demand, C is the correct answer. An increase in
price causes a reduction of the quantity
demanded.

Question 2

Answer 2

Cool Cola and Sparkle are substitute soft drinks. Which of the
following would occur if the price of Cool Cola increased?

Correct answer is A

A The demand for Sparkle will increase and the


demand curve shifts right.
B The supply of Cool Cola will decrease and the
supply curve shifts left.
C The supply of Sparkle will decrease and the
supply curve shifts left.
D The demand curve for Cool Cola will decrease and
the demand curve shifts left.

Consumers react to a change in the price of one


substitute good with a change in demand for the
other good. In this case, a higher price for Cool
Cola makes consumers increase the demand for
the substitute good Sparkle. This would be a shift
of demand for Sparkle to the right, making A the
correct answer.

Question 3

Answer 3

When the price of product X went up the demand for product Y


decreased. In this case, which of the following must be true?

Correct answer is D

A Products X and Y are substitutes.


B Product Y is an inferior good
C Product Y is a normal good
D Products X and Y are complements.

Complements are goods that are used together. If


one good becomes more expensive, there will be a
reduction in the consumption of that good and a
reduction in the consumption of the complement
too. In this case, the correct answer is D. A higher
price for X results in a reduction in quantity
demanded for X and a shift in the demand to the
left for the complement Y.

Business Environment & Concepts > Microeconomics > Supply and Demand

Question 4

Answer 4

When consumer income decreased, the consumption of


lunchmeat fell and the consumption of peanut butter increased.
Everything else equal, which of the following must be true?

Correct answer is D

A Lunchmeat and peanut butter are substitute


goods.
B Lunchmeat and peanut butter are complements.
C Lunchmeat is an inferior good.
D Peanut butter is an inferior good.

When income rises consumers buy less of an


inferior good and more of a normal good. In this
case, peanut butter is an inferior good and
lunchmeat is a normal good, making D the correct
answer.

Question 5

Answer 5

Everything else equal, what happens to the market equilibrium


for cell phones if input prices for making cell phones increases?

Correct answer is A

A The supply of cell phones decreases and the price


of cell phones rises.
B The supply of cell phones increases and the price
of cell phones rises.
C The supply of cell phones decreases and the price
of cell phones falls.
D The supply of cell phones increases and the price
of cell phones falls.

An increase in input prices will shift the supply


curve of cell phones left (reduced quantity supplied
for every price). Everything else equal, the new
equilibrium where the supply and demand curves
intersect will be at a higher price for cell phones.

Question 6

Answer 6

When the price of product Z increases from $10 to $11 the


quantity demanded declines from 500 units to 450 units. Which
of the following is true?

Correct answer is C

A The demand for product Z is price elastic


B The demand for product Z is price inelastic.
C The demand for product Z has unit price elasticity.
D Product Z is an inferior good..

When the price increases by 10% (1/10) the


quantity demanded falls 10% (50/500). The
percentage change in quantity demanded divided
by the percentage change in price is one. In this
case the elasticity of demand is one (unit price
elasticity).

Question 7

Answer 7

Which of the following products has the lowest price elasticity


of demand?

Correct answer is C

A Autos
B Recreational vehicles
C Bread
D Furniture

Products with a low elasticity of demand are not


price sensitive. These products are generally a
small percent of a consumer's budget, a necessity
rather than a luxury, a consumption good rather
than a durable good, and a good for which there is
no good substitute. Answers A, B, an D are rather
large ticket items that make up a large part of a
consumer's budget. They also tend to be luxuries
rather than necessities. Answer C is the best
example of a good with low elasticity of demand.

Business Environment & Concepts > Microeconomics > Supply and Demand

Question 8

Answer 8

The price of a normal good increases. Which of the following is


true?

Correct answer is A

A The quantity demanded falls due to both the


substitution and income effects.
B The quantity demanded falls due to the
substitution effect but increases due to the income
effect.
C The quantity demanded increases due to the
substitution and income effects.
D The quantity demanded increases due to the
substitution effect and falls due to the income effect.

The substitution effect occurs when a product price


goes up and consumers switch to another cheaper
product, resulting in a decrease in quantity
demanded. The income effect occurs when the
price of a good goes up effectively reducing the
amount of income left to consume a "normal good."
In this case, the higher product price reduces the
quantity demanded due to the substitution effect
and also reduces the quantity demanded due to
the income effect. If the product were an inferior
good rather than a normal good, the lower income
would result in an increase in the quantity
demanded.

Question 9

Answer 9

Which of the following is capable of explaining a lower market


equilibrium price for cell phones?

Correct answer is D

A A change in consumer tastes and preferences


making cell phones more attractive.
B Increased consumer income when cell phones
are a normal good.
C An increased expectation for higher prices of cell
phones in the future.
D An improved technology that makes it cheaper to
produce cell phones.

This question revolves around factors causing


shifts in supply and demand. The correct answer is
D since the improved technology would cause the
supply curve to shift to the right, resulting in a
lower equilibrium price. The other answers all shift
the demand curve to the right (increased demand),
which would result in a higher price for cell phones.

Question 10

Answer 10

When we observe a 5% increase in the price of product W


there is a corresponding 2% increase in the quantity
demanded for product Z. Which of the following is true?

Correct answer is A

A Products W and Z are substitutes.


B Products W and Z are complements.
C Product Z is an inferior good.
D Product Z is a normal good.

Products W and Z have a positive cross elasticity


(2% / 5% > 0). Consumers see Z as a substitute
for W and consume more Z when W becomes
more expensive. The correct answer is A. B would
be true only if the consumption of Z went down
when the price of W went up. Answer C refers to a
product where consumers buy more of it when
income falls. Answer D refers to a good where
consumers buy less of it when income rises.

Business Environment & Concepts > Microeconomics > Supply and Demand

Question 11

Answer 11

A company producing product G is considering a price


increase. Market research suggests that a 1% increase in price
results in a 2% reduction in quantity demanded. Which of the
following is true?

Correct answer is B

A The company will have higher total revenue if it


increases the price of product G.
B The company will have lower total revenue if it
increases the price of G.
C Total revenue will stay the same if the company
increases the price of product G.
D Without knowing more information it is impossible
to know how a price increase will affect total revenue.

Product G has an elastic demand since the


absolute value of the percent change in quantity
demanded exceeds the absolute value of the
percentage change in price. When demand is
elastic, an increase in price makes the total
revenue smaller. Answer B is correct in this case.
Answer A would be true if the demand for G were
inelastic. C would be true if the demand for G has
unitary elasticity. D is incorrect because we can
tell how total revenue moves when a price change
occurs if we know about the elasticity of demand.

Question 12

Answer 12

A federal law sets a price floor for milk. Which of the following
occurs due to a binding price floor?

Correct answer is B

A An excess demand for milk relative to the supply


of milk.
B An excess supply of milk relative to the demand
for milk.
C More price competition in the milk market.
D Less non-price competition in the milk market.

A price floor is a minimum price that is only binding


if it is set higher than the market price for milk. At
the binding floor price above the equilibrium price,
surpluses develop (QS > QD). This also causes
more non-price competition among sellers and a
reduction in quantity demanded by consumers.
The correct answer is B. A is appropriate for a price
ceiling below the equilibrium price. C and D are
incorrect because the price floor increases
non-price competition and reduces price
competition.

Question 13

Answer 13

Assume the federal government sets a binding price floor for


honey. Which of the following is true in this case?

Correct answer is A

A The price floor is above the equilibrium market


price of honey causing a surplus of honey (quantity
supplied exceeds quantity demanded.)
B The price floor is above the equilibrium market
price of honey causing a deficit of honey (quantity
demanded exceeds quantity supplied).
C The price floor is below the equilibrium market
price of honey causing a surplus of honey (quantity
supplied exceeds quantity demanded).
D The price floor is below the equilibrium market
price of honey causing a deficit of honey (quantity
demanded exceeds quantity supplied).

Price floors are set to provide producers with a


price higher than what can be obtained in a free
market, making C and D incorrect. Note that a floor
(minimum price) below the market equilibrium
would be non-binding. The floor is above the
market equilibrium resulting in a quantity supplied
greater than the quantity demanded at the floor
price. The answer is A.

Business Environment & Concepts > Microeconomics > Supply and Demand

Question 14

Answer 14

A local lake is used for recreation and as a source of water for


the community. Recreation causes some pollution of the water
but a major source of pollution comes from a manufacturing
plant that disposes waste into the lake. A local official
maintains that a recreation use tax should be set to clean the
lake and that the manufacturer should pay a special tax in
excess of the recreation tax. Which of the following is
consistent with the official's proposal?

Correct answer is C
This case represents an externality where the
manufacturer imposes a cost on other users of the
lake. The externality is "internalized" by making the
manufacturer pay for the added costs imposed on
other users of the lake. This concept is called full
cost accounting. Answer C is correct.

A Price leadership
B Price ceiling
C Full Cost Accounting
D Fixed Cost

Question 15

Answer 15

You see a simultaneous decrease in supply and demand for


tobacco products. Which of the following represents the effect
of these shifts on output quantity and market price?

Correct answer is B

A Output quantity decreases and output price


increases.
B Output quantity decreases and output price is
indeterminate.
C Output quantity increases and output price
decreases.
D Output quantity and output price will both fall.

Starting with an equilibrium, a shift of the supply


curve to the left and a shift of the demand curve to
the left will give a new equilibrium with a lower
level of output but the change in price will depend
on the relative magnitude of the shift in supply
relative to the shift in demand. Price may go up,
go down, or stay the same.

Question 16

Answer 16

The Federal Government sets a price freeze on gasoline prices


at the pump. Which of the following is a consequence of the
price freeze?

Correct answer is B

A A reduction in demand for gasoline at the pump.


B An excess quantity demanded for gasoline relative
to the quantity supplied of gasoline at the pump.
C A reduction in the supply of gasoline at the pump.
D An excess quantity supplied of gasoline relative to
the quantity demanded for gasoline at the pump.

The price freeze sets an artificial price for gasoline


that is lower than the market price. This does not
involve a shift of demand (Answer A) or a shift of
supply (Answer C). A freeze price below the
market price results in a quantity demanded that is
greater than the quantity supplied (Answer B).

Question 17

Answer 17

Which of the following is likely to cause a surplus for a given


product?

Correct answer is C

A A price ceiling
B A price freeze
C A price floor
D A reduction in an input price

A surplus occurs when the quantity supplied


exceeds the quantity demanded. We observe a
surplus when an artificial price is set above the
market equilibrium price. Answer C is correct
because a binding price floor is set above the
market equilibrium price to provide a minimum
price for producers. Note that if the price floor is not
above the market price it is not binding because
the market price would be a minimum.

Business Environment & Concepts > Microeconomics > Supply and Demand

Question 18

Answer 18

A government policy provides a subsidy for the production of


corn. Which of the following is a consequence of this policy,
everything else equal?

Correct answer is A

A Shift the supply curve of corn to the right


increasing the supply of corn.
B Shift the demand curve for corn to the left
reducing the demand for corn.
C Shift the supply curve for corn to the right and shift
the demand curve for corn left.
D Shift the supply curve for corn to the left and shift
the demand curve for corn to the right.

A subsidy effectively reduces the costs of


producing a product. In this case the subsidy will
increase the production of corn, shifting the supply
curve to the right making B incorrect and A correct.
The demand curve is unaffected by the subsidy,
making C and D incorrect..

Question 19

Answer 19

When the price of product R increased by 2% the quantity


supplied of R increased 1%. Which of the following is
consistent with this information?

Correct answer is B

A The supply of R is elastic.


B The supply of R is inelastic.
C The supply of R has unitary elasticity.
D R must be a normal good

An inelastic supply curve occurs when the


percentage change in quantity supplied is less
than the percentage change in price. In this case
the percentage change in quantity supplied is 1%
when the percentage change in price is 2%. The
measure of supply elasticity is .5. The correct
answer is B, the supply is inelastic.

Question 20

Answer 20

If consumers and producers expect higher prices for shoes in


the future, which of the following would be true?

Correct answer is D

A The change in the equilibrium price is


indeterminate but the equilibrium quantity will decrease.
B The equilibrium price will increase and the
equilibrium quantity is indeterminate.
C The equilibrium price will fall and the equilibrium
quantity will fall.
D The change in the equilibrium price is
indeterminate and the equilibrium quantity will increase.

Producers respond to higher expected prices for


shoes by producing more today, shifting the supply
curve to the right. Consumers respond to expected
higher prices of shoes by buying more shoes
today, shifting the demand curve right. The new
equilibrium is to the right (higher quantity) but the
price is indeterminate, depending on the magnitude
of each shift. Answer D is the correct response.

Business Environment & Concepts > Microeconomics > Supply and Demand

Question 21

Answer 21

A company sells widgets for $100 each and demand is 10,000


units per year. The company is thinking of raising the price to
$102. Which of the following statements is true?

Correct answer is B

A If 10,000 units are now demanded by customers,


demand is said to be elastic.
B If 9,900 units are now demanded by customers,
demand is said to be inelastic.
C If 9,800 units are now demanded by customers,
demand is said to be inelastic.
D If 9,700 units are now demanded by customers,
demand is said to be inelastic.

The price of the goods has gone up 2 percent


($2/$100). The fall in demand is 0 (10,000 units), 1
percent (9,900 units), 2 percent (9,800 units), or 3
percent (9,700 units). Demand elasticity is found
by taking the change in quantity demand and
dividing it by the change in price. If the ratio is
greater than 1, demand is said to be elastic (people
move to other products to avoid the increase). If
the ratio is less than 1, demand is inelastic (people
stick with the product despite the rise in prices). If
the ratio is 1, demand is unitary. Since the price
went up 2 percent, any drop of more than 2
percent gives a ratio of more than one and demand
is elastic. Since the price went up 2 percent, any
drop of less than 2 percent gives a ratio of less
than one and demand is inelastic. (A) is less than 2
percent so the ratio is less than one and that is
inelastic. (C) is exactly 2 percent so that is unitary.
(D) is more than 2 percent which gives a ratio of
more than one and demand is elastic. (B) is the
answer because demand fell only 1 percent which
makes the ratio 1/2 or less than 1. Demand is
inelastic; few people switched to other products
despite the increase in price.

Business Environment & Concepts > Microeconomics > Consumer Choice

Consumer Choice
Question 1

Answer 1

A consumer has $40,000 of disposable income to spend on


durable and nondurable goods. The price of durable goods is
$1,000 per unit and the price of non-durable goods is $100 per
unit. Which of the following is true?

Correct answer is B

A The consumer has a budget constraint with a


slope of 1 non-durable good for 1 durable good.
B The consumer's optimal consumption will occur
where the ratio of the marginal utility of durable goods to
the marginal utility of non-durable goods is 10 to 1.
C The consumer's optimal consumption will occur
with 10 times more consumption of non-durable goods
as durable goods.
D The consumer's optimal consumption of durable
goods and non-durable goods will occur where there are
40 times more non-durable goods than durable goods.

The consumer's budget constraint provides the


possible tradeoff between non-durable and durable
goods based on prices. Answers A, C, and D are
incorrect because they have the wrong slope
(tradeoff) between non-durable and durable goods.
Optimal consumption subject to the budget
constraint requires knowing consumer preferences
as well as the budget constraint. In this case the
optimal condition occurs where the ratio of the
price of non-durable goods to the price of durable
goods (1/10) equals the ratio of the marginal utility
of consumption of non-durable goods to the
marginal utility of consumption of durable goods.
Answer B describes this condition. Note that this is
the same condition as setting the marginal utility of
consuming nondurable goods divided by the price
of non-durable goods equal to the ratio of the
marginal utility of consuming durable goods divided
by the price of durable goods.

Question 2

Answer 2

Which of the following is consistent with the Diminishing


Marginal Utility Principle?

Correct answer is B

A Consumers have less total utility from consuming


additional units of a good.
B Each increment of additional consumption of a
good provides a smaller increment of additional utility.
C The first unit of consumption of a good offers a
smaller increment to total utility than following units of
consumption of a good.
D Each increment of additional consumption of a
good provides a larger increment of additional utility.

The additional utility from consumption of one


more unit of a good is the marginal utility.
Diminishing marginal utility occurs when the
addition to total utility of the last unit of a good
consumed is less than the addition to total utility of
the consumption of the prior unit of a good. This
relationship is explained by answer B.

Question 3

Answer 3

When consumer income increases from $40,000 per year to


$42,000 per year, consumers increased consumption by
$1,500 per year. Which of the following is true in this case?

Correct answer is D

A The marginal propensity to consumer is 1.5.


B The marginal propensity to save is .75.
C The marginal propensity to consume is .25.
D The marginal propensity to save is .25.

The marginal propensity to consume is equal to the


change in consumption divided by the change in
income. In this case the marginal propensity to
consumer is .75. The marginal propensity to save
is equal to 1 - marginal propensity to consume. In
this case, the marginal propensity to save is .25.
The correct answer is D.

Business Environment & Concepts > Microeconomics > Consumer Choice

Question 4

Answer 4

Products X and Y are complements. The cross-elasticity of


demand for product X with respect to Y must be:

Correct answer is B

A Zero
B Less than Zero
C Greater than Zero
D Greater than 1

For complements, when the price of one good


goes up it not only reduced the quantity demanded
for that good but it also reduces the demand for the
complement. In terms of an elasticity, we observe
a negative percentage change in the demand for
product Y in response to a positive percentage
increase in the price of X, if Y and X are
complements. The cross-elasticity of demand is
negative for complements, making B the correct
answer.

Business Environment & Concepts > Microeconomics > Production

Production
Question 1

Answer 1

Which of the following must be true in the long run?

Correct answer is C

A Fixed costs must be higher than variable costs.


B There are no variable costs in the long run.
C There are no fixed costs in the long run.
D Fixed costs decline rapidly as production goes up.

In the long run all costs are variable, making C the


correct answer. Firms can change the scale of
operations in the long run while the scale is fixed
in the short run.

Question 2

Answer 2

Which of the following is true about the short run cost structure
of a firm as the quantity produced increases?

Correct answer is B

A Average fixed costs decline and then rise as the


firm produces more output.
B Average total costs reach a minimum when
average total cost equals marginal cost.
C When marginal costs are higher than average
total costs the average total cost declines.
D Marginal costs continue to decline as the firm
produces more output.

In the short run, fixed costs decline as the firm


produces more output (A is false), since the fixed
cost is spread out over more units. When the
marginal cost is above the average total cost the
average total cost must be increasing (C is false).
Marginal costs decline in the early stages of
production but increase as the law of diminishing
returns sets in. The inefficiencies of putting more
variable inputs to work with the same fixed inputs
will eventually make marginal costs higher (D is
false). Average total costs decline as long as they
are above marginal costs (incremental costs are
less so the average comes down). When the
marginal cost is above the average total cost the
average total cost must be increasing (the last
increments are higher than the average so the
average rises). The average reaches its minimum
when it is equal to the marginal cost. As the
marginal cost continues to increase and rises
above the average total cost the average total cost
must begin to increase ( B is correct).

Question 3

Answer 3

What eventually happens as a firm produces more and more of


an output with a fixed production capacity (fixed cost)?

Correct answer is C

A Efficiency increases and marginal costs decline.


B Marginal productivity of inputs increases.
C The Law of Diminishing Returns sets in.
D Average fixed costs increase.

In the short run the firm has fixed costs (fixed


production capacity). As more variable inputs are
added to the fixed inputs the efficiency
(productivity) begins to decline. This phenomena is
called the Law of Diminishing Returns (Answer C is
correct). Marginal costs increase once diminishing
returns set in (Answer A is incorrect) and the
marginal productivity of inputs declines (Answer B
is incorrect). Average fixed costs decline over the
entire range of output since (total fixed costs /
number of units produced declines as the number
of units produced increase (Answer D is
incorrect).

Business Environment & Concepts > Microeconomics > Production

Question 4

Answer 4

Which of the following offers the best explanation of the


difference between the law of diminishing returns and
decreasing returns to scale.

Correct answer is A

A Both represent an onset of inefficiencies but the


law of diminishing returns applies to the short run and
decreasing returns to scale applies to the long run.
B The law of diminishing returns refers to
inefficiencies due to changing the scale of production
and decreasing returns to scale refer to inefficiencies
due to using too many variable costs in combination
with fixed costs.
C The law of diminishing returns refers to
efficiencies from increasing the scale of production and
decreasing returns to scale refers to inefficiencies from
reducing the scale of production.
D The law of diminishing returns refers to a
consumer's reduced marginal utility from increasing the
number of units consumed. Decreasing returns to scale
refers to lower consumer marginal utility from higher
levels of income.

Both the law of diminishing returns and decreasing


returns to scale are due to inefficiencies in
production. The law of diminishing returns is due
inefficiencies and lower productivity caused by
adding too many variable inputs to fixed amount of
inputs. The law of diminishing returns is relevant
only in the short run where some inputs are fixed.
Decreasing returns to scale allows for changes in
production capacity but diseconomies of scale set
in to produce inefficiencies. Decreasing returns to
scale is a long run phenomena where there are no
fixed costs. Answer A matches this explanation of
the differences in the law of diminishing returns
and decreasing returns to scale. Answer B has the
relationships backward in terms of fixity of costs.
Answer C is incorrect because it allows for
changes in the scale of production in the short an
in the long run. Answer D is incorrect because it
uses consumer utility rather than production..

Question 5

Answer 5

Which of the following best explain increasing returns to scale?

Correct answer is C

A In the long run, when a firm increases all factors


of production by 10% there is a 5% increase in output.
B In the short run, when a firm increases the use of
variable inputs by 5% there is an 8% increase in output.
C In the long run, when a firm increases all factors
of production by 10% there is a 12% increase in output.
D In the short run, when a firm increases the use of
variable inputs by 5% there is a 3% increase in output.

Returns to scale only occur in the long run since a


change in scale represents a change in fixed costs
(Answers B and D are incorrect). Increasing returns
to scale results in efficiencies where economies of
scale allow for output to increase by a higher
percent than the percent increase in inputs.
Answer C is a long run increase in the percent of
output that is greater than the percent increase of
inputs, reflecting increasing returns to scale.

Question 6

Answer 6

Which of the following represents the expected order of returns


to scale as a firm expands production?

Correct answer is D

A Increasing returns to scale, decreasing returns to


scale, and constant returns to scale.
B Decreasing returns to scale, increasing returns to
scale, and constant returns to scale.
C Constant returns to scale, increasing returns to
scale, and decreasing returns to scale.
D Increasing returns to scale, constant returns to
scale, and decreasing returns to scale.

The long run average cost curve falls, reaches a


flat segment, and then rises. This pattern is
consistent with increasing returns to scale
(economies of scale and falling average costs),
constant returns to scale (constant or flat average
cots, and finally decreasing returns to scale (rising
average costs due to diseconomies of scale). The
correct answer is D.

Business Environment & Concepts > Microeconomics > Production

Question 7

Answer 7

If marginal revenue is less than marginal cost the quantity sold


is:

Correct answer is C

A At a profit maximizing level.


B At a level that is too low.
C At a level that is too high.
D It is impossible to tell whether the level of output is
too high or too low from the data given.

The optimal quantity to produce and sell occurs


where the marginal revenue of the last unit sold
equals the marginal cost of the last unit produced.
The marginal revenue curve falls as more output is
produced and sold. The marginal cost increases as
more output is produced and sold. If the marginal
revenue is below the marginal cost the level of
output must be too high, since we moved beyond
the optimal output and sales quantity. This makes
answer C correct.

Question 8

Answer 8

Which of the following represents the additional output


obtained from using one additional unit of a resource?

Correct answer is C

A Marginal Revenue
B Marginal Cost
C Marginal Product
D Marginal Revenue Product

Marginal revenue is the addition to total revenue


from the sale of one additional unit of product
(Answer A is incorrect). Marginal cost is the
addition to total cost from producing one more unit
of product (Answer B is incorrect) The change in
total revenue from employing one more unit of a
resource or input is the marginal revenue product
(Answer D is incorrect). Marginal product is the
additional output obtained from employing one
additional unit of a resource or input (Answer C is
correct).

Question 9

Answer 9

The price of a high quality coal is $1,000 per unit. A cheaper


substitute for this coal only costs $500 per unit, but the
cheaper substitute is less productive. Which of the following
would be required to justify using the higher quality coal?

Correct answer is D

A The marginal product of the higher quality coal


must be larger than the marginal product of the cheaper
coal by 500.
B The marginal product of the higher quality coal
must be larger than the marginal product of the cheaper
coal by 1000.
C The marginal product of the higher quality coal
must be at least half as high as the marginal productivity
of the cheaper coal substitute.
D The marginal product of the higher quality coal
must be at least twice as high as the marginal product of
the cheaper coal substitute.

The choice of which input to use is based on


finding the cheapest cost per unit to produce,
everything else equal. In this case the ratio of
marginal product of the input to the input price
measures the cost per unit. If we are to use the
higher quality coal, the higher quality coal just be
at least twice as productive as the cheaper coal in
order to have ratio of the marginal productivity to
input price. Answer D is correct..

Business Environment & Concepts > Microeconomics > Market Structure

Market Structure
Question 1

Answer 1

You have a plot of land that can be used for farming or for
developing a golf course. You are willing to sell the land for
$400,000 in either case. A developer offers you $1,000,000 for
the land. What does the additional $600,000 represent in this
case?

Correct answer is A

A Economic Rent
B Normal Profit
C Marginal Revenue
D Equilibrium Price

When an input is purchased for a more than the


next highest bidder would pay, the difference in
the amounts offered is called an economic rent
(answer A). A normal profit represents what it costs
to keep entrepreneurial skills at work in their
current use. Marginal revenue is the added
revenue generated from the sale of one more unit
of product. The equilibrium price is a market price
set by supply and demand interaction.

Business Environment & Concepts > Macroeconomics > Performance Measures

Performance Measures
Question 1

Answer 1

What is the appropriate measure of the total market value of all


goods and services produced with a country's resources (not
necessarily within its borders)?

Correct answer is B

A Gross Domestic Product (GDP)


B Gross National Product (GNP)
C Net Domestic Product
D National Income

The GDP is the total market value of goods and


services produced within a country, but not
necessarily with the resources of that country. Net
Domestic product is the GDP minus depreciation.
National income is the net domestic product plus
income earned abroad minus indirect business
taxes. B is the correct answer.

Question 2

Answer 2

Which of the following is the correct definition for GDP using


the expenditure (input) approach?

Correct answer is A

A consumption plus investment plus government


expenditure plus net exports.
B consumption plus saving plus government
spending minus net exports
C wages plus interest plus rent plus profits plus
depreciation plus indirect business taxes.
D wages plus interest plus rent plus profits minus
depreciation minus business taxes

B is incorrect because savings is used rather than


investment and net exports are subtracted rather
than added. C is the output (income and cost)
approach rather than the input (expenditure)
approach. D is an incorrect because it is the output
approach and since depreciation and business
taxes are subtracted rather than added. A provides
the correct definition of the income (expenditure)
approach to GDP.

Question 3

Answer 3

Which of the following is the correct order of the four phases of


the business cycle?

Correct answer is C

A peak, recession, recovery, trough


B peak, recovery, trough, recession
C peak, recession, trough, recovery
D peak, trough, recession, recovery

From the peak (top performance) the economy


starts to decline (recession) and reaches a bottom
(trough). From this trough the economy eventually
improves (recovery) and rises to another peak.

Question 4

Answer 4

Which price index is based on a comparison of the prices of


items in a "typical" shopping cart to a base value?

Correct answer is D

A The GDP Price Deflator


B The Wholesale Price Index
C The National Income Price Index
D The Consumer Price Index

The GDP deflator uses a factor based on all


production in the economy at prices used in the
GDP calculation. The wholesale price index uses
the price of items in a typical cart of wholesale
quantities to a base value. There is no National
Income Price Index. The correct answer is D.

Business Environment & Concepts > Macroeconomics > Performance Measures

Question 5

Answer 5

Unemployment that occurs when unemployed laborers have


manufacturing skills but job openings are for laborers with
service skills.

Correct answer is B

A Frictional unemployment
B Structural unemployment
C Cyclical unemployment
D Full employment

Frictional unemployment occurs when there is job


turnover. Cyclical unemployment occurs when
aggregate labor supply exceeds aggregate labor
demand in a low point of the business cycle. Full
employment occurs when all workers willing to
work at market wages are employed using their
skills. The correct answer is structural
unemployment.

Question 6

Answer 6

Which of the following describes the relationship between


inflation and the unemployment rate?

Correct answer is D

A When the unemployment rate falls inflation falls.


B When the unemployment rate falls inflation rises
at a decreasing rate.
C When the unemployment rate falls the inflation
rate increases at a constant rate.
D When the unemployment rate falls the inflation
rate increases at an increasing rate.

The relationship between the unemployment rate


and the inflation rate is called the Phillips curve. As
the unemployment rate falls the inflation rate
increases at an increasing rate. This gives the
Phillips curve a shape that is convex to the origin.
The slope gets steeper as unemployment falls and
flatter as unemployment increases. The correct
answer is D.

Question 7

Answer 7

Which of the following measures the average change over


time in the selling prices received by domestic producers for
their output?

Correct answer is A

A Producer price index


B The US index of domestic production.
C The inflation index for domestic production.
D The sales index for domestic output of durable
goods

According to the Department of Labor Statistics,


the Producer Price Index (PPI) program measures
the average change over time in the selling prices
received by domestic producers for their output.
The prices included in the PPI are from the first
commercial transaction for many products and
some services.

Business Environment & Concepts > International Economics > Trade

Trade
Question 1

Answer 1

Country A can produce 10 units of Y or 20 units of X. Country


B can produce 30 units of Y or 90 units of X. Which of the
following must be true?

Correct answer is B

A Country A has an absolute advantage in X and Y.


B Country A has a comparative advantage in Y and
Country B has a comparative advantage in X.
C Constant returns to scale, increasing returns to
scale, and decreasing returns to scale.
D A fair term of trade between Country A and
Country B would be 1X for 4Y.

Absolute advantage occurs when a country can


produce more of everything. In this case Country B
has the absolute advantage. A comparative
advantage occurs when a product can be produced
cheaper. Country A can produce 1Y or 2X.
Country B can produce 1Y or 3X. Y is cheaper in
Country A (it only cost 2X to produce rather than
3X) and X is cheaper in Country B (it only cost 1/3Y
rather than 1/2 Y). A fair term of trade would be
between 2 and 3 Y for 1X. The correct answer is B.

Question 2

Answer 2

Which of the following represents a control strategy where a


country places a tax on imports?

Correct answer is C

A A quota.
B Shift of consumer preference.
C A tariff.
D A substitution strategy.

Countries have several control strategies to create


barriers to free trade. A quota is a limit on the
quantity of goods and services that can be
imported. A shift of consumer preferences
encourages consumers to buy domestic products.
The substitution strategy develops substitute
goods for imports. A tax on imports is a tariff. The
answer is C.

Question 3

Answer 3

Everything else equal, what is the consequence of an increase


in the $/euro rate?

Correct answer is D

A U.S. exports to Europe are now more expensive.


B European exports to the U.S. are now less
expensive.
C The dollar has appreciated
D The dollar has depreciated.

Question 4
Which of the following corresponds to interest and dividends
received by residents in the U.S. from investments in other
countries.
A A credit in the capital account
B A credit in the current account.
C A debit in the capital account.
D A debit in the current account.

When the $/euro increases the euro is worth more


and the dollar is worth less. The dollar has
depreciated. When the dollar is cheaper, U.S.
exports to Europe are less expensive. European
exports to the U.S. in euros are now more
expensive. The correct answer is D.

Answer 4
Correct answer is B
Interest and dividends received in the U.S. from
investments abroad represents a credit. Interest
and dividends represent income recorded in the
current account.

Business Environment & Concepts > International Economics > Trade

Question 5

Answer 5

Which of the following is true about the International Monetary


Fund (IMF)?

Correct answer is B

A It prints money to be used by member countries.


B It has a pool of currencies from which member
countries can borrow to meet short-term deficits in
balance of payments.
C It serves as an investment resource for member
countries allowing them to make extremely high rates of
return on invested capital.
D It allows deposits by member countries which are
then allocated each year as a way to reward countries
which have made the most progress economically.

According to its website, the IMF promotes


international monetary cooperation and exchange
rate stability, facilitates the balanced growth of
international trade, and provides resources to help
members in balance of payments difficulties or to
assist with poverty reduction.

Business Environment & Concepts > LLCs > Management1

Management1
Question 1

Answer 1

A limited liability company (LLC) is a form of business offering


limited liability to its owners. Which of the following is correct
regarding LLCs?

Correct answer is B

A LLCs are generally treated like corporations for tax


purposes.
B An operating agreement is used to establish rules
governing the membership, management, operation and
distribution of income of the company.
C LLCs are organized under "articles of
incorporation.
D In general, an LLC's income and deductions are
attributed to one member while the remaining members
have no tax consequences.

A limited liability company (LLC) is a form of


business offering limited liability to its owners who
are usually referred to as members. LLCs are
organized under "articles of organization, or "the
rules of organization" and are generally treated like
partnerships for tax purposes. An operating
agreement is used to establish rules governing the
membership, management, operation and
distribution of income of the company, with income
and deductions attributed to each member and
reported on that member's tax return.

Business Environment & Concepts > LLCs > General2

General2
Question 1

Answer 1

A Limited Liability Company (LLC) is a relatively new business


structure allowed by state statute. Which of the following
statements regarding LLCs is not correct?

Correct answer is D

A LLCs are popular because, similar to a


corporation, owners have limited personal liability for the
debts and actions of the LLC.
B LLCs are like a partnership, providing
management the benefit of pass-through taxation.
C Since most states do not restrict ownership,
members may include individuals, corporations, other
LLCs and foreign entities.
D In most states there a maximum of 10 members in
an LLC.

A Limited Liability Company (LLC) is a relatively


new business structure allowed by state statute.
LLCs are popular because, similar to a corporation,
owners have limited personal liability for the debts
and actions of the LLC. Other features of LLCs are
more like a partnership, providing management
flexibility and the benefit of pass-through taxation.
Owners of an LLC are called members. Since most
states do not restrict ownership, members may
include individuals, corporations, other LLCs and
foreign entities. There is no maximum number of
members. Most states also permit single
member LLCs, those having only one owner.

Question 2

Answer 2

A few types of businesses generally cannot be LLCs. These


include

Correct answer is A

A banks and insurance companies


B charitable organizations
C construction companies
D television and radio broadcasters

Question 3
The advantages of a Limited Liability Company include all of
the following EXCEPT:
A limited liability of members
B unlimited life
C profits flow through to members as to avoid
double taxation
D flexible profit distribution

Question 4
One aspect of the Limited Liability Company (LLC) which
would generally be considered advantageous is
A limited life
B ability to go public
C flexibility of management
D laws regarding LLCs vary from state to state

Only a few types of businesses generally cannot


be LLCs, such as banks and insurance companies.
State requirements vary slightly and federal tax
regulations impact this as well.

Answer 3
Correct answer is B
Like a corporation, owners of an LLC enjoy limited
liability. Profits pass through to members, thus
avoiding double taxation (though an LLC can elect
to be taxed as a sole proprietor, partnership, S
corporation or C corporation). The economic
benefits of member interests can be separated and
assigned, giving the assignee the profits (or
losses) similar to a partnership.

Answer 4
Correct answer is C
In general, members of an LLC can establish
governance and protective provisions pursuant to a
contract or operating agreement, giving
management greaster flexibility than a corporation.
Corporations can continue forever, whereas an
LLC may dissolve when a member dies or files for
bankruptcy. Businesses which intend to go public
may be better served by choosing the corporate
form of business. And though it is a very minor
factor, laws governing LLCs vary from state to
state more so than corporation laws.

Business Environment & Concepts > LLCs > LLC and corporation compared

LLC and corporation compared


Question 1

Answer 1

Limited Liability Companies, a relatively new form of business


in the United States, provide all of the following EXCEPT:

Correct answer is C

A LLCs can elect to be taxed like partnerships so


that only the individuals are taxed on profits.
B LLC permit management to shield personal assets
from business creditors.
C LLCs issue common stock which is freely
tranferable.
D LLC have a limited life that varies from state to
state.

The LLC is a distinct entity from its owners. The


LLC form of business combines the corporate
advantages of limited liability with the partnership
advantage of pass-through taxation. As a result,
the LLC can elect to be taxed like partnerships -- at
the individual level only. This is an advantage over
C corporations which are subject to double-taxation
once at the corporate level, and again at the
individual level when profits are distributed as
dividends to the shareholders. Similar to
corporations, LLCs permit the shielding of personal
assets from business creditors. LLCs have a
limited life that varies from state to state and LLCs
do not issue stock.

Business Environment & Concepts > LLCs > Members as agents (301) and authority via operating agreement

Members as agents (301) and authority via operating agreement


Question 1

Answer 1

Which if the following is a correct statement of the law with


respect to a Limited Liability Company (LLC)? I. A member is
an agent of a limited liability company solely by reason of being
a member. II. In general, a members actual authority to act for
an LLC will depend fundamentally on the operating agreement.

Correct answer is B

A I. only is correct.
B II. only is correct.
C Neither I. nor II. is correct.
D Both I. and II. are correct.

Under section 301 of the Revised Uniform Limited


Liability Company Act, a member is not an agent of
a limited liability company solely by reason of being
a member. Comments contained in the Act state
that, in general, a members actual authority to act
for an LLC will depend "fundamentally on the
operating agreement" (which is generally in writing,
but may be oral). General agency law is applicable
as well.

Business Environment & Concepts > LLLPs > General3

General3
Question 1

Answer 1

With regard to an LLLP (limited liability limited partnership)


which of the following is correct? I. The LLLP form primarily is
used to convert an existing limited partnership previously
created under state law. II. The general partner in an LLLP has
limited liability similar to the limited partners.

Correct answer is D

A Neither I nor II is correct.


B I only is correct.
C II only is correct.
D Both I and II are correct.

An LLLP is a limited liability limited partnership or,


more specifically, a limited partnership (LP) that
registers under state law so the general partner will
have limited liability, similar to the limited partners.
The LLLP form primarily is used to convert an
existing limited partnership previously created
under state law.

Business Environment & Concepts > Proprietorship > Fictitious names

Fictitious names
Question 1

Answer 1

Hallasz operates a lawn maintenance business. In addition to


himself, Hallasz employs one other supervisor, Liggan, who is
paid a regular salary in season, and has approximately 22
regular and 12 part-time employees. Hallasz trades as Green
Pastures Lawn Care Company. If Hallasz takes no further
action regarding the form of his business organization, Green
Pasture Lawn Care Company will

Correct answer is C

A be deemed a corporation.
B treated as a general partnership,with Hallasz and
Liggan as general partners.
C be considered a sole proprietorship.
D not be in violation of state filing requirements.

Since Hallasz and Liggan do not carry on the


business as co-owners (Liggan is paid a salary
rather than a share of profits) there is no
presumption of partnership. Since no formal filings
have been made, the business would not be
treated as a corporation, a limited partnership, an
LLC, or other entity requiring such formal filings.
Hallasz will be treated as a sole proprietorship, but
is in violation of common fictitious name
statutes for doing business under a fictitious name
without filing with state authorities.

Business Environment & Concepts > Proprietorship > Proprietorship - adding a partner

Proprietorship - adding a partner


Question 1

Answer 1

Lewis, a sole proprietor doing business as Lewis Bait & Tackle


Shop, added Tomms as an employee. Lewis paid Tomms a
regular salary plus 10% of all profits. As a result

Correct answer is A

A Lewis may deduct, as an expense, the salary paid


to Tomms.
B Lewis need NOT make matching FICA payments
for Tomms.
C Lewis may NOT terminate Tomms without Tomm's
assent.
D Lewis and Tomms have formed a partnership.

When a proprietorship or a partnership pays an


employee a percentage of profits as part of the
employee's compensation, this alone is not
sufficient to change the employee's status to that of
a partner.

Business Environment & Concepts > Financial Environment/Overview > Business Objective

Business Objective
Question 1

Answer 1

The goal of management in a corporation should be to

Correct answer is D

A Maximize profit
B Maximize EPS
C Maximize their salary
D Maximize shareholder wealth

Attempting to maximize profit or EPS can lead to


short term decision making that may not be optimal
in the long run. Maximizing shareholder wealth is
considered the same as maximizing share price.

Question 2

Answer 2

Which of the following is not considered a valid objective for


management of a corporation to attempt to achieve?

Correct answer is D

A Maximize the cash flow to the organization


B Maximize shareholder wealth
C Create economic value added (EVA)
D Maximize management compensation

There exists an agency issue within the corporate


form of ownership where shareholders own the
corporation but professional managers run the
entity hopefully with the best interests of the
shareholders at the forefront.

Business Environment & Concepts > Financial Environment/Overview > Financial Institutions

Financial Institutions
Question 1

Answer 1

Which of the following is not considered an advantage of the


corporate form of business organization.

Correct answer is A

A Taxation
B Limited Liability
C Infinite life
D Transferability of ownership

Corporations are subject to double taxation. The


corporation pays taxes on income and dividends
distributed to shareholders are taxed again as
capital gains.

Question 2

Answer 2

Which of the following institutions is not considered to be a


financial intermediary.

Correct answer is D

A Commercial bank
B Pension fund
C Life insurance company
D All of the above are financial intermediaries

Additional financial intermediaries include savings


and loans, mutual savings banks, credit unions,
and mutual funds.

Question 3

Answer 3

A retirement plan that promises to pay a specific benefit to the


beneficiaries is a (an)

Correct answer is C

A Defined contribution plan


B 401(k) plan
C Defined benefit plan
D 403(B) plan

Question 4
Which of the following is not a U.S. government agency?
A Government Employees Insurance Company
B Federal National Mortgage Association (Fannie
Mae)
C Federal Home Loan Mortgage Corporation
(Freddie Mac)
D Federal Housing Administration

401(k) and 403(B) are examples of defined


contribution plans. 403(B) is for non profits. In a
defined contribution plan the beneficiary and
employer make contributions and the beneficiary
makes investment decisions that determine the
value and benefits the plan can payout.

Answer 4
Correct answer is A
GEICO is an insurance company. The other three
originate (FHA) or provide the secondary market
(Fannie Mae and Freddie Mac) for mortgages in
the U.S.

Question 5

Answer 5

A fund where investors buy shares from and sell shares to the
fund at the funds net asset value (NAV) is known as what
type of fund?

Correct answer is B

A Closed end fund


B Open end fund
C Sector fund
D Exchange trade fund (ETF)

An open end fund often referred to as a mutual


fund. The size of an open ended fund changes
based on investor demand. With a closed end fund
a fixed number of shares are issued to investors
and they trade with each other on the secondary
market at prices that often differ from the funds
NAV.

Business Environment & Concepts > Financial Environment/Overview > Financial Institutions

Question 6

Answer 6

Endowment funds

Correct answer is C

A Are an example of a defined contribution plan


B Are offered by state and local governments
C Are funded by contributions to charitable and/or
educational institutions.
D Have a very high tax rate

Endowment funds are found at charitable and


educational institutions. They are not subject to
federal taxation.

Business Environment & Concepts > Financial Environment/Overview > Financial Markets

Financial Markets
Question 1

Answer 1

Markets where securities trade between investors after they


have been issued by a corporation are

Correct answer is B

A Primary markets
B Secondary markets
C Futures markets
D Spot markets

The primary market is where the initial public


offering takes place between the issuer and the
public. Spot and future markets refer to the timing
of a transaction.

Question 2

Answer 2

Which of the following is not a stock exchange.

Correct answer is D

A New York Stock Exchange (NYSE)


B American Stock Exchange (AMEX)
C Boston Stock Exchange
D NASDAQ

An exchange requires a physical/tangible location.


NYSE and AMEX are national exchanges. Boston
is a regional exchange. NASDAQ is a computer
based trading network that lacks a single physical
location.

Question 3

Answer 3

The original maturity of a Treasury bill is

Correct answer is C

A One to ten years


B Greater than 10 years
C One year or less
D Five years or less

Treasury notes have a maturity from one to 10


years. Treasury bonds have maturity great than 10
years.

Question 4

Answer 4

The purchase and sale of commodities for current delivery is


known as which market?

Correct answer is A

A Spot market
B Futures market
C Money market
D Capital market

The futures market involves setting a price today


for delivery at a future time period. The money
market is a market for securities with a maturity of
one year or less. The capital market is for longer
term investment products.

Question 5

Answer 5

A NYSE member acting as a dealer in a small number of


securities on the exchange is the definition of a (an)

Correct answer is C

A Floor trader
B Commission broker
C Specialist
D Market maker

This is a technical question. A specialist makes a


market in a stock on the NYSE. In the OTC market
the same function is provided by market
makers. The appropriate term for the NYSE is
specialist.

Business Environment & Concepts > Financial Environment/Overview > Financial Markets

Question 6

Answer 6

What is a call option?

Correct answer is B

A The obligation to sell a security at a specified price


in the future.
B The right to purchase a security at a specified
price for a defined period of time.
C The right to sell a security if a specified event
occurs.
D The obligation to provide specified information to
holders of a security if certain events take place.

A call option gives the holder the right to purchase


a security at a specified price for a certain period of
time (often three months or less). For example, if
the price of company X stock is $30, an investor
might buy a call option that would allow the
purchase of company X stock for $32 per share
over the next 3 months. If the price goes to $40 per
share, the investor has made a large profit from a
small investment.

Business Environment & Concepts > Financial Environment/Overview > Interest Rates

Interest Rates
Question 1

Answer 1

What is the normal shape of the yield curve.

Correct answer is A

A Upward sloping
B Inverted
C Flat
D Humped

The yield curve is a graph of the relationship


between bond yields and maturity. An upward
sloping curve provides investors with a higher
return to compensate for taking greater risk by
investing in longer maturity bonds.

Question 2

Answer 2

The annual rate of interest actually being earned or charged is


the definition of the

Correct answer is D

A Nominal rate
B Stated rate
C Annual percentage rate (APR)
D Effective annual rate (EAR)

Nominal and stated rates are different names for


the rate quoted in the contract. APR is equal to the
periodic rate times the number of periods per year.

Question 3

Answer 3

A bank charges a rate of 1.5% per month on their credit card.


What Effective Annual Rate (EAR) are they charging their
customers.

Correct answer is B

A 18%
B 19.60%
C 1.50%
D 21.60%

Question 4

EAR is equal to (1+ APR/12) to the 12th power


minus 1. (1+.18/12) to 12th power minus 1. 19.6%
APR is the periodic rate (1.5) times the number of
periods per year (12) or 18%

Answer 4
Correct answer is B

A headline in a newspaper recently proclaimed that LIBOR


reflects the price of funds in the international market. What is
LIBOR?
A Lowest Incremental Borrowing Ordered Rate
B London Interbank Offered Rate
C Lichtenstein Interest Based Offering Rate
D Late Invested Bank Overnight Rate

According to Wikipedia, the London Interbank


Offered Rate (or LIBOR) is a daily reference rate
based on the interest rates at which banks borrow
unsecured funds from other banks in the London
wholesale money market (or interbank market). It is
roughly comparable to the U.S. Federal funds rate.

Question 5

Answer 5

The nominal risk free rate of interest is a function of two


factors. They are:

Correct answer is B

A The prime rate plus inflation


B The real risk free rate plus inflation
C The treasury bill rate plus inflation
D The treasury bond rate plus inflation

Business Environment & Concepts > Financial Environment/Overview > Interest Rates

Question 6
Assume that the newspaper indicates that the Federal Reserve
has purchased a large quantity of US government securities.
Why was this action likely taken?
A It is a way to increase the monetary supply and,
therefore, put downward pressure on interest rates.
B It restricts the money that can be loaned by the
banks under its jurisdiction.
C It forces investors to find other investments, an
action which often causes a steep rise in the price of
gold.
D It expands the power of the Federal Reserve to
regulate unemployment rates for a short period of time.

Answer 6
Correct answer is A
One method by which the monetary supply can be
regulated is through open-market operations.
Federal Reserve Bank deposits can be used to
buy government securities as a way of raising the
supply of money. When more money is available,
it is easier to obtain and interest rates usually fall.

Business Environment & Concepts > Time Value of Money > Annuities/Perpetuities

Annuities/Perpetuities
Question 1

Answer 1

You have been offered $20,000 per year forever. This


represents what type of cash flow.

Correct answer is C

A Annuity
B Annuity due
C Perpetuity
D Consol

An annuity and annuity due are equal cash flows


over a specified period of time. A consol is a
perpetual bond issued by the British government.

Question 2

Answer 2

You purchased a preferred stock that pays a $3 annual divided.


When you bought the stock the required rate of return was
10%. You sold the stock when the required rate of return was
8%.

Correct answer is A

A you made a profit on the transaction


B you lost money on the transaction
C you broke even
D you are not sure what happened to you.

The value or price when you bought was the


dividend divided by the required rate or 3/.10 =
$30. When you sold the price was $3/.08 = $37.5.
You made a profit of $7.50.

Business Environment & Concepts > Time Value of Money > Present Value of a Future Amount

Present Value of a Future Amount


Question 1

Answer 1

You are to receive $1,000 per year for the next ten years. You
are better off, in terms of time value of money, receiving the
money

Correct answer is B

A At the end of each year


B At the beginning of each year.
C Spread equally over 12 months
D It does not matter when you receive the funds.

The sooner you receive payment the greater the


present value of the cash flow.

Business Environment & Concepts > Time Value of Money > Future Value of an Amount

Future Value of an Amount


Question 1

Answer 1

You are trying to decide at which bank you want to open a


savings account. You have four options. All four banks pay 6%
interest but compound differently. Which compounding will
result in you having more money in your account after five
years.

Correct answer is C
The more often a bank compounds the more
money that will be in your account. Compounding
involves paying interest on principal and previously
credited interest.

A Annual compounding
B Semi-annual compounding
C Quarterly compounding
D No compounding

Question 2
If you deposit $1,000 in a savings account that pays 8%
interest compounded quarterly, how much money will be in the
account at the end of 5 years? Future Value Factors for a
single amount of $1 in five periods is 1.4693 (8%), 1.2167
(4%), and 1.1041 (2 percent). Ten periods is 2.1589 (8%),
1.4802 (4%), and 1.2190 (2%). Twenty periods is 4.6610 (8%),
2.9111 (4%) and 1.4859 (2%)

Answer 2
Correct answer is C
Quarterly compounding requires that you look up
one fourth the interest rate (2%) and four times the
periods (five years is 20 quarterly periods). 2% and
20 periods results in a factor of 1.4859 times
$1,000 or $1,485.90.

A $1,469.30
B $1,480.20
C $1,485.90
D $2,159.90

Question 3

Answer 3

If you put $1,000 in a bank today at an interest rate of 10%


compound interest, how much money would you have in the
account at the end of year 5?

Correct answer is B

A $1,500
B $1,610.50
C $1,000
D $1,755.12

Future value problems can be solved without


financial calculators or tables. The future value of a
single sum is that amount times one plus the
interest rate raised to the power of the number of
periods. FV=PV (1 + i)n 1000(1.10)5

Question 4

Answer 4

If you place $1,000 in a bank account earning 8% compound


interest, approximately how many years will it take for your
initial deposit to double in value?

Correct answer is B

A 7 yrs
B 9 yrs
C 10 yrs
D 20 yrs

A short cut estimate (very close estimate) for the


time it takes for a sum to double in value is the rule
of 72. Take 72 and divide by the interest rate to
calculate the time it takes to double in value.
72/.08=9 yrs

Business Environment & Concepts > Time Value of Money > Future Value of an Amount

Question 5

Answer 5

You deposit $1,000 at the beginning of each of the next 5


years in an account earning 8% interest. How much money will
be in the account at the end of the 5th year? 8% FV Annuity 4
yrs 4.5061 5 yrs 5.8666 6 yrs 7.3359

Correct answer is D

A $4,506.10
B $5,866.60
C $7,335.90
D $6,335.93

This is a little tricky. The tables are set up for


end-of-the-year payments. Beginning of the year
payments means each payment has earned 1 year
extra interest which is why we want the 5 year
factor (5.8666) and multiple by the cash flow
($1,000) and get $5,866.60 and multiply by 1.08 for
the extra years interest.

Business Environment & Concepts > Time Value of Money > Present Value of a Future Sum

Present Value of a Future Sum


Question 1

Answer 1

The two factors that determine the present value of a cash flow
are the timing of the cash flow and the discount rate. Which
combination of timing and discount rate will result in the
highest present value.

Correct answer is D

A Discount rate Period received 10% 10 Years


B Discount rate Period received 5% 10 Years
C Discount rate Period received 10% 5 Years
D Discount rate Period received 5% 5 Years

A lower discount rate always results in a higher


present value. The sooner a cash flow is received
the more it is worth in present value terms. 5 years
and 5% will result in the highest present value

Business Environment & Concepts > Time Value of Money > Loan Amortization

Loan Amortization
Question 1

Answer 1

A client has taken out a 9% five year loan for $5,000. The
annual payment is $1,450. Since only the interest is tax
deductible it is necessary to amortize the loan. What is the
interest amount in year two?

Correct answer is B

A $450
B $360
C $270
D $810

Year 1 payment is $1,450. Principal balance is


$5,000. So 9% of ($5,000) or $450 is interest and
the remaining $1,000 pays down the principal to
$4,000. In year 2 interest is 9% ($4,000) or ($360).

Business Environment & Concepts > Security Valuation > Bond Valuation

Bond Valuation
Question 1

Answer 1

Omega Company has an outstanding 10 year bond rated AA


paying a 7.5% coupon rate (paid semi annually). The current
market rate of interest on 10 year AA rated bonds is 8%. Given
this information, the price of Omegas bond in relationship to
its face (par) value will be:

Correct answer is A

A The bonds price will be lower than its face value


B The price will be greater than face value
C Price will be equal to face value. The bond will sell
at par value.
D Not enough information is available to determine
the relationship

Question 2
May Corporation has $100 million of 10 years, AA rated, 8%
coupon (paid semi annually) bonds outstanding. The bonds
have a face value of $1,000 per bond. The current market rate
of interest for 10 years, AA rated bonds is 6%. Calculate the
current market price of a May Corporation bond. The present
value of $1 for various time periods and interest rates for those
periods is as follows: 5 periods: .863 (3 percent), .747 (6
percent), and .702 (8 percent); 10 periods: .744 (3 percent),
.558 (6 percent), and .492 (8 percent); 20 periods: .554 (3
percent), .312 (6 percent), and .239 (8 percent). The present
value of an ordinary annuity of $1 is also listed here for various
time periods and interest rates for those time periods: 5
periods: 4.580 (3 percent), 4.212 (6 percent), and 4.055 (8
percent); 10 periods: 8.530 (3 percent), 7.360 (6 percent), and
7.002; (8 percent); 20 periods: 14.877 (3 percent), 11.470 (6
percent), and 10.565 (8 percent). What does each $1,000
bond sell for?

Bonds are priced based on maturity and default


risk (rating). When a bond pays a coupon rate
below the prevailing market rate, it will sell at a
discount. If the coupon is greater than the current
market rate, the bond will sell at a premium. When
the coupon rate is equal to the market rate, the
bond will sell at par value.

Answer 2
Correct answer is C
Since the stated coupon rate of interest is greater
than the market rate, the bond will sell at a
premium. Because interest is paid semi-annually,
the yield rate will be 6 percent for a full-year but
only 3 percent for a six-month period. Because a
period of time is six-months and the bond matures
in 10 years, the computation uses 20 periods. The
$1,000 face value times .554 gives the present
value of the face value or $554. The $1,000 face
value times the stated rate of 8 percent for a
half-year period gives you interest payments of
$40. That is an ordinary annuity and should be
multiplied times 14.877 which gives a present
value of $595.08. Add the two present values and
you issue the bond for $1,149.08 ($554.00 plus
$559.08).

A $855.41
B $867.90
C $1,149.08
D $1,165.50

Question 3

Answer 3

If interest rates rise, what will happen to the price of a bond


and its yield to maturity (YTM)?

Correct answer is D

A Price increases and YTM increases


B Price increases and YTM decreases
C Price decreases and YTM decreases
D Price decreases and YTM increases

Bond prices move more inversely with interest


rates. When interest rates increase, bond prices
fall (decrease). A lower bond price will result in
higher current yields and eventual capital gains
which increases the bonds YTM.

Business Environment & Concepts > Security Valuation > Bond Valuation

Question 4

Answer 4

An investor can purchase a municipal bond yielding 7%. She is


in a 28% federal tax bracket and pays a state tax rate of 5%.
The municipal bond is issued by the state in which she resides.
Calculate her taxable equivalent yield (the rate she needs to
earn on a corporate bond to have the same after tax yield as
the municipal bond)

Correct answer is C

A 7%
B 9.72%
C 10.45%
D 33%

Question 5
The June Corporation has $100 million of 10 years, AA rated,
7% coupon (paid semi annually) bonds outstanding. The bonds
have a face value of $1,000 per bond. The current market rate
of interest for 10 years, AA rated bonds is 6%. Calculate the
current market price of a May Corporation bond. The present
value of $1 for various time periods and interest rates for those
periods is as follows: 5 periods: .863 (3 percent), .747 (6
percent), and .713 (7 percent); 10 periods: .744 (3 percent),
.558 (6 percent), and .508 (7 percent); 20 periods: .554 (3
percent), .312 (6 percent), and .258 (7 percent). The present
value of an ordinary annuity of $1 is also listed here for various
time periods and interest rates for those time periods: 5
periods: 4.580 (3 percent), 4.212 (6 percent), and 4.100 (7
percent); 10 periods: 8.530 (3 percent), 7.360 (6 percent), and
7.024; (7 percent); 20 periods: 14.877 (3 percent), 11.470 (6
percent), and 10.594 (7 percent). What does each $1,000
bond sell for?
A $684.89
B $713.45
C $990.58
D $1,74.70

Municipal bonds are exempt from federal taxation.


They are also exempt from state taxes if the bond
is issued by the state in which the taxpayer
resides. Corporate bonds are fully taxed. The TEY
is equal to the municipal bond rate divided by 1
minus the tax rate. .07/(1-.33 or .07/.67=10.45%

Answer 5
Correct answer is D
In this case, you do not actually need to make a
present value computation. Since the stated
coupon rate of interest is greater than the market
rate, the bond will sell at a premium and only one
answer is a number above the face value.
However, because interest is paid semi-annually,
the yield rate will be 6 percent for a full-year but
only 3 percent for a six-month period. Because a
period of time is six-months and the bond matures
in 10 years, the computation uses 20 periods. The
$1,000 face value times .554 gives the present
value of the face value or $554. The $1,000 face
value times the stated rate of 7 percent for a
half-year period gives you interest payments of
$35. That is an ordinary annuity and should be
multiplied times 14.877 which gives a present
value of $520.70. Add the two present values and
you issue the bond for $1,074.70 ($554.00 plus
$520.70).

Business Environment & Concepts > Security Valuation > Risk and Return

Risk and Return


Question 1

Answer 1

XYZ, Inc. has a beta coefficient of 1.5x. The current risk free
rate of return is 4% and the return on the overall stock market
is expected to be 10%. What is the expected return on XYZ's
stock using the capital asset pricing model (CAPM).

Correct answer is D

A 10%
B 15%
C 6%
D 13%

The CAPM formula is E(R) = RF + B [RM-RF]


Where E(R) = expected return on the security RF =
risk free rate of return RM = the return on the
market So E(R) = 4 + 1.5 [10-4] = 13%

Question 2

Answer 2

When comparing two investment alternatives that have


different expected returns and standard deviations, what
statistical measure should be used to select the superior
option.

Correct answer is D

A Correlation coefficient
B Semi-variance
C Covariance
D Coefficient of variation

When two investments have the same expected


return, the project with the lower standard
deviation is preferred. When projects have different
expected returns the coefficient variations (v) is
excluded. V = ?/E(R) V = standard deviation
divided by expected return The project with the
lower coefficient of variation is preferred.

Question 3

Answer 3

Bond ratings by firms such as Moodys or Standard and


Poors measure what type of risk.

Correct answer is C

A Interest rate risk


B Market risk
C Default risk
D Inflation risk

The highest rating is AAA. Bonds related BBB and


above are considered investment grade. Bonds
rated below BBB are called high yield or junk
bonds.

Question 4

Answer 4

The risk premium is comprised of all of the following except

Correct answer is D

A Business risk
B Financial risk
C Liquidity risk
D Systematic risk

The risk premium is comprised of five components:


business risk, financial risk, liquidity risk, currency
risk, and country risk.

Question 5

Answer 5

The security market line (SML) graphs the relationship between

Correct answer is D

A Expected return and standard deviation


B Business risk and financial risk
C Systematic risk and unsystematic risk
D Expected return and systematic risk (beta)

The SML graphs the relationship between


expected return and risk as measured by the beta
coefficient. The beta coefficient measures
systematic risk. The equation for the SML is the
capital asset pricing model (CAPM).

Business Environment & Concepts > Security Valuation > Risk and Return

Question 6

Answer 6

The number of times interest is earned helps to show how


easily an organization can meet its required interest payments.
Which of the following is the computation of this figure?

Correct answer is C

A Net income/interest expense


B Earnings before income taxes/interest expense
C Earnings before interest and taxes/interest
expense
D Gross profit/interest expense

Times interest earned takes the net income for the


period and eliminates the interest and tax figure
within that amount. This residual figure is divided
by the reported interest expense to arrive at the
number of times interest was earned during the
period.

Question 7

Answer 7

When comparing two investment alternatives that have


different expected returns and standard deviations, what
statistical measure should be used to select the superior
option.

Correct answer is D

A Correlation coefficient
B Semi-variance
C Covariance
D Coefficient of variation

When two investments have the same expected


return, the project with the lower standard
deviation is preferred. When projects have different
expected returns the coefficient variations (v) is
excluded. V = ?/E(R) V = standard deviation
divided by expected return The project with the
lower coefficient of variation is preferred.

Business Environment & Concepts > Security Valuation > Stock Valuation

Stock Valuation
Question 1

Answer 1

Using the constant growth dividend discount model, calculate


the value of a share of stock given the following information.
Do (current dividend) = $1.00 Ke (required rate of return on the
stock) = 9% G(forecasted rate of growth in
earnings%dividends) = 6%

Correct answer is B
Constant growth DDM models state that V=D?
(K-G) Where Di is the dividend 1 period from no D^
=Do (1+G) =1.00(1.06) or $1.06 1.06/(.09-03) =
$35.33

A $33.33
B 35.33
C 11.11
D 16.67

Question 2
ABC Company has 2 million shares of preferred stock
outstanding. The stock pays a $3 annual dividend. Assuming
investors have an 8% required rate of return, what is the value
of a share of ABC's preferred stock?

Answer 2
Correct answer is C
A preferred stock is perpetuity. The value of
perpetuity is calculated by dividing the cash flow
by the required rate of return. $3/.08 = $37.50

A $3
B $24
C $37.50
D Cannot calculate

Question 3

Answer 3

What happens to the value of a share of common stock when


the firms projected rate of growth in earnings and dividend is
expected to increase?

Correct answer is A

A Value increases
B Value decreases
C Value remains unchanged
D The growth rate does not impact valuation

Value of a share of common stock is calculated by


using the DDM (Dividend Discount Model). Value
is equal to D1/K-g). When G increases so does
value.

Question 4

Answer 4

Which type of security can be valued by dividing its annual


dividend by the required rate of return of investors?

Correct answer is B

A Common stock
B Preferred stock
C Bonds
D Convertible bonds

Preferred stock is an example of perpetuity. To


value perpetuity, you divide the annual dividend by
the required rate of return.

Business Environment & Concepts > Security Valuation > Stock Valuation

Question 5

Answer 5

When valuing a stock using the DDM (Dividend Discount


Model), it is necessary to estimate the firms forecasted
growth rate (G) in earnings and dividends. Given the following
information: EPS $1, Dividend .30, ROE 10%, ROA 5%,
Calculate the firms internal growth rate (G) in earnings:

Correct answer is C

A 10%
B 5%
C 7%
D 3%

G= ROE x retention rate. Retention rate is the


percentage of each dollar earned the firm keeps. It
is also pay out 1 minus the payout rate. The firm
has EPS of $1 and paid out $.30 in dividends. Its
rate is 30% and its retention rate is 70%. 10%
ROE x 70% retention is 7%.

Question 6

Answer 6

A firms price to earnings ratio (P/E) is determined by all of the


following except

Correct answer is D

A Required rate of return


B Expected growth rate of dividends
C Dividends payout ratio
D Risk free rate of return

P/E = dividend payout ratio divided by the required


rate of return minus the expected growth rate in
dividend. Payout/ (required return- growth rate)

Question 7

Answer 7

A company reports sales of $800,000, operating expenses of


$500,000, cash of $44,000, stockholders equity of $345,000,
and earnings per share of $3.00. The company also has
$575,000 in noncurrent liabilities and pays a cash dividend of
$.75 per share to its 20,000 shares of outstanding common
stock. What is the companys dividend payout ratio?

Correct answer is B

A 16 percent
B 25 percent
C 28 percent
D 34 percent

The dividend payout ratio is the companys cash


dividend per share ($.75) divided by the earnings
per share for the same period ($3.00). It is the
percentage of the reported income that goes to the
owners in the form of dividends. Here, the dividend
payout ratio is 25 percent or $.75/$3.00.

Business Environment & Concepts > Security Valuation > Common Stock

Common Stock
Question 1

Answer 1

When attempting to value stock that does not pay a dividend


which of the following techniques is not a valid valuation
approach?

Correct answer is D

A Multiple of earnings
B Multiple of cash flow
C Multiple of book value
D Super normal growth DDM

Valid methods include multiples of sales,


commissions, and EBITDA in addition to A-C. The
super normal DDM is a valuation method but it
requires the firm pay a dividend to be utilized.

Question 2

Answer 2

XYZ Companys current stock price is $50 per share. Next


years dividend is forecasted to be $2 per share. The firms
growth rate in dividends is expected to be 7% per year. The
risk free rate of interest is 4.5%. What is XYZs after tax cost
of issuing common stock ignoring any transactions costs?

Correct answer is D

A 4%
B 7%
C 11.50%
D 11%

The after tax cost of common stock to the firm is


equal to D1 (next years dividend)
_____________________ + growth rate in
dividends Net proceeds (price transaction costs)
2 50 + .07 = 11%

Question 3

Answer 3

Which of the following is not a generally accepted method of


calculating a firms cost of retained earnings?

Correct answer is D

A CAPM approach
B Dividend yield plus growth approach (DCF)
C Bond yield plus risk premium approach
D Seat of the pants estimate approach

Business Environment & Concepts > Working Capital Management > Derivatives/Options

Derivatives/Options
Question 1

Answer 1

The right to buy shares at a fixed price for a fixed period of time
is the definition of a(n).

Correct answer is C

A Futures contract
B Forward contract
C Call option
D Put option

A put is the right to sell Forward and future


contracts involve parties agreeing today to deliver
or purchase a commodity or security at a specific
price on a specific future date.

Question 2

Answer 2

A convertible bond allows the bondholder the option to

Correct answer is A

A Exchange the bond for a fixed number of shares


of common stock
B Sell the bond back to the company for a fixed price

B is the definition of a putable bond. C is the


definition of a bond with warrants attached.

C Keep the bond and buy shares of stock at a fixed


price
D Buy additional bonds at a fixed price

Question 3

Answer 3

A security frequently attached to a bond that allows the holder


to buy stock in the company at a specified price is the definition
of a (an)

Correct answer is D

A Preferred stock
B Put option
C Call option
D Warrant

A put is the right to sell stock at a fixed price. A call


option is the right to buy stock at a fixed price, but
it is not attached to a bond.

Question 4

Answer 4

The Jiminez Corporation buys 10,000 shares of Warrington


Corporation at $23 per share because the CEO of Jiminez
expects the price of Warrington to rise sharply over the next
few months. However, the CEO is concerned that economic
conditions might also cause the price of the shares to fall
quickly and sharply. He is concerned about explaining a big
loss if that might happen. How is Jiminez most likely to protect
itself from any such severe market drops?

Correct answer is C

A Jiminez could also buy stock warrants in


Warrington.
B Jiminez could also buy a call option for 10,000
shares of Warrington at a properly designated price.
C Jiminez could also buy a put option for 10,000
shares of Warrington at a properly designated price.
D Jiminez could also buy convertible bonds issued
by Warrington.

A put option allows Jiminez to sell the stock at a


specified price. For example, if the put option is for
$22, then Jiminez can sell the stock at that price
even if it drops much lower. Although there is a
cost for the put option, the loss is limited to a
specified amount. All of the other possibilities
(warrants, call option, and convertible bonds) allow
for the acquisition of more shares which does little
(unless the conversion price is extremely low) to
help protect against a severe fall in stock price.
They are all considered strategies to be used when
stock prices are expected to rise.

Business Environment & Concepts > Working Capital Management > Cash and Marketable Securities

Cash and Marketable Securities


Question 1

Answer 1

Which of the following is not an example of a money market


security.

Correct answer is C

A Commercial paper
B Bankers acceptance
C Treasury note
D Negotiable certificate of deposit

Money market securities have a maturity of one


year or less. Treasury notes have a maturity of
1-10 years. Treasury bills are an example of a
money market security.

Question 2

Answer 2

Which of the following is not a rationale for firms to hold cash.

Correct answer is D

A Transactions balance
B Compensating balance
C Precautionary balance
D Trade balance

The four reasons to hold cash are: transactions to


meet day to day cash outflows, compensating
balances required by banks, precautionary
balances to meet unexpected events, and
speculative balances to take advantage of
opportunities.

Question 3

Answer 3

Given the following information calculate the firms cash


conversion cycle. Inventory conversion period 75 days;
Receivables collection period 40 days; Payables deferral
period 25 days; Credit period 20 days

Correct answer is A
Cash cycle is inventory age plus receivables minus
payables. 75+40-25= 90 days.

A 90 days
B 85 days
C 60 days
D 10 days

Question 4
The Seeger Corporation is considering beginning to use
concentration banking. What does that indicate?
A Seeger will borrow money only from one banking
company in order to get a lower rate.
B Seeger will instruct its customers to mail
payments to a bank in their local area rather than to its
headquarters.
C Seeger will maintain all checking and other
accounts in only one bank in order to gain more interest
income.
D Seeger will begin to serve as its own banker for
most transactions to reduce the amounts paid for service
charges.

Answer 4
Correct answer is B
In managing cash, companies like to receive
payments from customers as quickly as possible.
Mail sent over a long distance can take several
days to arrive. Therefore, some companies have
payments sent by customers directed to local
banks around the country that can electronically
forward the money to the company. That can
speed up cash collection by several days. The
practice is referred to as concentration banking.

Business Environment & Concepts > Working Capital Management > Short Term Credit

Short Term Credit


Question 1

Answer 1

Which of the following is not a component of a firms credit


policy.

Correct answer is D

A Credit period
B Credit standards
C Collection policy
D Aging schedule

Credit policy has 4 components; Credit period when payment is due; Credit standards - criteria to
grant credit; Collection policy- enforcement of the
collection process; Discount - to speed up
payments

Question 2

Answer 2

Which of the following is not an advantage of short term debt


over long term financing options.

Correct answer is D

A Flexibility
B Cost
C Speed that funds can be obtained
D Risk

Short term debt is considered to be riskier than


long tern debt since it has to be either be paid off
or refinanced on a regular basis. If rates increase
the cost of borrowing will increase.

Question 3

Answer 3

If a supplier is not satisfied that a customer has the financial


ability to pay, they may require that the customer get a bank to
guarantee payment to the supplier. This is accomplished by
utilizing a

Correct answer is B

A Line of credit
B Letter of credit
C Bankers acceptance
D Revolving line of credit

A letter of credit states that the bank will guarantee


payment if the customer defaults. Lines of credit
allow the customer to borrow from the bank if
necessary, but do not guarantee payment to the
supplier. A bankers acceptance is a promissory
note backed by a letter of credit that has become a
money market security.

Question 4

Answer 4

Money kept by a firm in a low or zero interest bearing account


as a condition of a loan agreement is the definition of a (an)

Correct answer is A

A Compensating balance
B Letter of credit
C Line of credit
D Loan guarantee

The existence of a compensating balance is one


way that the actual cost of a loan exceeds the
stated contractual rate. A letter of credit puts the
lenders credit behind that of the borrower to
guarantee payment. A line of credit is a
prearranged borrowing limit.

Business Environment & Concepts > Working Capital Management > Inventory2

Inventory2
Question 1

Answer 1

An inventory strategy implemented to improve return on


investment by reducing in-process inventory and lowering
inventory carrying costs is the definition of

Correct answer is D

A Supply chain management


B Economic order quantity point (EOQ)
C Six Sigma
D Just in time (JIT)

JIT was introduced by the Japanese automobile


industry in order to reduce inventory and its
carrying costs. Instead of holding inventories of
engines, tires, etc. the manufacturer subcontracts
with suppliers to provide the necessary inputs for
that days or weeks expected production.

Question 2

Answer 2

A producer of single engine aircraft plans to build 1,600 aircraft


in the coming year. The fixed cost of placing an order is $50
regardless of order size. The variable cost of carrying inventory
is one dollar per unit per period on the average inventory
balance. In order to optimize the total cost of inventory, how
many aircraft should the firm order and how many times should
they place an order?

Correct answer is A

A 400 engines and 4 orders


B 800 engines and 2 orders
C 200 engines and 8 orders
D 100 engines and 16 orders

Economic Order Quantity points (EOQ) tells you


how many engines to order at one time. It is
determined [as the square root of 2 times the
annual demand (1,600 units) times the cost of
placing one order ($50)] divided by the cost of
carrying a unit for a year $1. That is (2 x 1,600 x
50) or 160,000. That is then divided by $1 so that it
stays 160,000. The square root of 160,000 is 400.
That is the number of units that should always be
ordered. Because 1,600 are needed, the orders of
400 are placed four times per year.

Question 3

Answer 3

A manufacturer of single engine aircraft operates 365 days per


year and produces 3,650 aircraft per year. Its engine supplier
takes 5 days from the time an order is placed to deliver
engines. Assuming the manufacturer does not wish to carry a
safety stock, at what level of engine inventory should they
place an order (reorder point) for new engines to ensure that
production is not interrupted?

Correct answer is C
In the absence of a safety stock, reorder point is
equal to daily usage times the time it takes for a
supplier to deliver. Daily usage is 3,650/365 or 10
x 5 days to deliver (lead time) is equal to 50
engines as a reorder point.

A 10 engines
B 365 engines
C 50 engines
D 730 engines

Question 4

Answer 4

An example of short term secured loans are those used to


purchase inventory. The loans are in three forms. Which of the
following is not one of the forms of inventory loans?

Correct answer is D

A Blanket inventory loan


B Trust receipt loan
C Warehouse financing
D Broad form lien

A blanket lien covers an entire group of usually low


cost homogeneous items (tires). With a trust
receipt the lender holds title and the borrower
keeps possession of the collateral (automobile
dealers). With warehouse receipts the lender
controls title and takes possession of the collateral

Business Environment & Concepts > Working Capital Management > Inventory2

Question 5

Answer 5

The Argentina Company started the year with inventory of


$52,000. During the year, the company bought another
$409,500 in inventory. At the end of the year, the company
takes a physical inventory and finds $60,000 still on hand
because revenues for the period were $525,000. On the
average, how long does the company take to sell an item of its
inventory?

Correct answer is B

A 45.8 Days
B 50.9 Days
C 53.2 Days
D 55.7 Days

To compute the number of days in inventory (the


number of days it takes to sell an item), the
company should find the average cost of goods
sold per day. Here, to do that, the accountant must
first compute the cost of goods sold which is
$52,000 (beginning inventory) plus $409,500
(purchases) less $60,000 (ending inventory) or
$401,500. The daily average is found by dividing
that number by 365 (although the exam might tell
you to use 360) which indicates that the company
sells inventory costing $1,100 each day. That
figure is then divided into the average inventory for
the year of $56,000 (the average of $52,000 and
$60,000). The number of days in inventory is
$56,000/$1,100 or 50.9 days.

Business Environment & Concepts > Working Capital Management > Accounts Receivables

Accounts Receivables
Question 1

Answer 1

The number of days between the time a firm acquires raw


materials or inventory until it collects cash from the sale of its
finished goods is called the

Correct answer is B

A Cash conversion cycle


B Operating cycle
C Inventory period
D Account receivable period

Operating cycle is equal to the average age of


inventory (inventory period) plus the average age
of accounts receivable (receivables period). Cash
conversion cycle would subtract the average age
of accounts payable from the operating cycle.

Question 2

Answer 2

When a firm sells its accounts receivable at a discount from


face value the process is referred to as

Correct answer is C

A Pledging
B Aging
C Factoring
D Discounting

Factors are firms that buy accounts receivable from


firms at a discounted price. This provides the seller
with cash flow advantages and can transfer the
costs associated with credit granting and
collections to the factor.

Question 3

Answer 3

A company is struggling because it is having difficulty collecting


its receivables in a timely manner. It started the year with
$300,000 in accounts receivable but ended with $660,000.
During the year, the company made sales on credit of
$9,125,000. On the average, how long did the company take
to collect its accounts receivable during this period?

Correct answer is A

A 19.2 Days
B 25.4 Days
C 27.6 Days
D 29.8 Days

This company makes sales of $25,000 each day


($9,125,000/365 days). The average receivable
balance for the year was $480,000 ([$300,000 +
$660,000]/2). Thus, the average receivable takes
19.2 days to collect: $480,000/$25,000.

Business Environment & Concepts > Working Capital Management > Accounts Payable

Accounts Payable
Question 1

Answer 1

Your supplier gives you credit terms of 2/10 net 30. This
means that if you pay within 10 days you take a 2% discount. If
not, the balance is due in full within 30 days. What is the
annual percentage cost to you of not taking the discount and
paying on the 30th day?

Correct answer is B

A 2%
B 37.24%
C 36%
D 24%

Your choice is to pay $.98 on the dollar on day 10


or $1 on day 30. The extra cost is .02/.98 or
.0204081. You save 20 days (30-10) by paying
later. To annualize the cost take 365 days and
divide by the days saved. 365/20=18.25 and
multiply this by the .0204081 percent cost (
.0204081) (18.25) = 37.24%

Question 2

Answer 2

The Madison Company takes 22 days to sell the average


piece of inventory (that is the inventory conversion period). It
takes another 27 days on the average to collect an account
receivable (that is the receivables conversion period). It takes
31 days to pay accounts payable on the average (that is the
payable conversion period). What is the cash conversion cycle
for this company?

Correct answer is A

A 18 days
B 49 days
C 53 days
D 80 days

The cash conversion cycle (which is sometimes


referred to as CCC) is the time it takes a company
to convert its resources into cash. It is actually the
total time to convert inventory into accounts
receivable and then collect those accounts less the
period it takes the company to then pay for the
inventory that it acquired. Hence, the computation
for the Madison Company would be 22 plus 27
minus 31 or 18 days.

Question 3

Answer 3

A company buys and sells widgets on credit. The inventory


conversion period is 31 days. The receivables conversion
period is 23 days. The accounts payable conversion period is
28 days. What is the cash conversion period?

Correct answer is B

A 23 days
B 26 days
C 54 days
D 82 days

The cash conversion period is the inventory


conversion period (31) plus the receivables
conversion period (23) less the payable
conversion period (28).

Business Environment & Concepts > Working Capital Management > Accounts Payable

Question 4
Lenit Corporation recently acquired widgets from a company in
the country of Becktle where the currency is the romtag.
Each romtag is currently worth $.40 and Lenit must pay
100,000 romtags in 60 days (value of $40,000). The
treasurer for Lenit believes that the value of the romtag is going
to go up over the next 60 days so that a more valuable
currency will have to be paid and money will be lost. What
action is the treasurer most likely to take?
A Acquire a forward exchange contract that allows
the company to buy 100,000 romtags in 60 days for
exactly $.41.
B Sell the widgets in the U. S. as quickly as possible
even if they must be sold at a loss.
C Buy the next batch of widgets in some other
country.
D Begin production of widgets in the U. S.

Answer 4
Correct answer is A
The fluctuation of currency values is always a
concern but one that can be managed through a
hedging arrangement. The company has a
100,000 romtag payable. To create a hedge, the
company needs to establish a 100,000 romtag
receivable. The forward exchange contract is set
up here so that the company will get 100,000
romtags in 60 days. That is a receivable. Any
change in the value of the romtag will cause equal
and offsetting changes in the receivable and the
payable. Thus, a gain on one cancels out a loss
on the other. The company does have to pay
$41,000 for the romtags that will be received which
is $1,000 more than the current value of the
liability. That is the cost of establishing the
hedge. The company knows that amount and has
judged it to be more reasonable than the
uncertainty of waiting to see what paying 100,000
romtags will cost the company in 60 days.

Business Environment & Concepts > Capital Budgeting/Financial Modeling > Investment Evaluation Methods

Investment Evaluation Methods


Question 1

Answer 2

Which of the following combinations of capital budgeting


decision rules is correct.

Correct answer is D

A Accept project if NPV >0 Accept if IRR < cost of


capital
B Accept if NPV cost of capital
C Accept if NPV >0 Accept if IRR > cost of capital

NPV is equal to the present value of the cash


inflows discounted at the cost of capital (10%) or
$1,039,531.65 minus the net investment of
$1,000,000 or $39,531.65. The present value of
the cash flow is $300,000 (3.240) = $972,000.
Answer A and C used the 6% cost of debt instead
of the cost of capital

D Accept if NPV Answer 1 Correct answer is C NPV


decision rule to accept if the NPV in dollars is greater
than zero. IRR decision rule to accept IRR if the IRR is
greater than the firm's cost of capital Question 2 Ajax is
considering investing in a project that will cost
$1,000,000 (net investment) today. The project has an
expected life of 4 years and will generate an annual after
tax cash flow of $300,000 per year at the end of each
year. The firms cost of debt is 6% and their cost of
capital is 9%. Present Value of an Annuity: 4 yrs
6%/3.465 and 9%/3.240. Calculate Ajax's net present
value (NPV). It is closest to:
A $1,039,531.65
B $972,000
C $39,531.65
D ($28,000)

Question 3

Answer 3

Ajax is considering investing in a project that will cost


$1,000,000 (net investment) today. The project has an
expected life of 4 years and will generate an annual after tax
cash flow of $300,000 per year at the end of each year. The
firms cost of debt is 6% and their cost of capital is 9%. The
project has an NPV of ($28,000). The projects Internal Rate
of Return (IRR) is:

Correct answer is C
Since the project has a negative NPV, this means
that the projects IRR is less than the firms cost
of capital (9%). Any project that is acceptable
based on NPV is also acceptable based on IRR. If
the NPV is positive, the projects IRR is above the
firms cost of capital.

A Greater than 9%
B Equal to 9%
C Less than 9%
D 12%

Question 4

Answer 4

Omegatron is contemplating investing in a project with a net


investment of $1m. The project will last five years with cash
flows of $300,000 per year. The present value of the cash
flows is $1,200,000 discounted at the firms WACC. Calculate
the projects benefit cost (profitability) index.

Correct answer is B

A 0.3
B 1.2
C $200,000
D4

B/C ratio is equal to the present value of the cash


flows divided by the net investment. 1.2m/1m or
1.20. An index greater than one (or equal to in
some cases) means that the project is acceptable.

Business Environment & Concepts > Capital Budgeting/Financial Modeling > Investment Evaluation Methods

Question 5

Answer 5

Given the following information, calculate the projects


payback period. Net Investment $500,000; Cash Flow - Yr 1
$150,000; Yr 2-$150,000; Yr. 3 $100,000; Yr 4 $200,000; Yr
100,000.

Correct answer is C

A 4 yrs
B 3 yrs
C 3.5 yrs
D 4.5 yrs

Payback is how long in years /months it takes the


firm to recoup its initial investment. After 3 years
$400,000 of the initial investment of $500,000 has
been recovered. An additional $100,000 is needed
to break even. Year 4 brings in $200,000 of which
is needed so 3.5 years is correct.

Question 6

Answer 6

The length of time required for an investments discounted


cash flow to equal its initial cost (net investment) is the
definition of which capital budgeting technique?

Correct answer is D

A Payback period
B Net present value
C Internal rate of return (IRR)
D Discounted payback period

Without the work discount this would be the


definition of payback period. NPV is evaluated in
dollars and IRR in percentages.

Question 7

Answer 7

The Modified Internal Rate of Return (MIRR) improves upon


the Internal Rate of Return (IRR) technique by addressing
what shortcomings of the IRR technique?

Correct answer is C

A Value additivity issue


B Multiple roots (answers) issue when cash flow
changes from positive to negative
C Reinvestment rate assumption for cash flows
D Dealing with mutually exclusive projects

A-C are limitations of IRR versus a NPV based


technique. IRR assumes reinvestment of each
cash flow at the IRR while NPV assumes
reinvestment at the cost of capital. MIRR allows
the reinvestment rate assumption to be modified
by the user.

Question 8

Answer 8

The equivalent annual annuity (EAA) sometimes called the


equivalent annual cost EAC) is a technique use to evaluate
projects that

Correct answer is D

A Have substantially different net investments


B Are mutually exclusive
C Have substantially different levels of risk
D Have different project durations (lives)

Standard capital budgeting techniques (NPV, IRR)


are not designed to compare projects with different
lives (a 3 yr versus 4 yr project). The EAA
calculates present value on an annual basis and
allows the analyst to compare projects with
different lives.

Business Environment & Concepts > Capital Budgeting/Financial Modeling > Investment Evaluation Methods

Question 9
Josiah Corporation is considering two investments. Investment
A will cost $10,000 today but pay back $14,000 in exactly two
years. Investment B will cost $20,000 today but pay back
$39,000 in exactly three years. The present value of $1
received in two years at a desired annual interest rate of 10
percent is .826. The present value of $1 received in three
years at a desired annual interest of 10 percent is .751. Which
of the following is true?
A Investment A has a positive net present value of
more than $3,600.
B Investment A has a positive net present value of
less than $1,100.
C Investment B has a positive net present value of
more than $9,100.
D Investment B has a positive net present value of
less than $6,500.

Answer 9
Correct answer is C
The net present value technique is a capital
budgeting method used to help evaluate the
wisdom of an acquisition. The present value of
future cash flows is determined at a desired rate
and compared to the initial cost. A positive net
present value means that the present value
exceeds the cost by the specified amount.
Investment A has a present value of $11,564
($14,000 times .826) for a positive net present
value of $1,564 ($11,564 less $10,000).
Investment B has a present value of $29,289
($39,000 times .751) for a positive net present
value of $9,289 ($29,289 less $20,000).

Keep working -- you CAN do it. You CAN pass the


CPA Exam.

Question 10

Answer 10

A company has a rule that an investment must earn an annual


effective income rate of 10 percent or more before the
investment can be made. Currently, the company is looking at
two investments. Investment A has a cost of $20,000 and its
expected cash flows have a present value based on a 10
percent annual income rate of $19,000. Investment B has a
cost of $34,000 and its expected cash flows have a present
value based on a 10 percent annual income rate of $33,000.
Which of the following statements is true?

Correct answer is D

A Both investments meet the companys criteria.


B Only Investment A meets the companys criteria.
C Only Investment B meets the companys criteria.
D Neither investment meets the companys criteria.

Investments should be made when they have a net


present value that is positive. The net present
value is the present value of the future cash flows
to be generated (computed at a desired rate) less
the cost to make the investment. Both of these
investments have a negative net present value of
$1,000 ($19,000 less $20,000 and $33,000 less
$34,000). They are not earning an annual income
rate of 10 percent because the resulting figure is
negative.

Business Environment & Concepts > Capital Budgeting/Financial Modeling > Investment Evaluation Methods

Question 11

Answer 11

On January 1, Year One, a company spends $60,000 on an


investment that will return $20,000 on each December 31 of
Years One, Two, and Three as well as an additional $30,000
on December 31, Year Three. The company has a desired rate
of return of 10 percent per year. The present value of $1 paid
in three periods at a 10 percent rate of return is .751. The
present value of an annuity due of $1 paid over three periods
at a 10 percent rate of return is 2.736. The present value of an
ordinary annuity of $1 paid over three periods at a 10 percent
rate of return is 2.487. Which of the following statements is
true?

Correct answer is B

A The investment has a positive net present value of


$ 7,590.
B The investment has a positive net present value of
$12,270.
C The investment has a positive net present value of
$16,860.
D The investment has a positive net present value of
$17,250.

A positive net present value indicates that the


present value of the future cash flows determined
based on a desired rate of return is higher than the
cost of making the investment. These specific cash
flows have a present value of $20,000 times 2.487
($49,740) plus $30,000 times .751 ($22,530) for a
total of $72,270. The first three payments are an
ordinary annuity since they begin one period after
the investment is bought. The cost of the
investment is $60,000 so that the net present value
is the excess $12,270. Investments with a positive
net present value are viewed as good investments.

Question 12

Answer 12

On January 1, Year One, the Aragoni Company is given the


opportunity to pay $50,000 today to get a four-year annuity
paying $20,000 per year for four years with each payment
made on December 31. The company has a desired rate of
return of 8 percent per year. The present value of a four-year
ordinary annuity of $1 at an 8 percent annual rate is 3.3121.
The present value of a four-year annuity due of $1 at an 8
percent annual rate is 3.5771. The present value of $1in four
years at an 8 percent annual rate is .7350. Aragoni believes
this to be a good investment. Which of the following
statements is true?

Correct answer is B

A The net present value is a positive $8,800.


B The net present value is a positive $16,242.
C The net present value is a positive $21,542.
D The net present value is a positive $30,000.

Since the payments are made at the end of each


year, this is an ordinary annuity and the present
value of the future cash flows is $66,242 ($20,000
times 3.3121). Net present value is the difference
in the cost of an investment ($50,000) and the
present value of the future cash flows ($66,242) or
$16,242. If the present value is larger, this net
present value is said to be positive and the
investment meets the investment criterion of the
company.

Business Environment & Concepts > Capital Budgeting/Financial Modeling > Investment Evaluation Methods

Question 13
A company is studying a possible investment opportunity. The
company can spend $22,000 and then get $5,000 in cash at
the end of each of the next 8 years. The company has a target
rate on such investments of 6 percent per year. The present
value of an annuity due of $1 per year at 6 percent annual
interest for 8 years is 6.58. The present value of an ordinary
annuity of $1 per year at 6 percent annual interest for 8 years is
6.21. Which of the following is true?
A Company should buy the investment because it
has a net present value of $9,050.
B Company should buy the investment because it
has a net present value of $10,900.
C Company should buy the investment because the
present value of 6.58 is larger than 6.0.
D Company should buy the investment because the
present value of 6.21 is larger than 6.0.

Answer 13
Correct answer is A
Net present value is one technique used by some
companies to evaluate the wisdom of an
investment. Because payments are made at the
end of each period, this is an ordinary annuity. The
present value of the cash flows is $5,000 times
6.21 or $31,050. Because the cost is $22,000, the
extra amount ($9,050 or $31,050 less $22,000) is
called the net present value. Investments with a
positive net present value are viewed as good
choices.

Business Environment & Concepts > Capital Budgeting/Financial Modeling > Cost of Capital

Cost of Capital
Question 1

Answer 1

Given the following information for ABC corporation: Balance


Sheet Value/After Tax Cost: Long Term Debt-50,000,000/6%;
Preferred Stock 20,000,000/9%; Common Equity
130,000,000/15%. Calculate the firm's cost of capital (WACC)

Correct answer is B

A 10%
B 12.15%
C 15%
D 6%

WACC is equal to the cost of each component of


capital times its proportional weight: WT/Cost/WT
Component: Debt 50/200/=.25x6%/1.5; PS
20/200=10x9%/.9; Common
130/200=.65x15%/9.75=12.15% (1.5 plus .9 plus
9.75 equals the 12.15%)

Question 2

Answer 2

When calculating a firms weighted average cost of capital, it


is necessary to determine the weight or percentage that each
type of capital comprises within the firms capital structure.
Which of the following is NOT a method used to calculate
weights.

Correct answer is D

A Book value
B Market value
C Optimal (target) values
D Replacement value

The four weighting systems that can be used are:


Book Values from the balance sheet; Market value;
Optimal or target capital structure weights;
Marginal weights would assign capital weights in
percentages that funds were actually raised.
Marginal weighing would weigh capital
components if the percentage funds were actually
raised. For example, if a project was funded using
only debt; debt weight would be 100$.

Business Environment & Concepts > Capital Budgeting/Financial Modeling > Calculating Cash Flows

Calculating Cash Flows


Question 1

Answer 1

KLM, Inc. is contemplating investing $1,000,000 to purchase a


new piece of machinery. The machine has an expected life of
5 years and will be depreciated on a straight line basis to a
zero salvage value (ignore any half year conversions). The
machine is expected to increase revenues by $300,000 per
year. The firms tax rate is 35%. What after-tax cash flow will
the machine generate in year one.

Correct answer is C
Cash flow is equal to: the increase in revenue
($300,000) minus depreciation of the asset
($200,000) or $100,000 change in income. The
$100,000 is taxed at 35% resulting in a $65,000
increase in net income. NI plus depreciation
($65,000 + $200,000) is equal to cash flow.

A $300,000
B $195,000
C $265,000
D $700,000

Question 2

Answer 2

Which of the following should not be considered when


estimating the cash flow from a potential investment?

Correct answer is D

A Opportunity cost
B Cannibalization (erosion in cash flow to existing
projects)
C Impact of the project on net working capital
requirements
D Sunk costs

A sunk cost has already been spent and cannot be


recovered. Therefore, it is not considered in the
investment decision process.

Question 3

Answer 3

Which of the following items would decrease the cash flow


attributable to a potential investment project?

Correct answer is C

A Higher levels of depreciation


B A lower rate of interest on borrowed funds.
C A higher tax rate
D A higher salvage value for the project

A higher tax rate will cause more money to be paid


out to the government as a result of the project.
Salvage value is a positive cash flow received at
the end of the project. Depreciation does not cause
cash flows directly but does decrease taxable
income and, thus, the amount paid for income
taxes. Reduction in interest rate causes less to be
paid out for interest so that more cash is retained.

Business Environment & Concepts > Capital Budgeting/Financial Modeling > Economic Value Added

Economic Value Added


Question 1

Answer 1

Net operating profit after taxes (NOPAT) or NOPLAT) minus


the firms dollar cost of capital (capital x WACC) is the
definition of

Correct answer is C

A Net present value (NPV)


B Internal rate of return (IRR)
C Economic valued added (EVA)
D Market value added (MVA)

NPA = present value of cash inflows minus the net


investment. IRR is the discount rate that makes the
present value of the inflows equal to the net
investment and forces the NPV to be equal to zero.
MVA= market value of the firm minus the book
value of the capital investment in the firm.

Question 2

Answer 2

Given the following information: NOPAT $10,000,000; EPS $2;


Capital invested $50,000,000; WACC 12%. Calculate the firm's
economic value added (EVA)

Correct answer is A

A $4,000,000
B $6,000,000
C $1,200,000
D 40%

EVA is expressed in dollars (eliminates D). EVA =


NOPAT - (capital x WACC) 10m - (50m x.12) 10m
- 6 m = $4,000,000

Business Environment & Concepts > Capital Budgeting/Financial Modeling > Return on Invested Capital (ROIC)

Return on Invested Capital (ROIC)


Question 1

Answer 1

Net income plus interest expense divided by the firms


average interest bearing debt plus shareholders equity is the
definition of which ratio?

Correct answer is C

A Return on assets (ROA)


B Return on equity (ROE)
C Return on invested capital (ROIC)
D Net profit margin

ROA = net income divided by total assets ROE =


net income divided by total equity Net profit margin
= net income divided by net sales ROIC = net
income plus interest divided by average total
invested capital. Invested capital is equal to
interest bearing debt plus shareholders equity.

Question 2

Answer 2

A firm has the following financial information: Net Income


$100,000; Interest expense $20,000; Average total assets
$1,000,000; Average total equity $550,000; Average total
invested capital $600,000. Based on this information, calculate
the firm's return on invested capital (ROIC).

Correct answer is A

A 20%
B 16.66%
C 10%
D 21.82%

ROIC (Return on Invested Capital) is the net


income of a company plus interest expense which
is then divided by the average total invested
capital. Here, that is: 100,000+20,000/600,000 or
20 percent.

Business Environment & Concepts > Long-Term Financing > Fixed Income Securities

Fixed Income Securities


Question 1

Answer 1

Which term refers to an unsecured bond.

Correct answer is D

A Mortgage
B Subordinated
C Senior
D Debenture

Mortgage means collateralized. Subordinated


means junior in standing. Senior means the
bondholders are in the front of the line in terms of
repayment.

Question 2

Answer 2

A provision in a bond contract that requires the issuer to retire


a set percentage of the bond issue each year is referred to as a

Correct answer is B

A Bond indenture
B Sinking fund
C Call provision
D Warrant

Sinking funds are designed to reduce the risk to


bondholders by gradually reducing the amount of
debt the firm has outstanding. In order to enforce
the sinking fund provision, the bonds are often
used with a call provision.

Question 3

Answer 3

The legal document that outlines the obligations of the bond


issuer is called a (an)

Correct answer is A

A Indenture
B Debenture
C Warrants
D Covenant

A debenture is an unsecured bond. Warrants are


long term options attached to bonds. Covenants
are provisions within an indenture detailing thins
the issuer must do (minimum ratios) or cannot do
(issue additional debt without permission).

Question 4

Answer 4

A bond that is rated below BBB is referred to as a (an)

Correct answer is B

A Investment grade bond


B Junk bond
C Debenture
D Subordinated

Bonds rated above BBB are investment grade.


Debenture means the bond is unsecured.
Subordinated means it is junior to other debt
issues.

Business Environment & Concepts > Long-Term Financing > Investment Banking

Investment Banking
Question 1

Answer 1

When an investment bank purchases the entire security


offering from the issuer and then attempts to resell it to the
public at a profit it is called a.

Correct answer is D

A Best efforts basis


B Private placement
C Rights offering
D Underwriting

Best efforts means the investment bank sells what


it can, in exchange for a commission. Rights
offerings involve offering the securities to existing
owners first. Private placement is selling the
securities to institutional investors rather than the
public.

Business Environment & Concepts > Long-Term Financing > Leases

Leases
Question 1

Answer 1

A lease must be classified as a capital lease if any one of four


conditions occurs. Which of the following is not one of those
conditions.

Correct answer is C
The correct answer is greater than or equal to 75%
of the assets life.

A Transfer of ownership occurs


B A bargain purchase plan option exist
C The lease period is greater than or equal to 50%
of the assets life
D The present value of the lease payments is
greater than or equal to 90% of the assets initial value

Question 2

Answer 2

A financial lease where the lessee sells an asset to the lessor


and subsequently leases it back is referred to as a (an)

Correct answer is D

A Synthetic lease
B Operating lease
C Capital lease
D Sale and lease back

B and C are accounting classifications for leases


determining whether or not assets and liabilities
show up on a firms financial statements.

Business Environment & Concepts > Long-Term Financing > Capital Structure Decisions

Capital Structure Decisions


Question 1

Answer 1

The extent to which fixed income securities (debt and/or


preferred stock) are used in a firms capital structure is the
definition of

Correct answer is C

A Operating leverage
B Operating break-even point
C Financial leverage
D Financial break-even point

Financial leverage and break-even refer to the


income statement from EBIT down to the last line
on the statement. Operating refers to items above
EBIT on the income statement.

Question 2

Answer 2

The least expensive source of long term capital to a firm is


generally

Correct answer is A

A Long term debt


B Preferred stock
C Retained earnings
D New common stock

Long term debt is generally cheapest for two


reasons: 1) the interest rate on the debt is tax
deductible and 2) debt is repaid first so it has less
risk.

Question 3

Answer 3

When a company needs to raise additional capital they often


seek the services of an investment banker. Which of the
following is not a method used by investment banks to help
firms raise needed capital?

Correct answer is D

A Best efforts selling syndicate


B Underwriting
C Private placement
D Clientele method

Best effort means they will sell what they can for a
commission but they take no ownership risk.
Underwrite means they guarantee a price to the
issuer and try to sell it at a higher price to earn a
profit. Private placement means they sell it to a
group of institutional investors.

Question 4

Answer 4

The Snape Corporation plans to raise $1 million for operating


purposes by either issuing a number of bonds to the public or
by issuing shares of common stock. Which of the following is
not an advantage of issuing the bonds?

Correct answer is B

A It is possible for owners to have an increase in


profits without making any additional investment.
B There is less risk associated with the bonds.
C The cost of bonds is lowered because the
resulting interest is deductible for tax purposes.
D If inflation results between the date of issuance
and the maturity date, the money paid back is worth less
than the money originally received.

All debt raises the risk of company bankruptcy. If a


cash flow problem arises and debts cannot be paid
as they come due, bankruptcy can quickly become
a possibility. However, debts do have advantages.
If the company can use the money to increase
income by more than the cost of the interest, this
leveraging increases net income without any
additional funds from the owners. Furthermore,
interest expense is tax deductible so that its cost is
reduced significantly (whereas dividend payouts
are not tax deductible). Finally, if inflation takes
place (which is common), the dollars that are
eventually used to repay the bonds are worth less
than the ones that were received on the day of
issuancea financial benefit

Business Environment & Concepts > Long-Term Financing > Dividend Policy

Dividend Policy
Question 1

Answer 1

The theory that a firms value is maximized by setting a high


dividend project ratio is the

Correct answer is B

A Dividend irrelevance theory


B Bird in the hand theory
C Tax preference theory
D Information signaling theory

Since investors are less certain of receiving future


capital gains, investors value a dollar of current
dividend more than a dollar of uncertain capital
gains and this will result in a high firm value.

Question 2

Answer 2

How often are dividends paid by U.S. corporations?

Correct answer is C

A Annually
B Monthly
C Quarterly
D Semi-annually

Dividends are paid quarterly

Question 3

Answer 3

Which of the following is the correct chronological calendar for


the four dates in the dividend payment process?

Correct answer is B

A Declaration date, date of record, ex-dividend rate,


and payment date
B Declaration date, ex-dividend date, date of record,
and payment date
C Ex-dividend date, declaration date, date of record,
and payment date
D Date of record, declaration date, ex-dividend date,
and payment date

Business Environment & Concepts > Long-Term Financing > Fixed Income Securities

Fixed Income Securities

Business Environment & Concepts > Long-Term Financing > Hybrid Securities

Hybrid Securities
Question 1

Answer 1

Which of the following long-term sources of funds is


considered a hybrid security.

Correct answer is B

A Retained earnings
B Preferred stock
C Common stock
D Bond

Preferred stock has features of bonds (a fixed


dividend) and equity (no maturity).

Business Environment & Concepts > Long-Term Financing > Common Stocks

Common Stocks
Question 1

Answer 1

Common stock owners often have the right to purchase a pro


rata share of new issues of common stock to maintain their
ownership percentage in the firm. This right is referred to as a
(an)

Correct answer is B

A Proxy right
B Preemptive right
C Founders shares
D Put option

A proxy authorizes someone else to vote for you.


Founders shares are issued to the original owners
of the firm. A put option allows you to sell at a fixed
rate for a fixed period of time.

Business Environment & Concepts > IT > IT Fundamentals

IT Fundamentals
Question 1

Answer 1

The best definition of computer hardware is

Correct answer is A

A Physical computer equipment


B Computer programs and applications
C Machine language
D Virtual Storage

Computer hardware is the physical computer


equipment. Computer programs and applications
are examples of software, machine language is an
example of a software program and virtual storage
is a technique utilized by the computer's operating
system.

Question 2

Answer 2

Which of the following statements relating to hardware


maintenance is true?

Correct answer is B

A Downtime is unscheduled, unanticipated and


costly
B Downtime is time when the computer is not
functioning; this may be scheduled or unscheduled.
C Routine service cannot be scheduled because
most systems are operational 24/7.
D Hardware maintenance is generally unscheduled
as most downtime is associated with unanticipated
problems

Hardware maintenance involves equipment


service. Routine service is scheduled.
Unscheduled maintenance arises when there are
unanticipated problems. Downtime is time when
the computer is not functioning; this may be
scheduled or unscheduled.

Question 3

Answer 3

CPU is an abbreviation for

Correct answer is D

A Critical Primary Unit


B Control Processing Utility
C Coded Primary Utility
D Central Processing Unit

The CPU or Central Processing Unit is a primary


hardware component where the actual processing
of data occurs

Question 4

Answer 4

The CPU includes:

Correct answer is A

A Primary storage, a control unit and an


arithmetic/logic unit.
B Primary storage and a control unit only
C A control unit and an arithmetic/logic unit only
D Primary storage and an arithmetic/logic unit only

The CPU contains primary storage, a control unit


and an arithmetic/logic unit.

Business Environment & Concepts > IT > IT Fundamentals

Question 5

Answer 5

The best definition of the Primary Storage of the CPU is

Correct answer is C

A The permanent main memory portion of the CPU


that holds the program, data and results during
processing
B The temporary main memory portion of the CPU
that holds interactive and batch results during
processing and reporting.
C The temporary main memory portion of the CPU
that holds the program, data and results during
processing
D The permanent main memory portion of the CPU
that holds the program, data and security logs during
and after processing

The Primary Storage is the main memory portion of


the CPU that holds the program, data and results
during processing. This storage is temporary
consisting of both RAM (Random Access Memory)
and ROM (Read Only Memory)

Question 6

Answer 6

RAM and ROM are abbreviations for

Correct answer is C

A RAM (Read Available Memory) and ROM (Right


Office Materials)
B RAM (Relational Access Master) and ROM (Read
Operational Memory)
C RAM (Random Access Memory) and ROM( Read
Only Memory)
D RAM (Random Available Memory) and ROM
(Related Operational Master)

The Primary Storage is the main memory portion of


the CPU that holds the program, data and results
during processing. This storage is temporary
consisting of both RAM (Random Access Memory)
and ROM (Read Only Memory)

Question 7

Answer 7

The function of the Control Unit of the CPU is to

Correct answer is B

A Control the user interface and store the program,


data and results as directed by that user interface on a
temporary basis
B Control and directs the operation of the computer
by reading or interpreting the program instructions and
directing the computer in the execution of those
instructions.
C Control and directs the operation of the computer
by writing program instructions based on business
requirements
D Control and directs the operation of the computer
by storing program instructions and security violations

The Control Unit of the CPU controls and directs


the operation of the computer by reading or
interpreting the program instructions and directing
the computer in the execution of those instructions.

Business Environment & Concepts > IT > IT Fundamentals

Question 8

Answer 8

The function of the Arithmetic/Logic Unit of the CPU is to

Correct answer is A

A Is the circuirty that performs the arithmetic


calculations and logical operations.
B Is add-in software that performs the arithmetic
calculations and logical operations.
C Is managed by IT Security to track the arithmetic
calculations and logical operations.
D Is a component of the primary storage that
calulates results during processing in RAM

The Arithmetic/Logic Unit of the CPU is the


circuitry that performs the arithmetic calculations
and logical operations. This unit may be combined
with the Control Unit.

Question 9

Answer 9

The function of the Bus is to

Correct answer is A

A Transfer data from the CPU to primary memory


and peripheral devices
B Transfer data from the software to database
applications
C Transfer instructions from primary memory to user
interface peripheral devices
D Transfer program instructions and directhe
computer in the execution of those instructions

The Bus is the circuitry connecting the CPU to the


primary memory and to peripheral devices.

Question 10

Answer 10

The function of input/output devices is to

Correct answer is D

A Control and directs the operation of the computer


by writing internal program instruction
B Carry electrical current through the CPU and
mother board
C Direct and control the data flow beteween the Bus
and the Arithmetic/Logic Unit
D Transfer data in and out of the CPU

Input and output devices transfer data in and out of


the CPU. Some examples include a keyboard,
monitor, scanner, printer, mouse, modem, joystick,
touchpad etc.

Question 11

Answer 11

Examples of input/output devices are

Correct answer is B

A UBC, USB, USA devices


B Keyboard, monitor, scanner, printer, mouse,
modem, joystick, touchpad etc.
C Circuitry diurecting and controlling secondary
storage devices
D Paper, reports, telephone, fax, email, etc

Input and output devices transfer data in and out of


the CPU. Examples include but are not limited to a
keyboard, monitor, scanner, printer, mouse,
modem, joystick, touchpad etc.

Business Environment & Concepts > IT > IT Fundamentals

Question 12

Answer 12

A Bar Code reader and modem are both examples of which of


the following devices

Correct answer is C

A Bus Circuitry
B RAM and ROM
C Input/Output devices
D Primary and Secondary Storage

Input and output devices that transfer data in and


out of the CPU. Other examples include but are
not limited to a keyboard, monitor, scanner, printer,
mouse, magnetic tape reader, joystick, touchpad
etc.

Question 13

Answer 13

The definition that best fits peripheral equipments is

Correct answer is D

A RAM and ROM


B Arithmetic/Logic Unit
C Primary and Secondary Storage
D Input/Output devices

Peripheral equipment is any device that is not a


part of the CPU but can be accessed by the CPU.
Examples include secondary storage and any input
or output device. Primary storage and the
Arithmetic/Logic Unit are part of the CPU. RAM
and ROM are forms of primary storage.

Question 14

Answer 14

Which of the following does not describe primary and


secondary storage

Correct answer is C

A Primary storage is internal to the CPU and


Secondary storage is external to the CPU.
B Primary storage is temporary while Secondary
storage is more permanent.
C RAM is a componet of primary storage and ROM
is a component of secondary storage
D Primary and Secondary stoarge both hold
progams, data and results

The Primary Storage is the temporary main


memory portion of the CPU consisting of both RAM
(Random Access Memory) and ROM (Read Only
Memory). Secondary storage consists of devices
external to the CPU consisting of disks, flash
drives, hard drives etc.

Question 15

Answer 15

The difference between hardware and software is

Correct answer is A

A Hardware is the physical computer equipment and


software is the programs that operate the system and
process data.
B Hardware is comprised of purchased components
whereas software is comprised of the systems that are
downloaded
C Programs reside and run on the hardware
whereas data and results are stored in the software
D Disaster Recovery & Business Continuity plans
are concerned with hardware but do not include
software

Hardware includes the physical tangible equipment


whereas software includes programs, the
operating system and the intangible instructions
and processes that allow the system to function.
Disaster Recovery & Business Continuity plans are
concerned with both hardware and software.

Business Environment & Concepts > IT > IT Fundamentals

Question 16

Answer 16

Which of the following statements relating to Machine


Language is not true.

Correct answer is C

A Machine language is a binary (one/off) language


that is interpreted by the computer hardware.
B A compiler is required to to translate higher level
languages into Machine Language.
C Machine Language is the most efficient form of a
fourth generation language.
D Writing programs in machine language is difficult
and prone to error.

Machine language is a binary (one/off) language


that is interpreted by the computer hardware. It is
the lowest level or first generation language and is
machine dependent. Writing programs in machine
language is difficult and prone to error. A compiler
is required to translate higher level languages into
machine language.

Question 17

Answer 17

Which of the following statements relating to Assembly


Language is not true.

Correct answer is C

A Assembly language must be translated into


Machine language by an Assembler
B It is easier to write programs in Assembly
language than it is to write programs in Machine
Language.
C Assembly Language can be run on any type of
hardware.
D Assembly Language is an efficient form of a
second generation language

Assembly language is a second generation


language that requires an assembler to translate
assembly language into machine language. It is a
machine dependent language utilizing short
commands for repetitive tasks but it is easier to
write programs in Assembly language than
Machine language.

Question 18

Answer 18

Which of the following statements relating to Procedural


Language is not true.

Correct answer is D

A Procedural Languages are a third generation


language that allows programmers to concentrate on the
procedures and functions of the programs.
B FORTRAN, COBOL and BASIC all forms of
procedural Languages.
C Procedural language programs are written in
source code which is then converted or translated into
object code, the machine language for a particular type
of computer.
D Source code is very similar to machine language
so the conversion process is quick and efficient

Procedural Languages are a third generation


language that allows programmers to concentrate
on the procedures and functions of the programs.
The programmer writes the program in source
code which is then converted or translated into
object code. Source code is more similar to English
while the object code is the machine language for
a particular type of computer. FORTRAN, COBOL
and BASIC all forms of procedural Languages.

Business Environment & Concepts > IT > IT Fundamentals

Question 19

Answer 19

Which of the following statements relating to COBOL FORTAN


and BASIC is true?

Correct answer is A

A FORTRAN was designed for scientific purposes,


COBOL was designed for Business and BASIC was
designed for educational purposes.
B All three are examples of a fourth generation
language that does not need to be converted to machine
code.
C All three are examples of object code that requires
a specifice hardware configuration.
D FORTRAN was designed for educational
purposes, COBOL was designed for Business and
BASIC was designed for end user purposes

FORTRAN (FORMula TRANslation) was designed


for scientific purposes. COBOL (Common
Business Oriented Language) was designed for
Business and BASIC (Beginner All purpose
Symbolic Instruction Code) was designed for
educational purposes. All three are examples of a
third generation language that must be converted
from source code to object or machine code.

Question 20

Answer 20

What is the main advantage to programmers utilizing a fourth


generation language(4GL)?

Correct answer is B

A 4GLs are delivered with pre-loaded databases


B 4GLs are relatively easy for programmers to use
C 4GLs are close to the level of machine language
which provides for flexibility
D 4GLs are an efficient use of computer resources

Fourth generation languages (4GL) have many


routine commands and procedure
pre-programmed. 4GLs are often associated with a
DataBase Management System and are relatively
easy for programmers to use. They are in fact
designed to reduce programming effort and
improve the process of software development.
This efficiency can come at the cost of computer
resources.

Question 21

Answer 21

What is the main advantage of a GUI Interface?

Correct answer is D

A It provides a creative outlet for system designers


B It is easier to write programs in GUI than it is to
write programs in Machine Language.
C It utilizes a series of algorithms to increase
processing efficiency.
D It a can eliminate the need for a user to learn a
complex set of commands

GUI or Graphical User Interface allows the user to


navigate the system and access programs though
a series of graphical icons, visual indicators, scroll
bars, pictorial and graphical symbols. A well
designed GUI interface can eliminate the need for
a user to learn a complex set of commands.

Question 22

Answer 22

Which of the following statements regarding a patch to a


program is true?

Correct answer is B

A A patch is a segment of the original source code


B A patch is a change or modification to an existing
program which may or may not be authorized.
C A patch to a program should be prohibited as it is
related to unauthorized or criminal behavior
D Patches are written in machine language or object
code

A patch is a change or modification to an existing


program. It may be to correct an error in
programming or as a result of a change in
requirements. For example, changes in tax laws
may require the implementation of a patch. A patch
may also be added for fraudulent purposes. For
example, to reallocate funds into a personal
account.

Business Environment & Concepts > IT > IT Fundamentals

Question 23

Answer 23

Which of the following statements best describes the function


of the operating system (OS)?

Correct answer is A

A A traffic controller managing the transfer of data


between programs and peripheral devices.
B A system log of all patches to existing programs.
C A complete listing and definition of the fields in the
master database.
D The master list of all repairs and changes to the
hardware and software

The Operating System (O/S) manages and


schedules application programs and system
functions. The OS tracks, coordinates and
allocates memory, inputs, outputs and performs
security functions. Several forms of OS are
available, Windows, Unix and Linus, MVS and DOS
are a few examples.

Question 24

Answer 24

Which of the following statements best describes the function


of Job Control Language or JCL?

Correct answer is A

A A language that prioritizes and controls when


application programs are initiated.
B A Human Resources initiative to manage the
creation and description of new positions.
C A language the converts source code to object
code or machine language as required by the system
where it is installed.
D An alternative to an application program when a
micro processing system is available

Job Control Language or JCL is a command


language that initiates programs, specifies
processing priorities, running sequences,
databases used and files used.

Question 25

Answer 25

The advantage of Virtual Storage is

Correct answer is D

A It is unlimited
B It does not use system resources
C It is located offsite
D It saves time and money

The operating system divides a program into pages


or segments and brings only the pages of the
program required for execution into memory. This
saves time and money as the unneeded portions of
the program remain in less expensive secondary
memory.

Question 26

Answer 26

Which of the following statements relating to an Applications


Program is true?

Correct answer is B

A An Applications Program is designed to manage


system resources.
B An Applications Program is designed to perform a
specific process or series of tasks.
C An Applications Program is designed to manage
the flow of applications and comply with the Fair Labor
Standards Act of 1986.
D An Applications Program is focused on the needs
of the Human Resources department

An Applications Program is designed to perform a


specific process or series of tasks. For example,
an accounts payable program is designed to match
an invoice to a purchase order and perform the
tasks required to process the invoice for payment.

Business Environment & Concepts > IT > IT Fundamentals

Question 27

Answer 27

Which of the following is the best explanation of an enterprise


resource planning system or ERP?

Correct answer is B

A It is a computer system designed to make certain


that all raw materials are received as needed within an
organization.
B It is an attempt to consolidate all of a company's
departments and functions into a single computer
system that services each department's individual needs
through the use of a common database.
C It is a computer program that should maintain
proper cash resources within a company at all times and
places.
D It is an ongoing planning process that measures
labor input and related output in an attempt to maximize
efficiency.

According to Wikipedia: Enterprise resource


planning (ERP) is an enterprise-wide information
system designed to coordinate all the resources,
information, and activities needed to complete
business processes such as order fulfillment or
billing. An ERP system supports most of the
business system that maintains - in a single
database - the data needed for a variety of
business functions such as manufacturing , supply
chain management, financials, projects, human
resources and customer relationship management

Question 28

Answer 28

In information technology, what is a LAN?

Correct answer is D

A A computer network across more than one


continent.
B A computer network between groups of a similar
type such as customers
C A computer network that contains data subject to
random access.
D A computer network within a single building or
relatively small geographic area.

A local area network (LAN) is a privately owned


network that allows people within a small
geographical region (possibly a single building) so
that the employees of a company can perform
specified operating activities.

Business Environment & Concepts > IT > Disaster Recovery and Business Continuity

Disaster Recovery and Business Continuity


Question 1

Answer 1

Which of the following definitions best describes a Hot Site?

Correct answer is D

A The organization's site


B An agreement with another comparable
organization to aid each other in the event of a disaster.
C An alternate site where the organization provides
and installs hardware.
D An alternate site with a commercial disaster
recovery services that allows the organization to
continue business operations.

A Hot Site is a commercial disaster recovery


service that allows a business to continue
computer operations in the event of a computer
disaster. For example, if a companys data
processing center becomes inoperable, that
enterprise can move all processing to a hot site
that has all of the equipment needed to continue
operation. This is also refereed to as a recovery
operations center (ROC) approach. Answer A is an
Internal Site, answer B is a Reciprocal Agreement
and answer C is a Cold Site.

Question 2

Answer 2

A Disaster Recovery & Business Continuation Plan should


allow the firm to perform all of the following actions except for

Correct answer is B

A Minimize the extent of disruption, damage and


loss.
B Relocate to another location.
C Resume normal operations as quickly as possible.
D Train and familiarize personnel to perform
emergency operations.

Relocation to another location may not be


necessary. A Disaster Recovery & Business
Continuation Plan should allow the firm to (A)
Minimize the extent of disruption, damage and
loss, (C) Resume normal operations as quickly as
possible, (D) train and familiarize personnel to
perform emergency operations as well as establish
an alternate (temporary) method for processing
information.

Question 3

Answer 3

Backup approaches in a Disaster Recovery & Business


Continuation Plan include all of the following except

Correct answer is A

A ABC Reclamation Methods


B Checkpoint copies of the database
C Grandfather-Father-Son batch systems
D Rollback Recovery

ABC Reclamation methods are fictional. Answers B


through D all valid backup approaches. Checkpoint
copies of the data base occur when at certain
points, copies are made of the database and
stored in a secure location,
Grandfather-Father-Son batch system are backup
files that are created based on the process of
updating a master file with the day's transactions
which creates a new generation. Rollback
Recovery is a process that removes the effects of
updates until it reaches a point where the system
was processing accurately.

Business Environment & Concepts > IT > Disaster Recovery and Business Continuity

Question 4

Answer 4

Which of the following definitions best describes a Cold Site?

Correct answer is C

A The organization's site


B An agreement with another comparable
organization to assist each other in the event of a
disaster.
C An alternate site where the organization provides
and installs hardware.
D An alternate site with a commercial disaster
recovery services that allows the organization to
continue business operations.

A Cold Site is similar to a Hot Site but the


organization provides and installs the equipment. It
is less expensive than a Hot Site but takes longer
to recover after a disaster. Answer A is an Internal
Site, answer B is a Reciprocal Agreement and
answer D is a Hot Site.

Question 5

Answer 5

Which of the following definitions best describes a Reciprocal


Agreement?

Correct answer is B

A The organization's site


B An agreement with another comparable
organization to aid each other in the event of a disaster.
C An alternate site where the organization provides
and installs hardware.
D An alternate site with a commercial disaster
recovery services that allows the organization to
continue business operations.

A Reciproccal Agreement, also called amutual aid


pact, is between at least two organizations with
comparable computing requirements agreeing to
assist each other in the event of a disaster. Answer
A is an Internal Site, answer C is a Cold Site and
answer D is a Hot Site

Question 6

Answer 6

Which of the following definitions best describes a, Internal


Site?

Correct answer is A

A The organization's site


B An agreement with another comparable
organization to aid each other in the event of a disaster.
C An alternate site where the organization provides
and installs hardware.
D An alternate site with a commercial disaster
recovery services that allows the organization to
continue business operations.

An internal site is another data processing center


within the organization. Generally this option is
only available for fairly karge organizations.
Answer B is a Reciprocal Agreement, answer C is
a Cold Site aqnd answer D is a Hot Site.

Business Environment & Concepts > IT > Disaster Recovery and Business Continuity

Question 7

Answer 7

Which of the following lists of requirements pertain to a


Disaster Recovery & Business Continuity Plan?

Correct answer is A

A Priorities, insurance, backup approach, specific


assignments, period testing and updating and
documentation.
B Press release contacts, evacuation routes,
specific assignments, period testing and updating and
documentation.
C Priorities, alternate investment opportunities,
insurance, Hot Site directions, Reciprocal agreements,
specific assignments, period testing and documentation.
D Price lists, confidential information storage plans,
priorities,backup approach, specific assignments and
documentation.

A Disaster Recovery & Business Continuity Plan


should include priorities, insurance, backup
approach, specific assignments, period testing and
updating and documentation.

Question 8

Answer 8

Prioritization in a Disaster Recovery & Business Continuity


Plan refres to

Correct answer is D

A Which backup appraoch is the most critical


B Which employees are the most critical
C Which emergencies are the most critical
D Which applications are the most critical

The prioritization in a Disaster Recovery &


Business Continuity Plan refres to which
applications are the most critical

Question 9

Answer 9

A Disaster Recovery & Business Continuity Plan should strive


for the following goals

Correct answer is B

A A plan should minimize the effects of streess on


critical personnel, facilitate the return to profitable
operations and train employees to perform under
emergency operations.
B A plan should minimize the extent of disruption,
damage and loss, establish an alternate method of
processing data, facilitate the return to normal
operations and train employees to perform under
emergency operations.
C A plan should minimize the extent of change and
insurance claims, establish an alternateflow of revenue,
facilitate the return to normal operations as soon as
possible and train employees to perform under
emergency operations.
D A plan should minimize the extent of disruption,
damage and loss, relocate to an alternate permanent
facility, facilitate the return to normal operations, and
train employees to perform as emergency personnel.

A plan should minimize the extent of disruption,


damage and loss, establish an alternate method of
processing data, facilitate the return to normal
operations as soon as possible and train
employees to perform under emergency
operations.

Business Environment & Concepts > IT > Disaster Recovery and Business Continuity

Question 10

Answer 10

A Disaster Recovery & Business Continuity Plan should include

Correct answer is C

A Specific assigments for executives but general


assignements for those in middle management and
below
B General assignments for personnel as the
situation may call for flexibility.
C Specific assignments for personnel who are
familiar with the plan and their responsibilities
D Surprise assignments for personnel because
everybody loves surprises

Specific assignments for personnel who are


familiar with the plan and their responsibilities may
include arranging for backup facilities, computer
operations and software, obtaining vital documents,
establishing communications and arranging for
supplies.

Question 11

Answer 11

The best backup facility option in a Disaster Recovery &


Business Continuity Plan is

Correct answer is C

A A Hot Site
B A Reciprocal Agreement
C A combination of the Hot Site, Cold Site,
Reciprocal Agreement and Internal Site
D The Checkpoint approach used in conjunction with
the Rollback Approach

A combination of approaches may be the best


option in the event of a catastrophic event and
most options will rely on a complete backup of the
entire system to become fully operational.

Business Environment & Concepts > IT > Business Information Systems

Business Information Systems


Question 1

Answer 1

Information systems process data and transactions within the


business organization. These actions include all but the
following.

Correct answer is C

A Collecting and entering transactions and data


B Processing data and providing users with the
information that they need to run the business
C validation and comparison of system generated
reports to input data
D Controlling the process and data

Options A, B and D are aspects of an information


system in the business. Option C represents a
manual action performed by an end user to validate
output of the system.

Question 2

Answer 2

Which of the following is an advantage of computer systems


over manual processing?

Correct answer is A

A Elimination of computational errors and errors


processing routine transactions
B Significant cost reductions to the organization in
both terms of time and money
C Faster processing times resulting in shorter
accounting cycles
D Elimination of antiquated processes and
procedures

An advantage of computer systems over manual


processing is the elimination of computational
errors and errors processing routine transactions

Question 3

Answer 3

Two general types of computer processing systems are

Correct answer is B

A Mainframe and microcomputer


B Transaction processing and management
reporting
C Super computers and minicomputers
D PDAs and cell phones

Transaction processing systems generally involve


a high volume of simple transactions for example
invoice processing, reservation systems, billing,
shipping etc.) Management reporting systems are
designed to provide management with information
they need to make informed decisions. An
executive information system that provides
information of a strategic nature is an example.
Mainframe, microcomputer,
supercomputersminicomputers and PDAs are all
examples of types of computers but rather than the
processing system.

Business Environment & Concepts > IT > Business Information Systems

Question 4

Answer 4

Examples of management reporting systems include all but the


following.

Correct answer is D

A Management Information Systems


B Expert systems
C Decision Support systems
D Accounts Payable system

An accounts payable system is a type to


transaction processing systems. Options A - C are
all types of management reporting systems.
Management Information Systems are designed to
provide informatrion for planning organizing and
controlling the operations of the business. Decision
Support Systems are designed to combine data
and model sot resolve problems. Expert systems
apply specific models to data to provide a specific
type of recommendation.

Question 5

Answer 5

What is the main difference between transaction processing


and management reporting systems

Correct answer is B

A There is no difference. They are basically the


same
B Transaction processing systems generally process
a large volume of transactions where management
reporting systems provide information used to support
business decisions.
C Transaction processing systems generally process
basis financial data where management reporting
system accumulate the results of the financial data.
D Transaction processing systems generally process
simple activities where management reporting systems
are more complex.

Transaction processing systems generally process


a large volume of transactions where management
reporting system provide information used to
support business decisions. Processing an invoice
for payment occurs in a transaction processing
system. Providing business results and forecasts
occurs in a management reporting system.

Question 6

Answer 6

What is the main difference between batch and on-line


processing?

Correct answer is B

A Batch processing is related to transaction


processing and on-line processing is used for
management reporting systems
B Batch processing accumulates and processes
similar transactions in batches while on-line processing
processes transactions as they occur
C Batch processing is relation to applications
programming while on-line processing is related to
systems programming
D There is no difference. They are basically the
same

Batch processing accumulates and processes


similar transactions in batches. An example is a
disbursements program that writes checks to all
vendors as an overnight process. On-line
processing processes transactions as they occur.
An example is a reservations program that books
an airline ticket.

Business Environment & Concepts > IT > Business Information Systems

Question 7

Answer 7

What is the main difference between on-line transaction


processing and on-line analytical processing?

Correct answer is B

A There is no difference. They are basically the


same
B On-line transaction processing supports day to
day processing while on-line analytical processing
allows for the day to day data to be analyzed
C On-line transaction processing operates in an
on-line environment while on-line analytical processing
is batch
D On-line transaction processing is a standard
processing method while on-line analytical processing is
cutting edge technology.

On-line transaction processing supports day to day


processing while on-line analytical processing
allows for the day to day data to be analyzed. For
example invoice processing may occur in an
on-line transaction process but vendor inquires
regarding payments may occur in an on-line
analytical environment.

Question 8

Answer 8

What is the main difference between on-line real time and


direct access processing?

Correct answer is B
They are basically the same.

A On-line real time processing processes data


without a database while direct access process requires
a database
B There is no difference. They are basically the
same
C On-line real time processing processes
transactional data while the direct access process is
related to management reporting
D On-line real time is an on-line process while direct
access processed in batch.

Question 9

Answer 9

The definition of a data warehouse is

Correct answer is C

A A collection of backup files


B A data and application storage facility
C An integrated collection of data used to support
management decisions
D A collection of interactive files and logistical
programs

An integrated collection of data used to support


management decisions. Data is periodically
downloaded from databases into a data
warehouse. A data mart is a limited version of a
data warehouse.

Question 10

Answer 10

The process of data mining includes

Correct answer is A

A The use of complicated statistical and graphical


processes and structured approaches to manipulate
data.
B A specific set of programming tools ironically
referred to as picks and shovels
C Manual manipulation of data
D A high level of programming experience

Data mining uses complicated statistical and


graphical processes and structured approaches to
manipulate data.

Business Environment & Concepts > IT > Business Information Systems

Question 11

Answer 11

Which of the following related to data mining is not true

Correct answer is D

A Data mining uses statistics and Artificial


Intelligence
B Data mining accesses data warehouses and data
marts
C Data mining is useful in analyzing situations and
potential problems
D Data mining is for the exclusive use of upper
management

Options A - C are related to data mining. Data


mining is useful and used by all levels of the
organization.

Question 12

Answer 12

Which of the following statements relating to Artificial


Intelligence (AI) is not true

Correct answer is D
Options A - C are related to AI while D is untrue

A AI is software designed to help humans make


decisions
B AI is an attempt to mimic the human thought
process
C AI deals with processes that involve a structured
or predicable approach
D AI contains serious ramifications and is closely
controlled

Question 13

Answer 13

Which of the following statements relating to Artificial


Intelligence (AI) is not true

Correct answer is C

A AI involves using a computer to reach the same


conclusion as a human
B An expert system is a form of AI
C AI is used primarily by the scientific community
D AI reaches a conclusion much faster than an
human

Options A, B and D are related to AI while C is


untrue. The business community for example
utilizes AI.

Question 14

Answer 14

Which of the following statements relating to an expert system


is not true

Correct answer is C

A An expert system utilizes expert knowledge


programmed into a systematic approach
B An expert system guides the decision making
process and provides decisions comparable to those of
an expert
C Expert systems are rarely implemented outside of
the medical and scientific communities.
D An expert system is a form of AI

Options A, B and D are related to an expert system


while C is untrue. Expert system are widely used in
many industries.

Business Environment & Concepts > IT > Business Information Systems

Question 15

Answer 15

Which of the following statements relating to centralized


processing is not true?

Correct answer is A

A Centralized processing has been replaced with


more efficient forms of processing.
B Processing occurs at one location
C This is the traditional model used with mainframe
computers
D Centralized processing is still in use

Option A is incorrect. Centralized processing


continues to be in use and is often combined with
decentralized or distributed processing.

Question 16

Answer 16

Which of the following statements relating to decentralized


processing is not true?

Correct answer is B

A Computers may be viewed as a collection of


independent databases rather than an integrated system
B Decentralized processing has been replaced with
more efficient forms of processing.
C Processing occurs at multiple locations
D Computers are not generally networked and do
not share data

Option B is incorrect. Decentralized processing


continues to be in use and is often combined with
centralized or distributed processing. End Users
utilizing PCs are essentially utilizing a form of
decentralized computing.

Question 17

Answer 17

Which of the following statements relating to distributed


processing is not true?

Correct answer is C

A Processing occurs at multiple locations


B Computers are networked and access a single
database
C Multiple databases are linked to form a single
master database
D A single database is updated utilizing both on-line
and batch transactions

Options C is incorrect. Distributed processing can


utilize a single database accessed by all users.

Question 18

Answer 18

A binary digit (0 or 1) which is the smallest storage unit in a


computer is the definition of a

Correct answer is A

A Bit
B Byte
C Character
D Field

A bit is a binary digit (0 or 1) which is the smallest


storage unit in a computer is the definition of a

Business Environment & Concepts > IT > Business Information Systems

Question 19

Answer 19

A group of related characters (e.g. employee name) is the


definition of a

Correct answer is D

A Bit
B Byte
C Character
D Field

A field is a group of related characters (e.g.


employee name, street address, personnel
number). A field is generally comprised of one or
more characters

Question 20

Answer 20

A letter, number or symbol is the definition of a

Correct answer is D

A Bit
B Byte
C Field
D Character

A character is a letter, number or symbol.


Character are generally comprised of one or more
bytes

Question 21

Answer 21

A group of eight digits (0 or 1) that is treated as a single unit is


the definition of

Correct answer is A

A Byte
B Bit
C Character
D Field

A byte is a group of eight digits (0 or 1) that is


treated as a single unit. A bit is a single binary digit,
either a 0 or 1. Bits are combined to create bytes.

Question 22

Answer 22

An example of a Master File is

Correct answer is A

A Vendor names and addresses


B Vendor invoices
C Checks issued
D Purchase order details

Question 23
A company has seven divisions. The Blue Division is very
slow at sending out invoices so that cash collections are
delayed. The management of the Blue Division looks at the
invoice systems operating within the other six divisions and
discovers that the Green Division has the best practices in this
area of the business. Therefore, where practical, they adopt
some of these practices for use by the Blue Division. What is
this process called?
A Maximization of Time
B Benchmarking
C Quantum Time Theory
D Haglers Theory of Routine Processes

A master file contains data that is relatively static.


Vendor names and addresses do not change that
often whereas invoices, check data and purchase
order details typically reside in a detail file. Detail
files may be associates with master files.

Answer 23
Correct answer is B
Company officials should always be looking for
better ways to do things so that their own
organization is able to become more efficient on a
continuous basis. Comparisons should be made
between the way that activities are executed by the
organization and the procedures carried out by
others, in other divisions and even in other
companies. The process of discovering
weaknesses and using the results obtained from
others to create improvement is known as
benchmarking.

Business Environment & Concepts > IT > IT Controls

IT Controls
Question 1

Answer 1

Which of the following is not a security consideration for small


computers?

Correct answer is A

A Bar-coding to track hardware locations


B Verification of Processing to ensure the system is
used for business related projects
C Security over data and in-house developed
software
D Appropriate authorization to purchase hardware
and software

Barcode tracking of the physical location of small


computers is not required. (B) Periodically, an
independent verification of the applications being
processed on the small computer system should
be made to prevent the system from being used fro
personal projects. (C) Security over data and
in-house developed software is important as most
companies can easily replace hardware but may
suffer a severe setback if the data and/or in-house
developed software is lost. (D) Purchases of
hardware and software should be reviewed for
compatibility, piracy and other issues.

Question 2

Answer 2

Establishing a corporate software policy, maintaining a log of


all software purchases and auditing individual computers to
identify installed software are all procedures to control what
small computer security issue?

Correct answer is D
All three procedures may control possible software
piracy. Then other answers are unrelated to IT
Security

A IT receiving report inconsistencies


B Asset management capitalization errors
C Employee theft and/or fraud
D Software piracy

Question 3

Answer 3

One of the risks not associate with End-User Computing


(EUC) is

Correct answer is A

A More time is spent by client personnel focusing on


the testing and documentation process
B Management often does not review the results of
applications appropriately
C More client personnel need to understand control
concepts
D End-User applications are not always adequately
tested before implemented

(A) Client personnel may not test accurately or


document the application. Answers B, C and D are
valid risks

Business Environment & Concepts > IT > IT Controls

Question 4

Answer 4

Which of the following statements relating to End-User


Computing (EUC) is true?

Correct answer is A

A Overall physical access controls become more


difficult when companies leave a controlled MIS
environment and become more dependent upon
individual users for controls.
B Overall physical access controls become more
easier when companies leave a controlled MIS
environment and become more dependent upon
individual users for controls.
C Overall physical access controls become more
visible when companies leave a controlled MIS
environment and become more dependent upon
individual users for controls.
D Overall physical access controls become tighter
when companies leave a controlled MIS environment
and become more dependent upon individual users for
controls.

(A) It is easier to control access in a controlled MIS


environment. It is more difficult to control access
outside of a controlled MIS environment where
control are less visible and dependent on individual
users.

Question 5

Answer 5

Physical access controls include all but the following

Correct answer is D

A Clamps or chains
B Regular backup
C Control over access from outside
D User reconciliations

(D) Clamps or chains to prevent removal of hard


disks or internal boards, regular backup and
control over access from outside are all forms of
physical access controls. Reconciliations are
typically an accounting process.

Question 6

Answer 6

Two methods to control access to appropriate users include

Correct answer is C

A Passwords and user IDs and allowing users to


load their own data
B Menus to access databases regular review of
personal transactions
C Independent review of transactions and restricting
user ability to load data
D Independent review of transactions and shared
passwords and user IDs

(C) Methods to control access to appropriate users


include passwords and user IDs, menus for EUC
access databases, independent review of
transactions, restricting user ability to load data,
requirement of appropriate validation, authorization
and reporting control when the end user uploads
data and record access to company databases by
the EUC application.

Question 7

Answer 7

Two methods to control access to appropriate users include

Correct answer is D

A Menus to access databases and ignorance of


transaction codes
B Restricting user ability to load data and hourly
backups
C Independent review of transactions and requiring
users to load their own software and data
D Passwords and user IDs and Independent review
of transactions

(D) Methods to control access to appropriate users


include passwords and user IDs, menus for EUC
access databases, independent review of
transactions, restricting user ability to load data,
requirement of appropriate validation, authorization
and reporting control when the end user uploads
data and record access to company databases by
the EUC application.

Business Environment & Concepts > IT > IT Controls

Question 8

Answer 8

Three types of control implications are:

Correct answer is A

A Require applications to be adequately tested


before use, backup of files and control access to
appropriate users
B Require backup of files, adequate documentation
and printer logs
C Control access to appropriate users, application
controls and decision trees
D Adequate documentation, application controls and
local area networks

(A) Control implications require applications to be


adequately tested before use, backup of files,
control access to appropriate users, adequate
documentation, and application controls are all
examples of control implications. Printer logs,
decision trees and local area networks are not.

Question 9

Answer 9

Which of the following statements are true?

Correct answer is C

A Software piracy may be avoided by eliminating


logs of software purchases.
B Auditing individual computers to identify macro
applications
C Software is copyrighted, and violation of copyright
laws may result in litigation against the company
D Establishing an industry software policy

(C) is true the other statements as incorrect. (A) is


incorrect because software piracy may be avoided
by maintaining rather than eliminating software
purchase logs. (B) is incorrect because computers
should be audited to identify install software rather
than macro applications and (D) is incorrect
because a corporate software rather than an
industry software policy should be established.

Question 10

Answer 10

Verification of processing involves all of the following except

Correct answer is B

A Prevents the system from being used for personal


projects
B Promotes the development of in-house developed
spreadsheets and databases because they are more
closely related to the needs of the business
C Should be performed periodically
D Prevents errors in internally developed software
from going undetected

(B) Controls should be in operation to assure the


accuracy of in-house developed spreadsheets and
databases. Although they may more closely
related to the needs of the business, verification of
processing is related to accuracy.

Question 11

Answer 11

Which of the following conditions is not an example of a


reliable system?

Correct answer is D

A Capable of operating without material error


B Capable of operating without fault
C Capable of operating without failure during a
specified period in a specified environment
D Capable of operating without error logs

A reliable system is one that operates without


material, fault, fault or failure during a specified
period in a specified environment. It should
generate error logs which can be related to
software and data problems. Because the question
here asks for the one that is NOT an example of a
reliable system, D is the answer.

Business Environment & Concepts > IT > IT Controls

Question 12

Answer 12

Which of the following statements is true?

Correct answer is A

A One framework for analyzing a reliable system is


presented by the AICPA's Trust Services.
B One framework for analyzing a reliable system is
presented by the AICPA's Treaty Services.
C One framework for analyzing a reliable system is
presented by the AICPA's Treatment Services.
D One framework for analyzing a reliable system is
presented by the AICPA's Tenancy Services.

(A) Trust Services provide assurance on


information systems.

Question 13

Answer 13

Which of the following principles is not included in the Trust


Services framework?

Correct answer is A

A Processing Integrity & Ethical Analysis


B On-line Privacy & Availability
C Processing Integrity & Confidentiality
D Security & Availability

(A) Ethical Analysis is not one of the principles of a


reliable system as defined by the framework
utilized by the AICPA

Question 14

Answer 14

The Security principle of a reliable system, examines whether


or not the system is protected against

Correct answer is B

A Unauthorized access (physical only)


B Unauthorized access (physical and logical)
C Unauthorized access (logical only)
D Authorized access (physical only)

(B) The Security principle of a reliable system,


examines whether or not the system is protected
against unauthorized access (both physical and
logical)

Question 15

Answer 15

Physical access controls prevent damage or other loss


including all but the following

Correct answer is D

A Weather
B Theft
C Disgruntled employees
D Computer based fraud

(D) Physical access controls prevent damage or


other loss including theft, acts of war, weather,
disgruntled employees or others

Question 16

Answer 16

Physical access controls prevent damage or other loss. Which


of the following is the best example

Correct answer is A

A Acts of War
B Unauthorized access to confidential data
C Malicious alteration of files
D Computer based fraud

(A) Physical access controls prevent damage or


other loss including theft, acts of war, weather,
disgruntled employees or others

Business Environment & Concepts > IT > IT Controls

Question 17

Answer 17

A lack of physical security that allows damage or other loss is

Correct answer is A

A An example of risk related to the Security principle


B An example of risk related to the Availability
principle
C An example of risk related to the Processing
Integrity principle
D An example of risk related to the Online Privacy
principle

(A) The Security principle requires that the system


is protected against unauthorized access, including
physical

Question 18

Answer 18

System failure that results in and interruption of business


operations or loss of data is

Correct answer is B

A An example of risk related to the Security principle


B An example of risk related to the Availability
principle
C An example of risk related to the Processing
Integrity principle
D An example of risk related to the Online Privacy
principle

(B) The Availability principle is concerned with the


availability of the system for operation and use as
committed or agreed in conformity with the entity's
availability policies.

Question 19

Answer 19

Invalid, incomplete, or inaccurate input data, data processing,


updating of master file or creation of output is

Correct answer is C

A An example of risk related to the Security principle


B An example of risk related to the Availability
principle
C An example of risk related to the Processing
Integrity principle
D An example of risk related to the Online Privacy
principle

(C) The Processing Integrity principle requires that


the system processing is complete, accurate,
timely and authorized.

Question 20

Answer 20

Disclosure of personal information of customers, or others,


such as social security numbers, credit card numbers, credit
rating and/or medical conditions is

Correct answer is D

A An example of risk related to the Security principle


B An example of risk related to the Availability
principle
C An example of risk related to the Processing
Integrity principle
D An example of risk related to the Online Privacy
principle

(D) The Online Privacy principle requires that


personal information obtained as a result of
e-commerce is collected, used, disclosed and
retained as agreed or committed.

Business Environment & Concepts > IT > IT Controls

Question 21

Answer 21

Disclosure of confidential information such as transaction


details, engineering details of products, business plans,
banking information, legal documents, inventory or other
account information, customer lists, or confidential details of
operations is

Correct answer is B
(B) The Confidentiality principle requires that
information designated as confidential is protected
as committed or agreed

A An example of risk related to the Security principle


B An example of risk related to the Confidentiality
principle
C An example of risk related to the Online Privacy
principle
D An example of risk related to the Availability
principle

Question 22

Answer 22

Examples of confidential data are

Correct answer is B

A Transaction details, engineering details of


products, and names of the board of directors
B Engineering details of products, legal documents
and customer lists
C Business plans, banking information and corporate
financial statements
D Inventory or other account information,
confidential details of operation and the website address

(B) Transaction details, engineering details of


products, business plans, banking information,
legal documents, inventory or other account
information, customer lists, or confidential details
of operations are all examples of confidential data.
The names of the members of the board of
directors, corporate financial statements and the
website address are not.

Question 23

Answer 23

The seven factors of the control environment are

Correct answer is A

A I - Integrity and ethical values C - Commitment to


competence H - Human resource policies and practices
A - Assignment of authority and responsibility M Management's philosophy and operating style B - Board
of director's or audit committee participation O Organization
B I - Integrity and ethical values C - Compensation
level and schedule H - Human resource policies and
practices A - Acceptance of accountability M Management's education and experience B - Board of
director's or audit committee participation O Organization
C I - Issue resolution and problem solving
techniques C - Commitment to competence H - Human
resource policies and practices A - Access to systems
and data M - Management style and structure B - Board
of director's or audit committee participation O Organization
D I - Integrity, trust and respect C - Commitment to
management goals H - Hiring practices and procedures
A - Allotment of resources M - Management's philosophy
and operating style B - Board of director's decision
making style O - Organization

(A) is the correct list of the seven factors of the


control environment

Business Environment & Concepts > IT > IT Controls

Question 24

Answer 24

What two distinct function involve the Information Systems


Department?

Correct answer is D

A Systems analysis and design


B Programming and systems support
C Systems support and data entry
D Systems development and data processing

(D) The information systems department is


involved with systems development and data
processing which are two distinct functions.

Question 25

Answer 25

Which of the following statements regarding segregation


controls is not true?

Correct answer is B

A Segregation controls segregate functions between


information departments and user departments
B Segregation of controls does not allow information
systems departments to initiate transactions but do
authorize transactions.
C Segregation of controls do not allow information
systems departments to initiate and authorize
transactions.
D Segregation of controls segregate programming,
operations and the library function within the information
systems department.

(B) Segregation of controls does not allow the


information systems department to authorize
transactions. Other valid segregation controls
segregate functions between information systems
departments and user departments, do not allow
information systems departments to initiate and
authorize transactions and segregate
programming, operations and the library function
within the information systems department.

Question 26

Answer 26

Which of the following is not an effective example of how


segregation of controls affects the organizational structure of
the information systems department?

Correct answer is B

A Systems development and data processing are


separate functions
B Database administration and data preparation
report to the same manager
C Operations and data library report to the same
manager
D Systems analysis and systems programming
report to the same manager

(B) Database administration typically falls under


systems development while data processing
typically falls under data processing. Systems
development and data processing should be
separate functions reporting to a single manager.
Functions under system development include
systems analysis, systems programming,
applications programming and database
administration. Functions under data processing
include data preparation, operations, data library
and data control.

Question 27

Answer 27

Which of the following is an effective example of how


segregation of controls affects the organizational structure of
the information systems department?

Correct answer is A

A Systems development and data processing


responsibilities are segregated functions
B Analysis and design responsibilities are
segregated functions
C Preparation and data entry responsibilities are
segregated functions
D programming and debugging responsibilities are
segregated functions

(A) Systems development and data processing


responsibilities are segregated functions. Systems
development and data processing should be
separate functions reporting to a single manager.
Functions under system development include
systems analysis, systems programming,
applications programming and database
administration. Functions under data processing
include data preparation, operations, data library
and data control.

Business Environment & Concepts > IT > IT Controls

Question 28

Answer 28

Which of the following is not an effective example of how


segregation of controls affects the organizational structure of
the information systems department?

Correct answer is C

A Functions under system development include


systems analysis, systems programming, applications
programming and database administration
B Functions under data processing include data
preparation, operations, data library and data control
C Functions under data processing include systems
analysis, systems programming, applications
programming and database administration
D Systems development and data processing
responsibilities are segregated functions.

(C) Systems development and data processing


should be separate functions reporting to a single
manager. Functions under system development
include systems analysis, systems programming,
applications programming and database
administration. Functions under data processing
include data preparation, operations, data library
and data control.

Question 29

Answer 29

Which of the following functions is responsible for the user


environment and requirements?

Correct answer is A

A Systems Analyst
B Database Administrator
C Systems Programmer
D Applications Programmer

(A) The systems analyst analyzes the user


environment and requirements and may
recommend changes to the current system, the
purchase of a new system or design a new system.
A systems flowchart is a tool or diagram used by
the systems analyst to define system
requirements.

Question 30

Answer 30

Which of the following is an example of a tool used by a


Systems Analyst?

Correct answer is C

A PERT Chart
B Gantt Chart
C Systems Flowchart
D Decision Tree

(C) The systems analyst analyzes the user


environment and requirements and may
recommend changes to the current system, the
purchase of a new system or design a new system.
The analysts is responsible for ensuring
programming and end user's needs are met. A
systems flowchart is a tool or diagram used by the
systems analyst to define system requirements.

Question 31

Answer 31

Which of the following functions is responsible for


implementing, debugging and modifying software?

Correct answer is C

A Systems Analyst
B Database Administrator
C Systems Programmer
D Applications Programmer

The systems programmer is responsible


implementing, modifying and debugging the
software required to interface with the hardware.
Examples include but are not limited to the
operating systems, telecommunications monitoring
and database management systems.

Business Environment & Concepts > IT > IT Controls

Question 32

Answer 32

Which of the following functions is responsible for writing


testing and debugging applications software?

Correct answer is D

A Systems Analyst
B Database Administrator
C Systems Programmer
D Applications Programmer

(D) The Applications Programmer is responsible


for writing testing and debugging applications
software. These specifications are usually provided
by the Systems Analyst. A program flowchart is a
tool or diagram used by the Applications
Programmer to document program logic.

Question 33

Answer 33

Which of the following functions is responsible for restricting


access to the database?

Correct answer is B

A Systems Analyst
B Database Administrator
C Systems Programmer
D Applications Programmer

(B) The Database Administrator or DBA is


responsible for maintaining the database and
restricting access to the database to authorized
users only.

Question 34

Answer 34

Which of the following is an example of a tool used by an


Applications Programmer?

Correct answer is A

A Program Flowchart
B Gantt Chart
C Systems Flowchart
D Decision Tree

(A) The Applications Programmer is responsible for


writing testing and debugging applications
software. A program flowchart is a tool or diagram
used by the Applications Programmer to document
program logic.

Question 35

Answer 35

Which function is generally responsible for providing


applications specifications to the Applications Programmer?

Correct answer is D

A End User
B Systems programmer
C Database Administrator
D Systems Analyst

(D) The Applications Programmer is responsible


for writing testing and debugging applications
software. These specifications are usually provided
by the Systems Analyst. A program flowchart is a
tool or diagram used by the Applications
Programmer to document program logic.

Question 36

Answer 36

Which function is generally responsible for preparing data to be


entered into the system?

Correct answer is B

A Database Administrator
B End Users
C Systems Programmer
D Data Operations

(B) Data may be prepared by End Users or user


departments and input to magnet tape or magnetic
disk.

Business Environment & Concepts > IT > IT Controls

Question 37

Answer 37

Which function is generally responsible for the daily computer


operations?

Correct answer is A

A Operator
B Database Administrator
C Systems Analyst
D Systems programmer

The Operator is responsible for the daily computer


operations of both the hardware and software. The
Operator mounts tapes, supervises operations on
a console, accepts inputs and distributes outputs.
The Operator should have documentation available
to run programs but is not responsible for detailed
program information.

Question 38

Answer 38

Which function should maintain documentation as to the


details of an applications program?

Correct answer is B

A Operations
B Applications Programmer
C Database Administrator
D End User

(B) The Applications Programmer is responsible for


writing testing and debugging applications
software. The Operator mounts tapes, supervises
operations on a console, accepts inputs and
distributes outputs. The Operator should have
documentation available to run programs but is not
responsible for detailed program information. The
Database Administrator or DBA is responsible for
maintaining the database and restricting access to
the database to authorized users only. The End
User may be responsible for the data but not the
programs that run the data.

Question 39

Answer 39

Help Desks are usually a responsibility of what function?

Correct answer is D

A Applications Programmer
B Database Administrator
C Outsourced Vendors
D Operations

(D) Help Desks are usually a responsibility of the


operations because of the operational nature of
their functions (e.g. assisting end users with
system problems and obtaining technical support)

Question 40

Answer 40

Which function is generally responsible for custody of


removable media?

Correct answer is B

A Operations
B Librarian
C Data Control
D Database Administrator

(B) The Librarian is responsible for custody of the


removable media (e.g. magnetic tape, disks) and
for the maintenance of program and systems
documentation. Many of these services have been
automated.

Question 41

Answer 41

Which function generally acts as the liaison between End


Users and the processing center?

Correct answer is B

A Librarian
B Data Control
C Database Administrator
D Operations

(B) The Control Group acts as the liaison between


End Users and the processing center. The group
records input data in a control log, follows the
progress of processing, distributes output and
ensures compliance with control totals.

Business Environment & Concepts > IT > IT Controls

Question 42

Answer 42

Ideally, functions in a control environment should be


segregated. This is not always possible. At a minimum, an
attempt should be made to segregate what three functions?

Correct answer is B

A analysis, design and programming


B programming, operations and the library
C database administration, programming and
analysis
D help desk, programming and design

Ideally, in a large company all of these functions


should be segregated. In a small company this
may not be possible. At a minimum, an attempt
should be made to segregate programming,
operations and the library.

Question 43

Answer 43

Which function is generally responsible for overseeing the


development, planning and the implementation of a website?

Correct answer is B

A Web Master
B Web Administrator
C Web Designer
D Web Coordinator

(B) The Web Administrator or Web Manager is


responsible of overseeing the development,
planning and the implementation of a website. This
is generally a management position.

Question 44

Answer 44

Which function is generally responsible for providing expertise


and leadership in the development of a website?

Correct answer is A

A Web Master
B Web Administrator
C Web Designer
D Web Coordinator

(A) The Web Master is responsible for providing


expertise and leadership in the development of a
website, including but not limited to design,
analysis, security, maintenance, content
development and updates.

Question 45

Answer 45

Which function is generally responsible for creating the visual


content of the website?

Correct answer is C

A Web Master
B Web Administrator
C Web Designer
D Web Coordinator

(C) The Web Designer is responsible for creating


the visual content of the website.

Question 46

Answer 46

Which function is generally responsible for the daily operations


of the website?

Correct answer is D

A Web Master
B Web Administrator
C Web Designer
D Web Coordinator

(D) The Web Coordinator is responsible for the


daily operations of the website

Business Environment & Concepts > IT > IT Controls

Question 47

Answer 47

Which function is generally responsible for writing programs for


commercial use?

Correct answer is C

A Systems Programmer
B Applications Programmer
C Internet Developer
D Intranet/Extranet Developer

(C) The Internet Developer is responsible for


writing programs for commercial use. Similar to a
software engineer or a systems programmer.

Question 48

Answer 48

Which function is generally responsible for writing programs


based on the needs of the company?

Correct answer is D

A Systems Programmer
B Applications Programmer
C Internet Developer
D Intranet/Extranet Developer

(D) The Intranet/Extranet Developer is responsible


for writing programs based on the needs of the
company.

Question 49

Answer 49

Risk assessment is a required component of internal control


because

Correct answer is A

A Changes in information systems and operations


may increase the risk of inaccurate financial data
B Change generally results in risk
C Changes in information systems may increase the
risk of inaccurate financial data. Changes in operations
are not a risk.
D Changes in operations may increase the risk of
inaccurate financial data. Changes in information
systems are not a risk.

(A) Changes in information systems and operations


may increase the risk of inaccurate financial data

Question 50

Answer 50

The size of a computer system may affect the accounting


systems because

Correct answer is B

A This is not true. There is no affect


B For small computer systems, purchased software
may be used and for large systems, software may be
developed "in-house"
C For small computer systems, "in house" software
may be used and for large systems, software may be
purchased.
D A variety of accounting software is not available
for smaller systems. There is a larger selection of
software to choose from for larger systems.

(B) The size of a computer system may affect the


accounting systems because a firm may purchase
software for small systems but develop their own
software for larger systems. Software developed by
a company's personnel is sometimes referred to as
"in-house" developed software.

Business Environment & Concepts > IT > IT Controls

Question 51

Answer 51

It may be easier for an auditor to audit smaller systems


because

Correct answer is A

A The auditor may be familiar with the software


B There are fewer tests to run
C Additional controls are in place
D There are fewer company personnel with access

(A) The size of a computer system may affect the


accounting systems because a firm may purchase
software for small systems but develop their own
software for larger systems. The auditor may be
familiar with the purchased software and the
"exception reports" may be standard and well
tested. The auditor may not be familiar with "in
house" developed software and although
exception reports may exist, controls should be
tested to a greater extent.

Question 52

Answer 52

Systems control activities can generally be divided into what


three categories?

Correct answer is B

A Internal, external and management


B general, application and user
C financial, cost and variance
D data, programming and access

(B) Control activities can generally be divided into


three categories: general, application and user.

Question 53

Answer 53

Application control activities can be divided into what two


categories?

Correct answer is D

A access control and user sign-ons


B security systems and databases logging
C analysis and design
D program controls and exception reports

(D) Program controls and exception reports.


Program controls relate to specific applications and
are a part of the computer program. Exception
reports require manual follow up and review of the
items listed on the exception reports.

Question 54

Answer 54

General system control activities can be divided into what four


categories?

Correct answer is B

A 1-hardware lease/buy analysis, 2-software


purchase/develop analysis, 3-access screening and
4-data control
B 1-developing new programs and systems,
2-changing existing programs and systems, 3-controlling
access to programs and data and 4-controlling computer
operations
C 1-evaluating new programs and systems,
2-documenting programs and systems, 3-controlling
output access and 4-controlling computer costs
D 1-Analyzing new programs and systems,
2-evaluating existing programs and systems,
3-controlling functional output from programs and
4-auditing transmission results

(B) General system control activities affect all


computer applications and can be divided into the
following four categories: 1-developing new
programs and systems, 2-changing existing
programs and systems, 3-controlling access to
programs and data and 4-controlling computer
operations.

Business Environment & Concepts > IT > IT Controls

Question 55

Answer 55

SAS No. 78 Consideration of Internal Control in a Financial


Statement Audit identifies what five components of internal
control?

Correct answer is A

A Control Environment, Risk Assessment, Control


Activities, Information and Monitor
B Applications, Database Management, Segregation
of Duties, Security and Revision
C Input, Output, Review, Approval and Distribution
D Foundation, Risk , Access, Identification and
Modification

(A) SAS No. 78 Consideration of Internal


Control in a Financial Statement Audit identifies
what five components of internal control: a) Control
Environment this is the foundation of all other
components; b) Risk Assessment the
identification and analysis of relevant risks to
achieve the entitys objectives c) Control Activities
the policies and procedures to ensure
management directives are carried out d)
Information and Communication identification,
capture, and exchange of data in a format and time
to allow proper tasks and responsibilities to be
performed e) Monitor The process that assesses
the quality of internal control performance over
time.

Question 56

Answer 56

SAS No. 78 a definition of the control environment which is


one of the five components of Consideration of Internal
Control in a Financial Statement is:

Correct answer is A

A The foundation of all other components


B The identification and analysis of relevant risks to
achieve the entitys objectives
C The policies and procedures to ensure
management directives are carried out
D The identification, capture, and exchange of data
in a format and time to allow proper tasks and
responsibilities to be performed

(A) SAS No. 78 Consideration of Internal


Control in a Financial Statement Audit identifies
what five components of internal control: a) Control
Environment this is the foundation of all other
components; b) Risk Assessment the
identification and analysis of relevant risks to
achieve the entitys objectives c) Control Activities
the policies and procedures to ensure
management directives are carried out d)
Information and Communication identification,
capture, and exchange of data in a format and time
to allow proper tasks and responsibilities to be
performed e) Monitor The process that assesses
the quality of internal control performance over
time.

Question 57

Answer 57

SAS No. 78 a definition of risk assessment which is one of


the five components of Consideration of Internal Control in a
Financial Statement is:

Correct answer is C

A The foundation of all other components


B The process that assesses the quality of internal
control performance over time.
C The identification and analysis of relevant risks to
achieve the entitys objectives
D The identification, capture, and exchange of data
in a format and time to allow proper tasks and
responsibilities to be performed

(C) SAS No. 78 Consideration of Internal


Control in a Financial Statement Audit identifies
what five components of internal control: a) Control
Environment this is the foundation of all other
components; b) Risk Assessment the
identification and analysis of relevant risks to
achieve the entitys objectives c) Control Activities
the policies and procedures to ensure
management directives are carried out d)
Information and Communication identification,
capture, and exchange of data in a format and time
to allow proper tasks and responsibilities to be
performed e) Monitor The process that assesses
the quality of internal control performance over
time.

Business Environment & Concepts > IT > IT Controls

Question 58

Answer 58

SAS No. 78 a definition of the control activities which is one


of the five components of Consideration of Internal Control in
a Financial Statement is:

Correct answer is D

A The foundation of all other components


B The process that assesses the quality of internal
control performance over time.
C The identification and analysis of relevant risks to
achieve the entitys objectives
D The policies and procedures to ensure
management directives are carried out

(D) SAS No. 78 Consideration of Internal


Control in a Financial Statement Audit identifies
what five components of internal control: a) Control
Environment this is the foundation of all other
components; b) Risk Assessment the
identification and analysis of relevant risks to
achieve the entitys objectives c) Control Activities
the policies and procedures to ensure
management directives are carried out d)
Information and Communication identification,
capture, and exchange of data in a format and time
to allow proper tasks and responsibilities to be
performed e) Monitor The process that assesses
the quality of internal control performance over
time.

Question 59

Answer 59

SAS No. 78 a definition of information and communication


which is one of the five components of Consideration of
Internal Control in a Financial Statement is:

Correct answer is A

A The identification, capture, and exchange of data


in a format and time to allow proper tasks and
responsibilities to be performed
B The foundation of all other components
C The identification and analysis of relevant risks to
achieve the entitys objectives
D The policies and procedures to ensure
management directives are carried out

(A) SAS No. 78 Consideration of Internal


Control in a Financial Statement Audit identifies
what five components of internal control: a) Control
Environment this is the foundation of all other
components; b) Risk Assessment the
identification and analysis of relevant risks to
achieve the entitys objectives c) Control Activities
the policies and procedures to ensure
management directives are carried out d)
Information and Communication identification,
capture, and exchange of data in a format and time
to allow proper tasks and responsibilities to be
performed e) Monitor The process that assesses
the quality of internal control performance over
time.

Question 60

Answer 60

SAS No. 78 a definition of monitor which is one of the five


components of Consideration of Internal Control in a
Financial Statement is:

Correct answer is B

A The foundation of all other components


B The process that assesses the quality of internal
control performance over time.
C The identification, capture, and exchange of data
in a format and time to allow proper tasks and
responsibilities to be performed
D The policies and procedures to ensure
management directives are carried out

(B) SAS No. 78 Consideration of Internal


Control in a Financial Statement Audit identifies
what five components of internal control: a) Control
Environment this is the foundation of all other
components; b) Risk Assessment the
identification and analysis of relevant risks to
achieve the entitys objectives c) Control Activities
the policies and procedures to ensure
management directives are carried out d)
Information and Communication identification,
capture, and exchange of data in a format and time
to allow proper tasks and responsibilities to be
performed e) Monitor The process that assesses
the quality of internal control performance over
time.

Business Environment & Concepts > IT > IT Controls

Question 61

Answer 61

Internal control system failure Even the best-designed


control systems are subject to failure due to:

Correct answer is C

A human error, lack of experience, poor training and


unethical behavior
B Board of Director authorizations, management
override, white collar crime and faulty judgment
C human error, faulty judgment, collusion and
management override
D greed, envy, gluttony and pride

(C) Internal control system failure Even the


best-designed control systems are subject to
failure due to: human error, faulty judgment,
collusion and management override

Question 62

Answer 62

Internal Control Goals process objectives that internal


control systems are designed to achieve include Operations
and Information process goals. Three examples of Operations
goals are:

Correct answer is A

A Effectiveness of operations, Efficient resources


and Security of resources
B Input validity, Input completeness and Input
accuracy
C Input validity, Input completeness and Security of
resources
D Effectiveness of operations, Update completeness
and Update accuracy

(A) Internal Control Goals process objectives


that internal control systems are designed to
achieve include Operations and Information
process goals. Operations Process Goals should
ensure: (1) Effectiveness of operations Strives to
ensure than an intended process is fulfilling its
intended purpose (such as proper management
authorization for overrides) (2) Efficient resources
to have enough resources to ensure benefits
exceed costs of controls (3) Security of resources
protect all tangible and intangible resources.

Question 63

Answer 63

Internal Control Goals process objectives that internal


control systems are designed to achieve include Operations
and Information process goals. Five examples of Information
goals are:

Correct answer is B

A Effectives of operations, Data Security, Input


accuracy, Update completeness and Update accuracy
B Input validity, Input completeness, Input accuracy,
Update completeness and Update accuracy
C Effectiveness of operations, Efficient resources,
Security of resources, Input validity and Update
completeness
D Effectiveness of operations, Efficient resource,
Input accuracy, Update completeness and Update
accuracy

(B) Internal Control Goals process objectives


that internal control systems are designed to
achieve include Operations and Information
process goals. Information Process Control Goals
should ensure: (1) Input validity where input data
be approved and reflect accurate economic events
(2) Input completeness all valid events are
captured (3) Input accuracy all events are
captured correctly (4) Update completeness all
events are reflected in respective master files (5)
Update accuracy all events are reflected
correctly within master file.

Business Environment & Concepts > IT > IT Controls

Question 64

Answer 64

Which of the following statements regarding control plans is


not true?

Correct answer is D

A Control Plans are policies and procedures that


assist in accomplishing control goals.
B Three levels of control plans are Control
Environment (top level), Pervasive Control Plans
(mid-level) and Application Control (detail level) Plans.
C Another way to view control plans is in relation to
the timing of their occurrence. Preventive control plans
stop problems from occurring; Detective control plans
discover problems that have already occurred;
Corrective control plans correct problems that have
already occurred.
D The Preventive Audit set of policies and
procedures is the only 100% effective control plan.

Control Plans are policies and procedures that


assist in accomplishing control goals. No control
plan is 100% effective. A combination of plans
must be used to maximize effectiveness. Three
levels are Control Environment (top level),
Pervasive Control Plans (mid-level) and
Application Control (detail level) Plans. Another
way to view control plans is in relation to the timing
of their occurrence. Preventive control plans stop
problems from occurring; Detective control plans
discover problems that have already occurred;
Corrective control plans correct problems that have
already occurred.

Question 65

Answer 65

Which of the following statements regarding Control Objectives


for Information and Related Technology - [cobiT] developed
by the Information Systems Audit and Control Foundation is
not true?

Correct answer is D

A The objectives were developed to provide


guidance on best practices for management and
Information technology.
B [cobiT] groups IT control processes into four
domains: a) Planning & Organization, b) Acquisition &
Implementation, c) Delivery & Support and d) Monitor
Monitor operations.
C The goal of the objectives is to establish strategic
vision for the IT area, identify automated and IT
solutions, deliver required IT services and to monitor
operations.
D The objectives were developed to provide an audit
trail of the accounting information system and the
adherence to best practices for management and
Information technology.

(D) Option D is gibberish. Control Objectives for


Information and Related Technology - [cobiT]
developed by the Information Systems Audit and
Control Foundation to provide guidance on best
practices for management and Information
technology. [cobiT] groups IT control processes
into four domains: a) Planning & Organization
Establish strategic vision for the IT area; develop
plan to achieve vision. b) Acquisition &
Implementation Identify automated and IT
solutions; integrate the solutions; manage changes
to existing systems; Manage Change with users. c)
Delivery & Support Deliver required IT services;
ensure security; provide on-going support. d)
Monitor Monitor operations.

Business Environment & Concepts > IT > IT Controls

Question 66

Answer 66

Which of the following statements regarding an audit trail is not


true?

Correct answer is B

A The audit trail is a record left by the accounting


information system of movements in individual
transaction data. This record provides a trail of the
processing of transactions and other events
B An audit trail does not allow for means to trace
back to individual business events from the general
ledger.
C An audit trail may start from the moment data
about the event is entered into the system until the final
entry is made in the financial statements.
D The audit trail of a sales transaction may enable
the tracing of the movement in data concerning the
transaction from the time the order is placed by the
customer until the time the payment data is entered in
the general ledger accounts.

(B) The audit trail is a record left by the accounting


information system of movements in individual
transaction data. This record, in the form of
references to the processing of the data, provides
a trail of the processing of transactions and other
events entered into by the entity. Note that while
some accounting information systems provide a
visible and complete audit trail, others may provide
an invisible and/or incomplete trail. Depending on
the accounting information system, the trail may
start from the moment data about the event is first
captured within the system to the time of its
ultimate disposition in the financial statements. For
example, the audit trail of a sales transaction may
enable the tracing of the movement in data
concerning the transaction from the time the order
is placed by the customer until the time the
transaction data is included in the appropriate
general ledger accounts. An audit trail should allow
for means to trace back to individual business
events from the general ledger. An auditor may
follow the audit trail of a transaction as part of a
systems "walk-through".

Question 67

Answer 67

Three types of system documentation used by auditors and


analysts are:

Correct answer is A

A Data Flow Diagrams, System Flowcharts and


Entity Relationship Diagrams
B Program Flowcharts, System Flowcharts and
Process Flow Diagrams
C Swim Lanes, Gantt Charts and PERT Charts
D Run Charts, Error Logs and System Cycle Time
Charts

(A) Transaction/Data Flow Auditors and analysts


document information systems to understand,
explain, and improve complex business processes
and operations. Three main types of system
documentation used by auditors and analysts are
(a) Data Flow Diagrams (DFDs) that illustrate the
system components and functions, data flows
among the components and sources, destinations
and storage of the data; (b) System Flowcharts that
illustrate Informational Processes (such as logic
flows, inputs, outputs, data storage), Operational
Processes (such as physical flows) and (c) Entity
Relationship Diagrams that illustrate the systems
key entities and the relationships among those
entities.

Business Environment & Concepts > IT > IT Controls

Question 68

Answer 68

Which of the following statements regarding Systems


Flowcharts is true?

Correct answer is B

A illustrate the system components and functions,


data flows among the components and sources,
destinations and storage of the data
B illustrate Informational Processes (such as logic
flows, inputs, outputs, data storage), Operational
Processes (such as physical flows
C illustrate the systems key entities and the
relationships among those entities
D Illustrate the access to data from applications
programs via interfaces and manual entry

(B) Transaction/Data Flow Auditors and analysts


document information systems to understand,
explain, and improve complex business processes
and operations. Three main types of system
documentation used by auditors and analysts are
(a) Data Flow Diagrams (DFDs) that illustrate the
system components and functions, data flows
among the components and sources, destinations
and storage of the data; (b) System Flowcharts that
illustrate Informational Processes (such as logic
flows, inputs, outputs, data storage), Operational
Processes (such as physical flows) and (c) Entity
Relationship Diagrams that illustrate the systems
key entities and the relationships among those
entities. Option D is gibberish

Question 69

Answer 69

Which of the following statements regarding Entity


Relationship Diagrams is true?

Correct answer is C

A illustrate the system components and functions,


data flows among the components and sources,
destinations and storage of the data
B illustrate Informational Processes (such as logic
flows, inputs, outputs, data storage), Operational
Processes (such as physical flows
C illustrate the systems key entities and the
relationships among those entities
D Illustrate the access to data from applications
programs via interfaces and manual entry

(C) Transaction/Data Flow Auditors and analysts


document information systems to understand,
explain, and improve complex business processes
and operations. Three main types of system
documentation used by auditors and analysts are
(a) Data Flow Diagrams (DFDs) that illustrate the
system components and functions, data flows
among the components and sources, destinations
and storage of the data; (b) System Flowcharts that
illustrate Informational Processes (such as logic
flows, inputs, outputs, data storage), Operational
Processes (such as physical flows) and (c) Entity
Relationship Diagrams that illustrate the systems
key entities and the relationships among those
entities. Option D is gibberish

Question 70

Answer 70

Which of the following statements regarding Data Flow


Diagrams is true?

Correct answer is A

A illustrate the system components and functions,


data flows among the components and sources,
destinations and storage of the data
B illustrate Informational Processes (such as logic
flows, inputs, outputs, data storage), Operational
Processes (such as physical flows
C illustrate the systems key entities and the
relationships among those entities
D Illustrate the access to data from applications
programs via interfaces and manual entry

(A) Transaction/Data Flow Auditors and analysts


document information systems to understand,
explain, and improve complex business processes
and operations. Three main types of system
documentation used by auditors and analysts are
(a) Data Flow Diagrams (DFDs) that illustrate the
system components and functions, data flows
among the components and sources, destinations
and storage of the data; (b) System Flowcharts that
illustrate Informational Processes (such as logic
flows, inputs, outputs, data storage), Operational
Processes (such as physical flows) and (c) Entity
Relationship Diagrams that illustrate the systems
key entities and the relationships among those
entities. Option D is gibberish

Business Environment & Concepts > IT > IT Controls

Question 71

Answer 71

Which of the following statements relating to segregation


control is not true?

Correct answer is D

A User departments participate in systems design


B Both users and information systems personnel test
new systems
C Management, users and information systems
personnel approve new systems before they are placed
into operation
D All master and transaction file conversions should
be created by the user department

(D) Options A through C are true . All master and


transaction file conversions should be controlled to
prevent unauthorized changes and to verify the
accuracy of the results. File conversions are
generally overseen by Information systems
personnel.

Question 72

Answer 72

Computer hardware is extremely reliable because:

Correct answer is B

A Silicon gaskets prevent the effects of moisture


B Chip Technology
C Stronger plastics encase the hard drive
D Cooling systems technology has improved

(B) Computer hardware is extremely reliable,


primarily due to chip technology.

Question 73

Answer 73

Which of the following diagnostics is not an example of self


diagnostics utilized by computer hardware?

Correct answer is C

A Diagnostic Routines and Boundary Protection


B Echo and Parity Checks
C CT and Memory Scans
D Periodic or Preventative Maintenance

(C) Although chip technology has improved


drastically, hardware has also improved to include
self diagnostics. The following controls are
examples of these diagnostics. Diagnostic
Routines - provided by the manufacturer and
activated when the system is booted up; Boundary
protection - maintains separate areas of
processing so that multiple programs can run
simultaneously; Echo Check - used in
transmissions where the receiving hardware sends
back the data received as a confirmation; Parity
check - one bit is added to a block of bits so that
the ones in the block always add up to either an
odd or even number; Periodic or Preventative
maintenance - the systems is serviced regularly to
ensure that it is operating as intended.

Question 74

Answer 74

Which of the following actions is not required when changing


an existing program or system?

Correct answer is D

A The change should be reviewed by the


information system manager
B The change should be tested
C The change should be documented
D The change should be approved by the CIO

(D) Changes should be approved but not


necessarily by the CIO. All other options are
recommended as a part of the change control
process.

Business Environment & Concepts > IT > IT Controls

Question 75

Answer 75

Controlling access to programs and data can be accomplished


by:

Correct answer is A

A Segregation controls, physical access controls


and hardware and software access controls
B Backup and recovery, Contingency processing,
File protection rings and internal and external labels
C Preprinted forms, check digits, control., batch and
proof totals, hash totals, record counts and limit or
reasonable tests
D Menu driven input, field and validity checks,
missing data and field size checks and logic checks

(A) Segregation controls, physical access controls


and hardware and software access controls are all
methods to control access to programs and data.
Option B represents methods to control computer
operations. Options C and D represent methods to
validate or edit input.

Question 76

Answer 76

Controlling computer operations can be accomplished by:

Correct answer is B

A Segregation controls, physical access controls


and hardware and software access controls
B Segregation controls, Backup and recovery,
Contingency processing, File protection rings and
internal and external labels
C Preprinted forms, check digits, control., batch and
proof totals, hash totals, record counts and limit or
reasonable tests
D Menu driven input, field and validity checks,
missing data and field size checks and logic checks

(B) Segregation controls, Backup and recovery,


Contingency processing, File protection rings and
internal and external labels represent methods to
control computer operations. Option A represents
controlling access to programs and data. Options C
and D represent methods to validate or edit input.

Question 77

Answer 77

Examples of input validation or edit controls include all but the


following

Correct answer is D

A Preprinted forms, check digits, control., batch and


proof totals, hash totals, record counts and limit or
reasonable tests
B Menu driven input, field and validity checks,
missing data and field size checks and logic checks
C Redundant data checks and closed loop
verification
D Segregation controls, physical access controls
and hardware and software access controls

(D) Options A, B and C are all examples of input


validation or edit controls. Option D lists examples
of methods that control access to programs and
data.

Business Environment & Concepts > IT > IT Controls

Question 78
User control activities test the completeness and accuracy of
computer processing controls. Which of the following lists is
not an example of a user control activity?
A Checks of computer output aginst source
documents, control totals or other input
B Reviewing computer logs
C Policies and procedures that document authorized
users and receipients of data
D Preprinted forms, check digits, control., batch and
proof totals, hash totals, record counts and limit or
reasonable tests

Answer 78
Correct answer is D
(D) Options A, B and C are all examples of user
control activities. Option D lists examples of input
validation or edit controls.

Question 79

Answer 79

A company uses computer processing to calculate and print out


employee paychecks each period. The company is worried that
data will be entered into the system incorrectly so that one or
more paychecks will be overstated as a means of stealing
money from the company. Which of the following is most likely
to detect such fraud?

Correct answer is A

A Control totals are calculated for significant figures.


B All paychecks must be hand delivered to the
employee by a paymaster.
C A system is devised so that employees who are
overpaid can file a form to reimburse the company.
D Checks that are not picked up by an employee are
given to a person in authority to uncover the reason.

Question 80
A company has asked one of its computer programmers to
create a new program to compute and print payroll checks for
each of its 8,000 hourly employees. In writing this program,
the programmer includes a step where an error report must be
produced for any employee who is to receive payment based
on a rate that is below the federal minimum wage rate. What
is this type of control called?
A Legality implementation control
B Minimal control
C Limit test
D Pay incentive test

Companies worry that numbers such as hours


worked or pay rates will be increased in a scheme
so that employees can steal money. For example,
if 50 hours is entered rather than 40 hours for an
employee, that person will get 25 percent more
than has been earned. Thus, company officials
determine control figures before computations are
made and then verify those totals after the
computations are made to ensure that no
unauthorized changes have been made. Answers
B and D are important controls but they are
verifying that paychecks are written to actual
employees rather than that the amounts have not
been manipulated.

Answer 80
Correct answer is C
In writing computer programs, programmers often
insert high and low barriers to make sure that any
errors are caught if they are especially large.
These controls are referred to as limit tests
because they place limits around the amounts that
can be entered at certain points. Therefore, the
programmer might well require that additional
processing would be necessary if a paycheck was
(for example) above $50.00 per hour or below the
minimum federal wage rate. Those would be limit
tests.

Business Environment & Concepts > Planning, Control and Analysis > Basic Cost Terminology

Basic Cost Terminology


Question 1

Answer 1

A company is building a widget. During the current year, a


widget requires direct materials with a cost of $500. The
company must also spend $400 for direct labor. Finally, factory
overhead related to the production of a widget is $300. What is
the prime cost associated with a widget?

Correct answer is A
Prime costs are easily traceable to specific units of
production and include direct labor and direct
materials. In this case, that is $500 plus $400 or
$900.

A $900
B $700
C $1,200
D $500

Question 2

Answer 2

A company is building a widget. During the current year, a


widget requires direct materials with a cost of $600. The
company must also spend $200 for direct labor. Finally, factory
overhead related to the production of a widget is $100. What is
the conversion cost associated with a widget?

Correct answer is C

A $100
B $200
C $300
D $800

Conversion cost include direct labor and factory


overhead. They are the cost to convert materials
into finished products. Here, those costs are $200
for labor and $100 for overhead for a total of $300.

Business Environment & Concepts > Planning, Control and Analysis > Cost-Volume-Profit Analysis

Cost-Volume-Profit Analysis
Question 1

Answer 1

The difference between total sales in dollars and total variable


expenses is called:

Correct answer is D

A net operating income.


B net profit.
C the gross margin.
D the contribution margin.

Question 2

Answer 2

A company had sales last year that earned a contribution


margin of $600,000. That was 30 percent of the sales for the
period. The company also had net income of $420,000. What is
the break-even point for this company in sales dollars?

Correct answer is C

A $420,000
B $540,000
C $600,000
D $660,000

Contribution margin is sales minus variable cost. If


contribution margin is $600,000 and net income is
$420,000, the fixed cost for the company must
have been $180,000. The contribution margin is 30
percent of sales. To generate enough revenue to
earn a contribution margin of $180,000 so that net
income is zero (the break-even point), the
company needs sales of $180,000/30 percent or
$600,000.

Question 3

Answer 3

If the level of activity increases within the relevant range:

Correct answer is C

A variable cost per unit and total fixed costs also


increase.
B fixed cost per unit and total variable cost also
increase.
C total cost will increase and fixed cost per unit will
decrease.
D variable cost per unit and total cost also increase

By definition, if the level of activity increases within


the relevant range total cost will increase and fixed
unit cost per unit will decrease. Variable cost will
remain same per unit.

Question 4

Answer 4

Total Fixed cost along the relevant range will:

Correct answer is B

A increases as volume increases


B stay the same as volume increases
C Decrease as volume increase
D are not associated with volume

By definition total fixed cost does not vary in with


the level of volume along the relevant range.

Question 5

Answer 5

Fixed unit cost along the relevant range will:

Correct answer is C

A increases as volume increases


B stay the same as volume increases
C Decrease as volume increase
D are not associated with volume

By definition fixed unit cost decreases as the level


of volume increase along the relevant range. (Total
fixed cost / Relevant Range Volume = Fixed unit
cost)

Business Environment & Concepts > Planning, Control and Analysis > Cost-Volume-Profit Analysis

Question 6

Answer 6

Variable cost along the relevant range with:

Correct answer is A

A increases as volume increases


B stay the same as volume increases
C Decrease as volume increase
D are not associated with volume

By definition variable cost decreases as the level of


volume increase along the relevant range. (Total
fixed cost / Relevant Range Volume = Fixed unit
cost)

Question 7

Answer 7

How does variable unit cost change as production moves along


the relevant range for a product?

Correct answer is B

A It increases as volume increases


B It stays the same as volume increases
C It decreases as volume increase
D Increases or decreases will vary based on the
type of product.

The variable cost per unit for a product (assume,


for example, $3.00 per unit) will stay at that rate as
long as production is within a relevant range. Total
variable cost goes up as production increases but
the variable cost per unit remains at $3.00.

Question 8

Answer 8

When applying Cost Volume Profit (CVP) to specific cases, it is


important to keep in mind some underlying assumptions.
Which is not an assumption:

Correct answer is C

A selling price does not change with activity level


B The sales mix remains constant
C Fixed unit Cost are constant
D Cost can be separated into fixed and variable
elements

When applying CVP to a specific case and in


interpreting the results there from, it is important to
keep in mind the underlying assumptions: 1)
selling price does not change with activity level, 2)
The sales mix remains constant 3) Cost can be
separated into fixed and variable elements 4) Total
fixed Cost are constant 5) Variable Cost per unit
are constant

Question 9

Answer 9

A company produces 10 different products in 10 different


plants. From the items produced in Plant 8, the company
makes sales of $900,000. Variable costs are $300,000 in that
plant and the fixed costs are $700,000. Because the plant is
losing $100,000 per year, company officials close it at the start
of the current year and lease the building to a competitor for a
flat $360,000 per year ($30,000 per month). How does this
decision impact the companys overall net income for this
year?

Correct answer is D

A Net income will go up by $660,000.


B Net income will go up by $700,000.
C Net income will go down by $40,000.
D Net income will go down by $240,000.

Currently, the company is losing $100,000 from


Plant 8. Unless stated otherwise, the fixed costs
will not be eliminated by closing the plant. Such
costs include property taxes and insurance that are
often unaffected by revenues and other variables.
After closing, revenues will be $360,000 from the
lease but the fixed expenses of $700,000 will
remain so that the company will lose $340,000.
Going from a loss of $100,000 to a loss of
$340,000 reduces the companys net income by
$240,000.

Business Environment & Concepts > Planning, Control and Analysis > Cost-Volume-Profit Analysis

Question 10
Last year, a company made 100,000 widgets and sold them for
$20 each. The fixed costs were $500,000 and the company
made a profit of $100,000. In the new year, the company
plans to raise the price to $25 per unit but sell only 90,000.
What is the anticipated profit?
A $110,000
B $250,000
C $320,000
D $490,000

Answer 10
Correct answer is D
Last year the company had sales of $2 million
(100,000 times $20 each). Fixed costs were
$500,000 so the amount left over to cover variable
costs and produce a profit was $1.5 million. The
profit was $100,000 so the variable cost must have
been $1.4 million. That is $14 per unit (since it is a
variable cost). This year, sales are $2,250,000
($25 times 90,000) but fixed costs (because they
are fixed) remain at $500,000. Variable cost is $14
per unit or $1,260,000 ($14 times 90,000). The
profit is $490,000 ($2,250,000 less $1,260,000 and
less $500,000).

Business Environment & Concepts > Planning, Control and Analysis > Activity Based Costing

Activity Based Costing


Question 1

Answer 1

Activity-based costing is based on two principles. First,


activities consume resources. Second,

Correct answer is B

A these resources are consumed by business units.


B these resources are consumed by products,
services, or other cost objectives.
C these resources are consumed in direct portion to
Direct Labor.
D these resources are consumed batch size.

By definition, Activity based-costing (ABC) is based


upon two principles. First, activities consume
resources. Second, these resources are consumed
by products, services, or other cost objectives
(output).

Question 2

Answer 2

Activity Based Costing is typically used to distribute which of


the following cost to products and services?

Correct answer is D

A Conversion Cost
B Marketing Cost
C Cost of Goods Sold
D Overhead Cost

ABC allocates overhead costs to products on the


basis of the resources consumed by each activities
cost driver.

Question 3

Answer 3

The Waterson Company produces widgets by combining three


pounds of material with four hours of labor. Widgets are first
designed by company engineers and then produced in the
companys manufacturing facility. The company is currently
making several thousand Blue45 widgets. A cost of $4,000 has
been assigned to the production of these widgets for each of
the 12 hours of design time that was required. The inclusion of
the design time in the cost of these units indicates that which of
the following is most likely being applied?

Correct answer is C

A Traditional process cost accounting


B Traditional job order cost accounting
C Traditional activity-based cost accounting
D Traditional standard cost accounting

Traditional process, job order, and standard cost


accounting have always focused exclusively on the
manufacturing process. Raw materials are turned
into a final product and the material, labor, and
factory overhead are included in arriving at a cost
figure. However, activity-based costing extends
that definition to include all value-adding activities
such as the design function. Designing Blue45
widgets is a cost of this product even though it is
not a manufacturing cost.

Business Environment & Concepts > Planning, Control and Analysis > Activity Based Costing

Question 4

Answer 4

A company produces a product in a long-series of automated


steps that requires little direct material and direct labor but a lot
of overhead. What costing system is the company most likely to
use?

Correct answer is C

A Job-order costing
B Process costing
C Activity-based costing
D Just-in-time costing

In this situation, a company might use job-order


costing or process costing. There is not enough
information to tell if products are unique (where
job-order costing is appropriate) or mass produced
(where process costing is appropriate). However,
in connection with either of these approaches, the
company is likely to apply activity-based costing
because there are many steps in the process and
overhead is a significant cost. Activity-based
costing breaks the process down into individual
activities and determines a specific overhead cost
driver for each of these activities. In that way, the
costing is designed specifically for the process in
use. Overhead for one activity might be applied
based on machine hours if that is appropriate to
the activity whereas overhead for another activity
might be applied based on direct labor cost if that
is appropriate.

Business Environment & Concepts > Planning, Control and Analysis > Budgeting and Responsibility Accounting

Budgeting and Responsibility Accounting


Question 1

Answer 1

The Master budget is a completive expression of


management's:

Correct answer is C

A operating budget only.


B financial budget only
C operating and financial budgets.
D view of controller cost.

By definition, a master budget is a comprehensive


expression of management's operating and
financial plans for a future period that is
summarized as budgeted financial statements. It
consists of an operating and financial budget.

Question 2

Answer 2

Cost Volume Profit analysis provides management with:

Correct answer is D

A costing information for their operating budget.


B fixed cost per unit and total variable cost only.
C variable cost per unit and total fixed cost only.
D profitability estimates at all levels of production in
the relevant range.

Question 3

Answer 3

The Hadley Company produces widgets which are seasonal


products. Therefore, the company produces 10,000 units per
month for the first six months of the year (January through
June) and then 30,000 units per month for the last six months
of the year (July through December). It takes three pounds of
product XYZ to produce one widget. The company plans to
start each month with enough of product XYZ for the next two
months so that shortages are avoided. How much product XYZ
should the company plan to acquire during the month of May?

Correct answer is D

A 20,000 pounds
B 30,000 pounds
C 60,000 pounds
D 90,000 pounds

Question 4
A company manufactures widgets. A widget takes 2 pounds
of material at $9 per pound and 3 hours of labor at $16 per
hour. Factory overhead is applied at the rate of $6 per direct
labor hour. The company wants to have a gross profit
percentage of 30 percent. What should the company set as
its sales price for each widget?
A $109.20
B $115.40
C $118.60
D $120.00

The company starts May expecting to make 20,000


units over the next two months (10,000 in May and
10,000 in June). Thus, 60,000 pounds of material
(at three pounds per unit) should be on hand to
make those units. During May, 10,000 units will be
produced and use up 30,000 pounds of the
material, leaving 30,000 pounds on hand. At the
beginning of June, the company anticipates
making 40,000 units over the subsequent two
months (10,000 in June and 30,000 in July). On
that date, the company wants to hold 120,000
pounds of material (at three pounds per widget).
To raise the quantity of material from 30,000
pounds to the desired 120,000 pounds as of the
beginning of June, an additional 90,000 must be
bought during May.

Answer 4
Correct answer is D
Each widget has a total cost of $84.00. That is
direct material of $18 (2 times $9), direct labor of
$48 (3 times $16), and factory overhead of $18 (3
times $6). Because the company wants a gross
profit percentage of 30 percent, the sales prices
(SP) should be: SP 84 = .30SP. Or, with
rearrangement: .70 SP = 84. By dividing both
sides by .70, SP = 120. That is the sales price to
get that gross profit percentage.

Business Environment & Concepts > Planning, Control and Analysis > Variable and Absorption (full) costing

Variable and Absorption (full) costing


Question 1
An accountant for Rembrandt Corporation is trying to
determine whether to use variable costing or absorption costing
when calculating income internally for a project. Which of the
following statements is not true?
A variable costing considers fixed manufacturing
cost as a product cost.
B variable costing considers fixed manufacturing
cost as a period cost.
C absorption costing is acceptable for external
reporting.
D absorption costing considers fixed manufacturing
cost as a product cost.

Answer 1
Correct answer is A
Variable and absorption costing methods account
for fixed manufacturing overhead in different ways.
Under variable costing, fixed manufacturing
overhead is expensed immediately as a period cost
whereas under absorption costing, such amounts
are treated as a product cost and added to the cost
of inventory. Absorption costing is used for
external reporting purposes. Variable cost is used
for internal decision making purposes.

Question 2

Answer 2

The difference between product and period costs is:

Correct answer is A

A Product costs are cost that case be assoicated to


production and period cost are recognized in the period
incurred.
B Period costs are cost that case be assoicated to
production and product cost are recognized in the period
incurred.
C Period costs and product cost are assoicated to
production.
D Product costs and period cost are assoicated to
production.

Answer A is correct because Product costs are


cost that case be assoicated to production and
period cost are recognized in the period incurred.
B, C, and D are incorrect by definition.

Question 3

Answer 3

For a manufacturing company, which of the following is an


example of a period rather than a product cost?

Correct answer is D

A Depreciation on machines
B Salaries for Direct Labor
C Insurance for factory equipment
D Salaries for sales force

Answer D is correct, salaries for sales force are


period cost. Answers A, B, and C are product cost.

Business Environment & Concepts > Planning, Control and Analysis > Variable and Absorption (full) costing

Question 4
A company begins producing widgets this year and
manufacturers 30,000 and sells 25,000. Direct material cost
$130,000 and direct labor cost $90,000. Factory overhead was
$140,000: $80,000 variable and $60,000 fixed. Selling and
administrative expenses were $200,000: $90,000 variable and
$110,000 fixed. For internal reporting purposes, the company
uses variable (direct) costing. What is the ending amount of
finished goods?
A $48,000
B $50,000
C $60,000
D $65,000

Question 5
What is a scattergraph?
A A method for forecasting product demand on an
X/Y axis.
B A graph to help compute the relationship between
two variables.
C A chart designed to pinpoint unexpected changes
as a result of variable costing.
D A graphed picture created to pick up negative
trends in sales figures over time.

Answer 4
Correct answer is B
Direct (variable) costing only considers variable
manufacturing cost in determining the cost of
inventory. To produce 30,000 units, the variable
manufacturing costs were $300,000 (direct material
of $130,000, direct labor of $90,000, and variable
factory overhead of $80,000). That is $10 per unit.
The company produced 30,000 units but only sold
25,000 so 5,000 units were left. At $10 each, that
is a total cost (according to variable costing) of
$50,000.

Answer 5
Correct answer is B

Companies, especially manufacturing companies,


often try to determine the relationship between two
variables. For example, what is the relationship
between the unemployment rate and company
sales? How does a rise in temperature impact the
number of units produced by a particular plant?
Over time, the two variables can be plotted on a
graph. When enough observations have been
made, a straight-line is drawn through those points
and used to indicate the perceived relationship of
the variables. It is a method that lacks precision
but can help management get a feel for the
relationship.

Business Environment & Concepts > Planning, Control and Analysis > Financial Planning

Financial Planning
Question 1

Answer 1

Mary and Joe are doing financial planning for their small
business, M&J Outfitters. Mary says to Joe, the financial
planning process consist of the following. Mary was right about
the financial planning process except for:

Correct answer is C

A analyzing the investment and financing


alternatives available to a firm.
B forecasting the future consequences of the
alternatives.
C measuring subsequent performance against
established goals is not necessary.
D deciding which alternatives to undertake

Financial Planning process is the process of: 1)


analyzing the investment and financing alternatives
available to a firm, 2) forecasting the future
consequences for the alternatives, 3) deciding
which alternatives to undertake, 4) measuring
subsequent performance against established goals.

Business Environment & Concepts > Cost Accounting > Job and Process Costing

Job and Process Costing


Question 1

Answer 1

The Work in Process inventory account of a manufacturing firm


shows a balance of $3,000 at the end of an accounting period.
The job cost sheets of two uncompleted jobs show charges of
$500 and $300 for materials, and charges of $400 and $600 for
direct labor. From this information, it appears that the company
is using a predetermined overhead rate, as a percentage of
direct labor costs, of:

Correct answer is B
The answer is 120%. Based on the solutions
approach we subtract $1,800 (Direct materials =
$800+ Direct labor = $1,000) from the WIP
balance of $3,000. That gives a remainder of
$1,200 of overhead. $1,200/$1,000 equals the
percentage of direct labor costs or 120%.

A 83%.
B 120%.
C 40%.
D 300%.

Question 2

Answer 2

In computing its predetermined overhead rate, Minor Company


included its factory insurance cost twice. This error will result in:

Correct answer is C

A the ending balance of Finished Goods to be


understated.
B the credits to the Manufacturing Overhead
account to be understated.
C the Cost of Goods Manufactured to be overstated.
D the Net Operating Income to be overstated.

Answer A is incorrect because the ending balance


of Finished Goods would be overstated not
understated. Answer D is incorrect because it
would cause the Net Operating Income to be
understand because of the additional Insurance
Expense. Thus answer C is the only correct
answer. Finish goods inventory would be
overstated transferring to Cost of Goods
Manufactured to be overstated.

Question 3

Answer 3

Job Order Costing is a costing system that distributes cost:

Correct answer is D

A to customer orders based on Direct Material,


Direct Labor and Overhead consumed by this specific
job. Usually these jobs are homogeneous.
B of direct labor and direct materials to product and
services based on JIT methodologies.
C to COGS to a complex series of algorithms.
D to customer orders based on Direct Material,
Direct Labor and Overhead consumed by this specific
job. Usually these jobs are considered unique.

Job order costing is a system for allocating costs to


groups of unique products made to customer
specifications. Each job is material in natural and
accounted for separately. Answer A is incorrect
because job order costing accounts for specific
jobs not homogeneous.

Business Environment & Concepts > Cost Accounting > Job and Process Costing

Question 4

Answer 4

The Happy Company buys dough from an independent


company and uses it to produce both cookies and fudge
squares. Because the cookies and the fudge squares have
approximately the same weight and can be sold for relatively
equal prices, the cost of the dough is allocated between them
based on the quantities produced. However, some of the
dough is damaged slightly during mixing and must be used to
produce cakes. Because cake production is relatively minor for
this company, it is handled as a by-product and entered into
the companys records at its retail value less $2 per cake for
processing. This year, the company bought 800,000 pounds of
dough for $1.5 million. From this, 2.4 million cookies were
produced and 1.6 million fudge squares. In addition, 100,000
cakes were produced and sold at $5 each. What amount of the
cost of the dough is allocated to the fudge squares?

Correct answer is B

A $450,000
B $480,000
C $510,000
D $540,000

The value of the by-product must be removed first


before the remaining joint costs can be assigned.
For accounting purposes, the cakes are valued at
$300,000. That is 100,000 cakes times $3 (the
retail price of $5 less the standard $2 processing
charge). This reduction leaves $1.2 million to be
allocated between cookies and fudge squares.
Companies can make this assignment in any
logical fashion and several techniques are
frequently encountered. Here, they have chosen to
use the quantity produced. There were 1.6 million
fudge squares this year which made up 40 percent
of the 4.0 million total for both cookies and fudge
squares (2.4 million plus 1.6 million). Cookie
production made up the other 60 percent. The joint
cost of $1.2 million is assigned 60 percent
($720,000) to cookies and 40 percent ($480,000)
to fudge squares.

Question 5

Answer 5

An example of where job order costing is not appropriate is:

Correct answer is C

A Manufacture of custom made cabinets


B Providing medical services to a patient
C Manufacturing plastic tubs
D Building a bridge

Job Order costing is a system to allocate costs to


specific, unique or individual products and
services. Process costing is a system to allocate
costs to similar items.

Business Environment & Concepts > Cost Accounting > Job and Process Costing

Question 6

Answer 6

Littlepup Company produces toy dogs that are sold throughout


the United States. They are costing using process cost
accounting. At the beginning of December, Year One, Littlepup
has 30,000 toys in process that are 80 percent complete as to
material but only 30 percent complete as to conversion costs
(direct labor and factory overhead). During December, 100,000
more toy dogs are stated with a total of 120,000 being
completed prior to the end of the year. The remaining 10,000
units are 60 percent complete as to material and 10 percent
complete as to conversion costs. The companys accountants
have computed the equivalent units of work for the month
using both a weighted-average system and a first-in, first-out
system. How many more units will be used in the
weighted-average system?

Correct answer is A

A Material24,000 units more; conversion


costs9,000 units more
B Material18,000 units more; conversion
costs8,000 units more
C Material6,000 units more; conversion
costs21,000 units more
D Material6,000 units more; conversion
costs1,000 units more

Question 7
A company produces bottles of orange juice. This year the
company spent $30,000 to produce 21,000 bottles of orange
juice. However, 1,000 bottles of juice were lost when a valve
on a machine broke. So, only 20,000 bottles of juice were
actually shipped to the stores. Of that amount, 18,000 bottles
were sold and 2,000 bottles remain on the shelf. The
company believes that breakage and loss of up to 10 percent of
the final output is normal. Any breakage and loss of above 10
percent is viewed as abnormal. What is the cost of the
companys ending inventory?
A $2,857
B $2,943
C $2,988
D $3,000

A weighted average system determines the


equivalent units of work done based on the total
that has been accomplished by the end of the
period. During December, 120,000 units were
completed (120,000 units of material and 120,000
units of conversion costs). In addition, 10,000 units
were partially complete (10,000 times 60 percent
or 6,000 units of material and 10,000 times 10
percent or 1,000 units of conversion costs). So, the
total in a weighted-average system is 126,000
units of material (120,000 plus 6,000) and 121,000
units of conversion costs (120,000 plus 1,000). A
first-in, first-out approach makes one subtle
distinction. It does not include any work done prior
to December. Therefore, the amount of work on
the beginning work in process is removed from the
numbers computed above to leave only the work
done during December. For material, 24,000 units
were already in work-in-process at the start of the
month (30,000 units 80 percent complete) so total
equivalent units for December is 102,000 (126,000
total less 24,000 units of work in previous month).
For conversion costs, 9,000 units were already in
beginning work-in-process (30,000 units 30
percent complete) so total equivalent units for
December is 112,000 (121,000 total less 9,000 in
previous month). It is the handling of the beginning
work in process (24,000 and 9,000 equivalent
units) that makes the difference.

Answer 7
Correct answer is D
The company only lost 5 percent of its output
(1,000 bottles versus 20,000 bottles as final
output). That is within the normal range and is
viewed as a cost of producing the good units. The
cost of lost units is only separated and put into a
loss account if it is viewed as abnormal. Here,
20,000 units cost $30,000 to produce which is
$1.50 per bottle. The company still has 2,000
bottles left. At $1.50 per bottle, that is an ending
inventory of $3,000.

Business Environment & Concepts > Cost Accounting > Job and Process Costing

Question 8
A company spends $200,000 and produces 10,000 pounds of
product X and 20,000 pounds of product Y. The joint cost will
be allocated between these two based on units produced. The
company also produced 1,000 pounds of product Z that can be
sold for $6 per pound after spending $1 per pound to put into
appropriate packages. Product Z is viewed as a by-product
and recorded at net realizable value. What portion of the joint
cost is allocated to product X?
A $60,000
B $64,250
C $65,000
D $66,667

Answer 8
Correct answer is C
The by-product must be handled first. This
company records it at net realizable valuethe net
amount of cash that can be received. Each pound
is sold for $6 after spending $1 so the net
realizable value is $5 per pound or $5,000 for the
entire 1,000 pounds. That part of the $200,000 is
recorded for the by-product. That leaves $195,000
to be split between product X and product Y.
According to the information, the company
allocates this joint cost based on units. One-third
of the units (10,000 pounds out of a total of 30,000
pounds for the two major products) comes from
product X. So, 1/3 of the joint cost or $65,000 (1/3
times $195,000) is assigned to product X.

Business Environment & Concepts > Cost Accounting > Basic Terminology

Basic Terminology
Question 1

Answer 1

A planning and control system that measures the cost of


significant activities, identifies non value-added cost, and
identifies activities:

Correct answer is C

A Activity Based Costing


B Job Costing
C Cost Management System
D Process Costing

A Cost Management system is, by definition, a


planning and control system that measures the
cost of significant activities, identifies non
value-added cost, and identifies activities.

Business Environment & Concepts > Cost Accounting > Cost Behavior

Cost Behavior
Question 1

Answer 1

The difference between mixed costs and variable costs is

Correct answer is B

A There is no difference
B Mixed costs contain a component that does not
vary with the level of activity and a component that does
while variable costs only have one component that
varies with the level of activity
C Mixed costs contain a component that does not
vary with the level of activity and a component that does
while variable costs only have one component that does
not vary with the level of activity
D Mixed costs contain budget and actual costs while
variable costs are based on the budget only

Mixed costs or semi variable costs have both a


fixed and variable component. A fixed cost does
not vary with the level of activity while a variable
cost does vary proportionally in total with the level
of activity

Question 2

Answer 2

The definition of the relevant range is

Correct answer is B

A The level of activity where the organization is


profitable
B The level of activity where fixed costs remain fixed
C The level of activity where variable costs continue
to vary
D The level of activity where all costs are mixed

Relevant range is the level of activity where fixed


costs remain fixed (e.g. do not vary)

Question 3

Answer 3

A cost that is not described by a straight line over the relevant


range is the definition of

Correct answer is A

A A nonlinear cost
B A fixed cost
C A variable cost
D A joint cost

(A) A nonlinear cost function is a cost that is not


described by a straight line over the relevant range.
A fixed cost does not vary with the level of activity
while a variable cost does vary proportionally in
total with the level of activity but both can be
represented by a straight line. Joint costs can be
fixed or variable and are costs that are common to
multiple products

Business Environment & Concepts > Cost Accounting > Cost of Goods Manufactured

Cost of Goods Manufactured


Question 1

Answer 1

Which pf the following costs is not an example of factory


overhead that might be found on a cost of goods manufactured
statement

Correct answer is D

A Supplies
B Indirect Labor
C Depreciation
D Materials available for use

(D) Materials available for use is listed on a cost of


goods manufactured statement as a Direct
Materials

Question 2

Answer 2

Information from the Cost of Goods manufactured statement


flows through to which of the following statements

Correct answer is A

A Income Statement
B Balance Sheet
C Cash Flow Statement
D Statement of retained Earnings

(A) The results of the cost of goods manufactured


statement flow through to the cost of goods sold
section on the Income Statement

Question 3

Answer 3

The order in which costs flow through a system is

Correct answer is D

A Materials Inventory, Finished Goods, Work in


Process, Cost of Goods Sold
B Work in Process, Finished Goods, Materials
Inventory, and Cost of Goods Sold
C Cost of Goods Sold, Materials Inventory, Work in
Process, and Finished Goods
D Materials Inventory, Work in Process, Finished
Goods and Cost of Goods Sold

In manufacturing, costs flow through the Materials


Inventory, Work in Process, Finished Goods on
the balance sheet and then to Cost of Goods Sold
on the Income Statement. The transfer from Work
in Process to Finished Goods is the Cost of Goods
Manufactured.

Business Environment & Concepts > Cost Accounting > Cost of Goods Manufactured

Question 4

Answer 4

A company started production on 110 widgets. Due to an


abnormal loss, 8 of the widgets had to be thrown away after
they were finished. Another 12 of the widgets were not
complete at the end of the year. These units were 100 percent
complete as to direct materials but only 50 percent complete as
to conversion costs. The company spent $2,200 on direct
materials, $832 on direct labor, and $520 on factory overhead.
What amount of cost was moved into finished goods for the
year?

Correct answer is A

A $2,970
B $3,025
C $3,118
D $3,234

The company had 110 units: 8 were lost, 12 were


not completed, so the remaining 90 must have
been completed. Because the loss of 8 units was
viewed as abnormal, their cost is computed and
then moved into a loss account on the income
statement. The number of units of direct material is
110 (90 completed, 8 lost that were completed, and
12 in-process that were completed as to direct
material). The direct material cost per unit was
$2,200/110 units or $20 each. The number of units
of conversion cost (direct labor and factory
overhead) is 104 (90 completed, 8 lost that were
completed, and 12 that were 50 percent complete
as to conversion costs). The conversion cost per
unit is $1,352/104 units or $13 each. Because 90
units were completed (and not lost through an
abnormal loss), cost transferred from
work-in-process to finished goods is 90 times $33
($20 plus $13) or $2,970. Notice that the decision
about using FIFO or weighted average is not
addressed because there is no beginning work in
process.

Business Environment & Concepts > Cost Accounting > Overhead

Overhead
Question 1

Answer 1

Which of the following statements regarding manufacturing


overhead is true?

Correct answer is C

A Accounting for manufacturing overhead is not


important as it is not a component in product costing
B Manufacturing overhead is tied directly to the final
product
C Manufacturing overhead can not be traced directly
to the final product
D Manufacturing overhead consists of direct labor
and materials

(C) Overhead consists of all costs other than direct


labor and direct materials and can not be traced
directly to the product

Question 2

Answer 2

Which of the following are examples of activity bases that can


be used to apply manufacturing overhead to the final products?

Correct answer is A

A Labor hours, labor costs and machine hours


B Total production hours
C Total revenue from items produced
D Total number of items produced

(A) The activity base should have a causal


relationship to the incurrance of the overhead
costs. Labor hours, labor costs and machine hours
are all examples of activity bases.

Question 3

Answer 3

Which of the following definitions describes the concept of


Theoretical capacity as it is related to the application of
overhead?

Correct answer is A

A Output is produced efficiently 100% of the time


B Output produced 100% of the time less the output
lost due to non-production time
C Long run product demand over a multiple year
period
D Expected output for the current year only.

(A) Theoretical capacity - Output is produced 100%


of the time; Practical capacity adjusts Theoretical
capacity for non-production time such as holidays
and maintenance shutdowns; Normal volume
adjusts Theoretical capacity for long run product
demand over a multiple year period and Expected
annual capacity adjusts Theoretical capacity for the
expected output for the current year only.

Question 4

Answer 4

Which of the following definitions describes the concept of


Practical capacity as it is related to the application of overhead?

Correct answer is B

A Output is produced efficiently 100% of the time


B Output produced 100% of the time less the output
lost due to non-production time
C Long run product demand over a multiple year
period
D Expected output for the current year only.

(A) Theoretical capacity - Output is produced 100%


of the time; Practical capacity adjusts Theoretical
capacity for non-production time such as holidays
and maintenance shutdowns; Normal volume
adjusts Theoretical capacity for long run product
demand over a multiple year period and Expected
annual capacity adjusts Theoretical capacity for the
expected output for the current year only.

Business Environment & Concepts > Cost Accounting > Overhead

Question 5

Answer 5

Which of the following definitions describes the concept of


Expected Annual capacity as it is related to the application of
overhead?

Correct answer is D

A Output is produced efficiently 100% of the time


B Output produced 100% of the time less the output
lost due to non-production time
C Long run product demand over a multiple year
period
D Expected output for the current year only.

(A) Theoretical capacity - Output is produced 100%


of the time; Practical capacity adjusts Theoretical
capacity for non-production time such as holidays
and maintenance shutdowns; Normal volume
adjusts Theoretical capacity for long run product
demand over a multiple year period and Expected
annual capacity adjusts Theoretical capacity for the
expected output for the current year only.

Question 6

Answer 6

Which of the following definitions describes the concept of


Normal Volume as it is related to the application of overhead?

Correct answer is C

A Output is produced efficiently 100% of the time


B Output produced 100% of the time less the output
lost due to non-production time
C Long run product demand over a multiple year
period
D Expected output for the current year only.

(A) Theoretical capacity - Output is produced 100%


of the time; Practical capacity adjusts Theoretical
capacity for non-production time such as holidays
and maintenance shutdowns; Normal volume
adjusts Theoretical capacity for long run product
demand over a multiple year period and Expected
annual capacity adjusts Theoretical capacity for the
expected output for the current year only.

Question 7

Answer 7

The most common approach in use to the application of


overhead is

Correct answer is D

A Theoretical capacity
B Practical capacity
C Regionally adjusted capacity
D Expected annual capacity

Most firms use the Expected Annual Capacity


approach to the application of overhead. Expected
Annual Capacity adjusts the output that could be
produced if the operation ran efficiently 100% of
the time for the expected output for the current
year.

Question 8

Answer 8

Utilizing the Expected Annual capacity approach to overhead


application, can result in over applied overhead in which of the
following situations. (Over applied overhead occurs when more
overhead is applied than was incurred)

Correct answer is D

A Actual overhead costs were more than expected


and actual production was less than expected.
B Estimated overhead costs were less than actual
and estimated production was equal to actual.
C Actual overhead costs were equal to estimated
overhead costs and estimated production was equal to
actual.
D Actual overhead costs were less than expected
and/or production was higher than expected.

(D) Overhead is applied based on a calculated rate


per unit. This calculated rates uses estimate
overhead costs divided by an estimated activity
level. If either the estimated overhead is higher
than the actual overhead or the estimated activity
level is lower than the actual activity level,
overhead can be overapplied.

Business Environment & Concepts > Cost Accounting > Standard Cost Accounting

Standard Cost Accounting


Question 1

Answer 1

It takes 10 pounds of plastic to make a widget under normal


conditions and plastic normally costs $2 per pound. This year,
a company buys 100,000 pounds of plastic at $2.20 per pound
and uses 73,000 pounds to make 7,000 widgets. In a standard
cost accounting system, what is the material price variance?

Correct answer is D

A $14,000 unfavorable
B $14,600 unfavorable
C $15,400 unfavorable
D $20,000 unfavorable

In a standard cost accounting system, a company


determines the material price variance when
materials are first bought (other variances are
during production). Here, the company bought
100,000 pounds of plastic and paid $.20 more than
the standard price for each pound. That is an
unfavorable price variance of $20,000 (100,000
pounds times $.20).

Question 2

Answer 2

It takes 10 pounds of plastic to make a widget under normal


conditions and plastic has a standard cost of $2 per pound.
This year, a company buys 100,000 pounds of plastic at $2.20
per pound and uses 73,000 pounds to make 7,000 widgets. In
a standard cost accounting system, what is the material usage
variance?

Correct answer is A

A $6,000 unfavorable
B $6,600 unfavorable
C $60,000 unfavorable
D $66,000 unfavorable

The company produced 7,000 widgets which


should have taken 10 pounds of plastic each or
70,000 pounds in total. They actually used 73,000
pounds so the company used 3,000 pounds more
than the standard. Usage or quantity variances are
measured at standard price. The 3,000 extra
pounds means that the material usage variance is
$6,000 (3,000 pounds at the standard price of $2
per pound).

Question 3

Answer 3

A company is producing wagons. The company expects to


produce 10,000 wagons this year and take three hours of direct
labor to complete each one. The direct labor is supposed to
cost $15 per hour. The company actually works 31,000 hours
but only produces 9,800 wagons. The labor cost is $496,000 or
$16 per hour. In a standard cost accounting system, what is
the labor rate variance?

Correct answer is D

A $29,400 unfavorable
B $30,000 unfavorable
C $30,600 unfavorable
D $31,000 unfavorable

Company employees were paid a rate $1 per hour


over the standard ($16 per hour less $15 per
hour). The company employees worked a total of
31,000 hours. For each of those hours, the
employee was paid $1 over standard. The labor
rate variance is unfavorable (company paid more
than standard) and equals $31,000 ($1 per hour for
31,000 hours).

Business Environment & Concepts > Cost Accounting > Standard Cost Accounting

Question 4

Answer 4

A company is producing wagons to sell. The company expects


to produce 10,000 wagons this year and take three hours of
direct labor to complete each one. The direct labor is supposed
to cost $15 per hour. The company actually works 31,000
hours but only produces 9,800 wagons. The labor cost is
$496,000 or $16 per hour. In a standard cost accounting
system, what is the labor efficiency variance?

Correct answer is C

A $15,000 unfavorable
B $16,000 unfavorable
C $24,000 unfavorable
D $25,600 unfavorable

The company was supposed to take three hours to


produce a wagon. They produced 9,800 wagons so
that should have taken 29,400 hours (9,800 times 3
hours). They actually worked 31,000 hours so they
took 1,600 hours too much (31,000 less 29,400).
They were supposed to pay their workers $15 per
hour so the labor efficiency variance is $24,000
(1,600 hours at $15 per hour). It is unfavorable
because they took longer than they were
supposed to take to produce this quantity.

Question 5

Answer 5

A company is producing wagons to sell. The company expects


to produce 10,000 wagons this year and it should take 12 feet
of lumber to produce each one. This lumber should cost $.70
per foot. The company actually buys 130,000 feet of lumber for
$94,000. The company then uses 120,000 feet of lumber to
produce 9,800 wagons. In a standard cost accounting system,
what is the material price variance?

Correct answer is B

A $2,200 unfavorable
B $3,000 unfavorable
C $3,600 unfavorable
D $4,000 unfavorable

The company should have paid $91,000 for the


lumber that it bought (130,000 feet at $.70 per
foot). It actually paid $94,000 which was $3,000
more than the standard price.

Business Environment & Concepts > Partnerships > New partner

New partner
Question 1

Answer 1

Gent, Hill and Inge were partners in GHI Enterprises, a general


partnership. After three years of operation the partnership
found itself in need of technical expertise and voted to admit
Jade, who possessed such expertise, into the partnership. As
a result of Jade's admission into GHI Enterprises,

Correct answer is C

A Jade has personal liability for all existing and new


partnership debts.
B Jade will have no liability for any partnership
debts.
C If Jade contributes $40,000 to be admitted to the
partnership, these funds can be subject to preexisting
partnership obligations.
D If Hill files personal bankruptcy, the partnership
will be placed in bankruptcy as well.

A newly admitted partner is liable for pre-existing


partnership debts only to the extent of funds
contributed by the new partner. In addition, the
personal bankruptcy of one partner does not result
in the bankruptcy of the partnership.

Business Environment & Concepts > Partnerships > Assignment

Assignment
Question 1

Answer 1

Isis, Julia and K.D. are partners in a general partnership which


grows roses for shipment around the world. Due to financial
difficulties unrelated to the business, K.D. assigned her interest
in the partnership to Grande Bank. As a result of this
assignment,

Correct answer is B

A Grande Bank is entitled to examine the books and


records of the partnership.
B Grande Bank is now the owner of K.D.'s interest
in the partnership profits and surplus.
C Grande Bank is a partner in the partnership, but
without liability for partnership debts.
D The partnership is dissolved unless Isis and Julia
vote to continue the partnership.

An assignee of a partnership interest is entitled to


the assignor's profits and surplus but does not
become a substitute partner, and therefore has no
right to manage the partnership or to examine its
books and records.

Question 2

Answer 2

Claremont sold a parcel of real estate to Fulton for $500,000


with $100,000 paid in cash, and the balance to be paid over
time. The instrument in which Fulton promised payment did not
qualify as a negotiable instrument. Nevertheless, the
instrument was assigned to Grace without Fulton's consent.
Which is correct?

Correct answer is D

A An assignment is ineffective unless the obligor


consents.
B Fulton is bound by the assignment immediately.
C Since the instrument is not negotiable, it cannot
be transferred.
D If Fulton, after receiving notice of the assignment,
pays the full balance to Claremont, Fulton will remain
liable to Grace.

An assignment is effective immediately, but is only


binding on the obligor when the obligor has been
given notice. Thus, if Fulton, after receiving notice
of the assignment, pays the full balance to
Claremont, Fulton will remain liable to Grace. An
instrument is assignable even though it does not
qualify as a negotiable instrument.

Business Environment & Concepts > Partnerships > Death of partner

Death of partner
Question 1

Answer 1

Cobb, Heft and Farrel are partners in a wholesale plumbing


supply business. The partnership has been in business for
twenty-five years, owns inventory and fixtures and a
warehouse. Its only debt is a mortgage on the warehouse
building. If Cobb dies

Correct answer is C

A Cobb's heirs would become partners in the


business.
B Cobb's estate would have no liability with respect
to the warehouse mortgage obligation.
C the plumbing supply partnership is NOT
automatically dissolved.
D Cobb's heirs will be entitled to vote on the
admission of a new partner.

Under the Revised Uniform Partnership Act, the


death of a partner not longer causes the
partnership to automatically dissolve. The
deceased partner's heirs do not become partners
in the partnership, but are required to buy out the
remaining partners.

Business Environment & Concepts > Partnerships > Profits and Losses

Profits and Losses


Question 1

Answer 1

Flemming, Drew and Flemming, a general partnership,


operated a surgical medical practice under an agreement
whereby Gabe and Emma Flemming, who worked only part
time, would each be paid 1/4 of the profits, and Drew, who
worked full time, would receive 1/2 the profits. The Flemmings
each contributed $500,000 to get the partnership under way,
while Drew contributed no money or property. After one year in
business, the partnership had losses of $90,000. What amount
of this loss should be allocated to Drew?

Correct answer is C

A $0
B $30,000
C $45,000
D $90,000

If partners in a partnership have no agreement as


to the sharing of losses, losses will be shared in
the same manner as profits.

Business Environment & Concepts > Partnerships > Negligence of partner

Negligence of partner
Question 1

Answer 1

Able, Weiss and Peters operate as partners a pet grooming


business. Weiss inadvertently gave one of the dogs he was
grooming a treat containing an ingredient to which the dog was
highly allergic, and which resulted in the dog's death. The dog's
owner filed suit against the partnership and was able to
establish that she had warned Able, who had taken the dog in
that morning, of the dog's allergy. Under these circumstances

Correct answer is C

A neither Weiss nor the partnership would be liable


because no loss was suffered.
B the partnership will not be liable because notice
must be given to all partners.
C Weiss can be held personally liable for the loss of
the dog.
D the partnership can avoid liability if it can prove
that only Weiss was negligent.

When a partner in a partnership commits


negligence in the course of partnership business,
the partners and the partnership are jointly and
severally liable for injuries or damages proximately
caused by the negligent conduct.

Business Environment & Concepts > Partnerships > LLLP1

LLLP1
Question 1

Answer 1

In jurisdictions which permit a limited partnership to become a


limited liability limited partnership

Correct answer is A

A the general partner of the limited partnership will


enjoy limited liability.
B the limited partners will have authority to act for
the partnership.
C the general partner is removed from the
partnership.
D the new entity is permitted to make profits from
passive investments only.

In the few jurisdictions which permit a limited


partnership to become a limited liability limited
partnership, the general partner of the limited
partnership has limited liability.

Business Environment & Concepts > Partnerships > Limited partnership

Limited partnership
Question 1

Answer 1

Katzen, King and Cranz are partners in Fine China Importers,


L.P., a limited partnership in which Cranz is designated as the
general partner and Katzen and King are limited partners.
Under the provisions of the Uniform Limited partnership Act
(2001): I. King has the authority to bind the partnership to a
contract to hire a part-time temporary employee; II. Katzen will
be personally obligated if Fine China Importers, L.P., defaults
on one of its loan obligations. III. King has a fiduciary duty to
the limited partnership by reason of being a limited partner.

Correct answer is D

A I and III are true.


B II and III are true.
C III only is true.
D None of the above are true.

Under Section 302 of the Uniform Limited


partnership Act (2001), a limited partner does not
have the right or the power as a limited partner to
act for or bind the limited partnership. Under
Section 303, an obligation of a limited partnership,
whether arising in contract, tort, or otherwise, is not
the obligation of a limited partner. A limited partner
is not personally liable, directly or indirectly, by way
of contribution or otherwise, for an obligation of the
limited partnership solely by reason of being a
limited partner. And, under Section 305, a limited
partner does not have any fiduciary duty to the
limited partnership or to any other partner solely by
reason of being a limited partner.

Question 2

Answer 2

Green, Able and Holmes wish to form Able, L.P., and operate
as a limited partnership. Under the provisions of the Uniform
Limited partnership Act (2001), which of the following would
NOT be required of Able, L.P.?

Correct answer is B

A Designation of an agent for service of process.


B Removal of Able's name from the partnership
name if Able is a limited partner.
C Delivery to the state of current annual reports.
D Deliver to local filing offices of a certificate stating
the name of the limited partnership, the name and the
street and mailing address of each general partner, and,
whether the limited partnership is a limited liability
limited partnership.

As with a corporation, the formation of a limited


partnership requires compliance with certain
statutory formalities, including designating and
maintaining an agent for service of process, annual
delivery to the state current reports, and delivering
to local filing offices of a certificate stating the
name of the limited partnership, the name and the
street and mailing address of each general partner,
including whether the limited partnership is a
limited liability limited partnership. The most recent
version of the Uniform Limited partnership Act
(2001) permits the use of a limited partner's name
in the partnership name.

Question 3

Answer 3

Under the provisions of the Uniform Limited Partnership Act


(2001),

Correct answer is C

A the name of a limited partnership may NOT


contain the name of any partner.
B the name of a limited partnership may contain the
phrase 'limited partnership,' the abbreviation L.P.
or the abbreviation LLC.
C the name of a limited liability limited partnership
must contain the phrase limited liability limited
partnership or the abbreviation LLLP or L.L.L.P.
and must not contain the abbreviation L.P. or LP.
D the name of a limited partnership may use the
designation Corp. or Corporation in its name
instead of L.P.

The name of a limited partnership may contain the


name of any partner. The name of a limited
partnership that is not a limited liability limited
partnership must contain the phrase limited
partnership or the abbreviation L.P. or LP
and may not contain the phrase limited liability
limited partnership or the abbreviation LLLP
or L.L.L.P.. The name of a limited liability
limited partnership must contain the phrase
limited liability limited partnership or the
abbreviation LLLP or L.L.L.P. and must not
contain the abbreviation L.P. or LP. Neither
type of entity may substitute the designation of
Corporation.

Business Environment & Concepts > Partnerships > Limited partnership

Question 4

Answer 4

A limited partnership must maintain at its designated office


which of the following information: I. A current list showing the
full name and last known street and mailing address of each
partner, separately identifying the general partners and the
limited partners; II. A copy of the initial certificate of limited
partnership and all amendments to and restatements of the
certificate; III. A copy of the limited partnership's federal, state,
and local income tax returns and reports, if any, for the three
most recent years.

Correct answer is D

A I only
B II only
C I and II only
D I, II and III

Section 111 of the Limited Partnership Act (2001),


which addresses "Required Information," provides:
A limited partnership shall maintain at its
designated office the following information: (1) a
current list showing the full name and last known
street and mailing address of each partner,
separately identifying the general partners, in
alphabetical order, and the limited partners, in
alphabetical order; (2) a copy of the initial
certificate of limited partnership and all
amendments to and restatements of the certificate,
together with signed copies of any powers of
attorney under which any certificate, amendment,
or restatement has been signed; (3) a copy of any
filed articles of conversion or merger; (4) a copy of
the limited partnership's federal, state, and local
income tax returns and reports, if any, for the three
most recent years; (5) a copy of any partnership
agreement made in a record and any amendment
made in a record to any partnership agreement.

Question 5

Answer 5

Under the provisions of the Uniform Limited Partnership Act


(2001), which of the following characteristics are NOT shared
by a corporation and a limited partnership?

Correct answer is D

A Both may be subject to the requirements of federal


securities laws.
B Both limited partnerships and corporations have
unlimited life.
C Limited partners enjoy limited liability similar to
officers in a corporation.
D Both issue stock as evidence of ownership.

A limited partnership is an entity distinct from its


partners. A limited partnership has a perpetual
duration which is a change from previous law. Both
are subject to federal securities laws, but only a
corporation issues stock.

Question 6

Answer 6

Under the Uniform Limited Partnership Act: I. A limited


partnership is an entity distinct from its partners. II. A limited
partnership has a perpetual duration.

Correct answer is C

A I only is correct
B II only is correct
C Both I and II are correct.
D Neither I nor II is correct.

Under the Uniform Limited Partnership Act (2001)


a limited partnership is an entity distinct from its
partners. A limited partnership is the same entity
regardless whether its certificate states that the
limited partnership is a limited liability limited
partnership. A limited partnership may be
organized under the ULPA for any lawful purpose,
and, a limited partnership has a perpetual duration.

Business Environment & Concepts > Partnerships > Notice

Notice
Question 1

Answer 1

Under the Uniform Partnership Act, a partner has notice of a


fact if I. notice has been printed in a newspaper of record; II.
the partner has reason to know it exists from all of the facts
known to the person at the time in question.

Correct answer is B

A I only is correct.
B II only is correct.
C Both I and II are correct.
D Neither I nor II is correct.

The Uniform Partnership Act, in Section 102,


states: A person has notice of a fact if the person:
(1) knows of it; (2) has received a notification of it;
or, (3) has reason to know it exists from all of the
facts known to the person at the time in question.

Business Environment & Concepts > Partnerships > Presumption

Presumption
Question 1

Answer 1

Under the Uniform Partnership Act, which is true?

Correct answer is B

A Hansel and Ray own, as tenants in common, an


apartment complex and share the profits therefrom.
Hansel and Ray are presumed to be partners.
B Price and Fields carry on a business together and
share profits. Even though they express, in writing, their
intent not to be treated as a partnership, the law
presumes them to be a partnership.
C Blair, who receives a share of the profits of a
business, is presumed to be a partner in the business
even though the profits were received in payment of
rent.
D Weston, a full time employee of a partnership,
who is paid a percentage of partnership profits, is
presumed to be a partner.

The Uniform Partnership Act provides that an


association of two or more persons who carry on
as co-owners a business for profit will be treated as
a partnership, whether or not the persons intend to
form a partnership. There is no presumption of a
partnership in the following instances: (1) Joint
tenancy, tenancy in common, tenancy by the
entireties, joint property, common property, or part
ownership even if the co-owners share profits
made by the use of the property. (2) A person
receives a share of the profits of a business where
the profits were received in payment: (i) of a debt
by installments or otherwise; (ii) for services as an
independent contractor or of wages or other
compensation to an employee; (iii) of rent.

Question 2

Answer 2

Jen and Barry devised a formula and process for producing


extraordinarily good ice cream. Jen and Barry decided to
market their product, "Vermont's Best Ice Cream," and divide
the profits 60% to Jen, 40% to Barry. Barry and Jen had no
oral or written partnership agreement. Jen and Barry

Correct answer is B

A would divide losses equally, should losses occur


in the future.
B will be deemed a partnership in most legal
respects.
C must enter into a written agreement should the
business last more than one year.
D need not pay Social Security taxes or Self
Employment taxes since they are not incorporated.

Whenever two or more persons carry on as


co-owners a business for profit, they will be
deemed a partnership as their activities meet the
definition of a partnership under state partnership
law. If there is no agreement as to the division of
losses, losses are shared in the same proportion
as profits. Payment of Social Security taxes is not
dependent on the form of the business.

Business Environment & Concepts > Partnerships > Profits

Profits
Question 1

Answer 1

Ben, Skip, Trip and Chad are partners in a retail clothier which
trades as Washington Heights Fashions. Trip works
approximately 80 hours per week, primarily waiting on
customers. Ben works about 20 hours per week stoking
shelves and waiting on customers. Chad's main involvement
is attending trade shows and acting as purchasing agent,
which averages about 20 hours per week. Skip has no
involvement in the day to day business, but financed 100% of
the venture. There is no written agreement as to the division of
profits.

Correct answer is B

A Skip is entitled to interest and principal payments,


but not a share of the profits.
B Trip is entitled to the same share of profits as
Chad.
C Any losses would be divided evenly among Ben,
Trip and Chad only.
D Skip is entitled to 100% of the profits until he is
repaid in full.

In the absence of an agreement among partners


on the division of profits, profits will be divided
equally regardless of which partner has contributed
more capital, more labor or more expertise.

Business Environment & Concepts > Partnerships > Agreement

Agreement
Question 1

Answer 1

A partnership agreement may NOT:

Correct answer is D

A specify a division of losses which varies


significantly from the division of profits.
B identify specific types or categories of activities
that do not violate the duty of loyalty.
C reduce a partner's duty of care.
D eliminate a partners power to dissociate from the
partnership.

Under the (Revised) Uniform Partnership Act, a


partnership agreement may not: unreasonably
restrict the right of access to books and records;
eliminate the duty of loyalty (but, the partnership
agreement may identify specific types or
categories of activities that do not violate the duty
of loyalty, if not unreasonable); unreasonably
reduce the duty of care; eliminate the obligation of
good faith and fair dealing under (but the
partnership agreement may prescribe reasonable
standards by which the performance of the
obligation is to be measured); or, vary the power to
dissociate as a partner under (except to require the
notice to be in writing).

Question 2

Answer 2

Ulman, Voit and Walton formed a general partnership and


consulted you regarding certain matters that they wished to
cover in their written Agreement of Partnership. The general
business of the partnership is rental and sale of office
equipment. Which of the following provisions would violate the
Uniform Partnership Act? A provision that

Correct answer is B

A specifies that Walton's 50% ownership of a


competing business will not be considered breach of
loyalty.
B denies Voit the right of access to books and
records.
C states that any two partners may ratify, after full
disclosure of all material facts, transactions that
otherwise would violate the duty of loyalty.
D requires that a partner may not dissociate without
written notice.

Section 103 of the Uniform Partnership Act


provides that relations among the partners and
between the partners and the partnership are
governed by the partnership agreement. However,
the Act provides that the partnership agreement
may NOT: unreasonably restrict the right of access
to books and records; unreasonably reduce the
duty of care; eliminate the obligation of good faith
and fair dealing, but the partnership agreement
may prescribe the standards by which the
performance of the obligation is to be measured, if
the standards are not manifestly unreasonable;
vary the power to dissociate as a partner, except to
require the notice be in writing; eliminate the duty
of loyalty but: (i) the partnership agreement may
identify specific types or categories of activities that
do not violate the duty of loyalty, if not manifestly
unreasonable; or (ii) all of the partners or a number
or percentage specified in the partnership
agreement may authorize or ratify, after full
disclosure of all material facts, a specific act or
transaction that otherwise would violate the duty of
loyalty.

Business Environment & Concepts > Partnerships > Agreement

Question 3

Answer 3

The primary focus of the (Revised) Uniform Partnership Act is


the small, often informal, partnership. Larger partnerships
generally have a partnership agreement addressing, and often
modifying, many of the provisions of the Act. Which of the
following statements is correct with respect to the overall
treatment found in the (Revised) Uniform Partnership Act? I.
The Revised Act enhances the entity treatment of
partnerships to achieve simplicity for state law purposes,
particularly in matters concerning title to partnership property.
II. The Revised Act is largely a series of "default rules" that
govern the relations among partners in situations they have not
addressed in a partnership agreement.

Correct answer is C

A I only is correct.
B II only is correct.
C Both I and II are correct.
D Neither I nor II is correct.

The Uniform Partnership Act ("Revised Act" or


"RUPA") gives supremacy to the partnership
agreement in almost all situations. The Revised
Act is, therefore, largely a series of "default rules"
that govern the relations among partners in
situations they have not addressed in a partnership
agreement. The primary focus of RUPA is the
small, often informal, partnership. Larger
partnerships generally have a partnership
agreement addressing, and often modifying, many
of the provisions of the partnership act. The
Revised Act enhances the entity treatment of
partnerships to achieve simplicity for state law
purposes, particularly in matters concerning title to
partnership property. RUPA does not, however,
relentlessly apply the entity approach. The
aggregate approach is retained for some purposes,
such as partners' joint and several liability.

Question 4

Answer 4

Heinline starts a business and invites his friend Walsingham to


work with him. Heinline makes the following statement: lets
just share profits 50-50. That is all that is ever said. Which of
the following is true?

Correct answer is A

A This is a partnership unless evidence to the


contrary exists.
B This is not a partnership because the agreement
was not in writing.
C This is not a partnership because the agreement
was incompleteit did not specify the handling of
losses.
D This is not a partnership because Heinline did not
specifically mention the word partner or partnership.

A partnership agreement can be very informal and


does not have to be in writing or any other
specified form. Simply agreeing to split profits is
normally an indication that a partnership has been
formed. If no statement is made as to losses, the
same ratio used for profits is also used for any
subsequent losses.

Business Environment & Concepts > Partnerships > Authority

Authority
Question 1

Answer 1

Bevil Associates, Architects, is general partnership whose


partners include Jefferson, Locke and Payne. Which of the
following actions would NOT bind Bevil Associates?

Correct answer is D

A Execution by Locke of a promissory note for


working capital for the firm.
B Purchase by Jefferson of a computer system
(hardware and software) which has capabilities vastly
exceeding the needs of the business.
C Payne's hiring of a new associate whom Locke
and Jefferson were adamantly against.
D All of the above would bind the partnership.

Each partner in a general partnership is an agent


of the partnership, having the authority to bind the
partnership to contracts entered into in the
ordinary course of business. All of the contracts
illustrated in the question fall into this category.

Business Environment & Concepts > Partnerships > Property

Property
Question 1

Answer 1

Under the (Revised) Uniform Partnership Act, which statement


is not true with respect to partnership property:

Correct answer is A

A Property is not partnership property if it was


acquired in the name of one of the partners and the title
indicates his partnership capacity but fails to state the
partnership name.
B Ultimately, it is the intention of the partners that
controls whether property belongs to the partnership or
to one or more of the partners in their individual
capacities, at least as among the partners themselves.
C Property is presumed to be partnership property if
purchased with partnership assets, even if not acquired
in the name of the partnership.
D Property acquired in the name of a partner,
without use of partnership assets, is presumed to be
separate property, even if used for partnership
purposes.

Section 204 of the Uniform Partnership Act states:


(a) Property is partnership property if acquired in
the name of: (1) the partnership; or (2) one or
more partners with an indication in the instrument
transferring title to the property of the person's
capacity as a partner or of the existence of a
partnership but without an indication of the name
of the partnership. (b) Property is presumed to be
partnership property if purchased with partnership
assets, even if not acquired in the name of the
partnership. (c) Property acquired in the name of
one or more of the partners, without an indication
in the instrument transferring title to the property of
the person's capacity as a partner or of the
existence of a partnership and without use of
partnership assets, is presumed to be separate
property, even if used for partnership purposes.

Business Environment & Concepts > Partnerships > Dissociation, Dissolution

Dissociation, Dissolution
Question 1

Answer 1

Under the (Revised) Uniform Partnership Act, a partnership at


will is dissolved, and its business must be wound up, upon the
occurrence of

Correct answer is C

A the death of a partner.


B the bankruptcy of a partner.
C notice of a partner's intent to withdraw.
D the incarceration of a partner.

A partnership is dissolved, and its business must


be wound up, upon the occurrence of the
partnership's having notice from a partner of that
partner's express will to withdraw immediately as a
partner, or on a later date specified by the partner.
RUPA dramatically changes the law governing
partnership breakups and dissolution. An entirely
new concept, "dissociation," is used in lieu of the
UPA term "dissolution" to denote the change in the
relationship caused by a partner's ceasing to be
associated in the carrying on of the business.
"Dissolution" is retained but with a different
meaning. Under RUPA, unlike the UPA, the
dissociation of a partner does not necessarily
cause a dissolution and winding up of the business
of the partnership. Bankruptcy or death of a
partner results dissociation, but not necessarily
dissolution.

Business Environment & Concepts > Partnerships > Dissolution, Winding up

Dissolution, Winding up
Question 1

Answer 1

After only two years of operation, Brooks Associates, a general


partnership, dissolved with acrimony among the partners and
accusations of fraud and mismanagement. The partners, Loew,
Burns and Chang, were able to agree only that the partnership
should cease operating immediately. Notwithstanding the
dissolution of the partnership,

Correct answer is A

A Chang still has apparent authority to bind the


partnership.
B Burns will have no liability for partnership debts if
Loew and Chang later decide to continue the
partnership business.
C If Loew attempts to continue the partnership
business on his own, he will have actual authority to
bind the partnership.
D Brooks Associates still has authority to sign a
contract for new business.

Upon dissolution of a partnership, partners no


longer have actual express authority to bind the
partnership except with respect to winding up the
business. Each partner will, however, still possess
a residual of apparent authority to bind the
partnership. Exercising such authority to bind the
partnership will be wrongful as among the
partners, but will be binding on the partners and
the partnership as against third parties.

Question 2

Answer 2

Under the (Revised) Uniform partnership Act, all of the


following are correct with regard to dissolution of a partnership
except:

Correct answer is D

A In general, a partnership continues after


dissolution only for the purpose of winding up its
business.
B The partnership is terminated when the winding
up of its business is completed.
C At any time after the dissolution of a partnership
and before the winding up of its business is completed,
the partners may waive the right to have the
partnership's business wound up and the partnership
terminated.
D A filing with the appropriate state authorities is
required for dissolution to occur.

In general, a partnership continues after


dissolution only for the purpose of winding up its
business. The partnership is terminated when the
winding up of its business is completed. At any
time after the dissolution of a partnership and
before the winding up of its business is completed,
all of the partners, including any dissociating
partner other than a wrongfully dissociating
partner, may waive the right to have the
partnership's business wound up and the
partnership terminated. In that event the
partnership resumes carrying on its business as if
dissolution had never occurred, any liability
incurred by the partnership or a partner after the
dissolution and before the waiver is determined as
if dissolution had never occurred. Since no filing or
other formality is required, the date will often be
determined only by hindsight.

Business Environment & Concepts > Partnerships > Dissolution, Winding up

Question 3

Answer 3

Which of the following statements reflects one of the general


principles found in the (Revised) Uniform Partnership Act
(RUPA)?

Correct answer is A

A Dissociation by a partner's death or bankruptcy


results in a buyout of the partner's interest rather than
automatic dissolution of the partnership.
B A partnership may NOT by agreement alter the
events which, under RUPA, cause automatic dissolution.
C RUPA provides no procedures for buyout rights in
a dissolved partnership, as these are left to partnership
agreement only.
D RUPA does away with the separate entity theory
for partnerships.

Question 4
Able, Baker, and Cannon are partners. Able gets 70 percent
of the profits and losses and has a capital balance of $100,000.
Baker gets 20 percent of the profits and losses and has a
capital balance of $60,000. Cannon gets 10 percent of the
profits and losses and has a capital balance of $10,000. The
partnership has $170,000 in assets but no liabilities. The
partnership is being liquidated. Assets of $70,000 are sold for
$40,000 in cash. Which of the partners gets this available
cash?
A Able gets $28,000, Baker gets $8,000, and
Cannon gets $4,000
B Able gets $13,333, Baker gets $13,333, and
Cannon gets $13,334
C Able gets $6,667 and Baker gets $33,333
D Able gets $20,000 and Baker gets $20,000

Dissociation by a partner's death or bankruptcy


results in a buyout of the partner's interest rather
than automatic dissolution of the partnership. This
is a change from prior law. A partnership may by
agreement alter the events which, under RUPA,
cause automatic dissolution. RUPA provides
default procedures for buyout rights in a dissolved
partnership in the event such procedures are not
covered in the partnership agreement. RUPA has
moved toward the separate entity theory for
partnerships, which is also a change from prior law.

Answer 4
Correct answer is C
The actual sale of these assets created a $30,000
loss ($70,000 less $40,000). That loss is assigned
70 percent to Able ($21,000 bringing the capital
down to $79,000), 20 percent to Baker ($6,000
bringing the capital down to $54,000), and 10
percent to Cannon ($3,000 bringing the capital
down to $7,000). To decide what to do with the
available cash, the partnership must assume that
the remaining $100,000 in assets ($170,000 less
$70,000) are a total loss. In that case, this
$100,000 anticipated loss is assigned 70 percent
to Able ($70,000 bringing the capital down to
$9,000), 20 percent to Baker ($20,000 bringing the
capital down to $34,000), and 10 percent to
Cannon ($10,000 bringing the capital down to a
negative $3,000). Then, finally, the remaining
negative $3,000 balance in Cannons capital must
be divided between Able and Baker based on their
relative profit and loss ratios or 70/90 of the $3,000
to Able or $2,333 and 20/90 of the $3,000 to Baker
or $667. This final allocation reduces Ables
capital from $9,000 to $6,667 and Bakers capital
from $34,000 to $33,333. That is how the
available cash is distributed.

Business Environment & Concepts > Partnerships > Liability of partners

Liability of partners
Question 1

Answer 1

Hoffman, Golden and Stiles ware partners in a venture which


manages assisted living facilities. Hoffman, Golden and Stiles
agreed that, should the venture become profitable, profits
would be divided 1/2 to Stiles and 1/4 to each of the remaining
partners. No agreement was made with respect to losses. The
partnership was profitable for several years, then losses
started mounting. At the time the partnership was dissolved,
losses of had accumulated, and no cash remained for the
payment of various debts. In a suit by one of the partnership's
employees for unpaid wages of $24,000, in which all
necessary parties were names as defendants

Correct answer is C

A Hoffman would personally be liable for not more


than $8,000.
B Stiles's personal liability would be limited to
$12,000.
C Golden could be held liable for $24,000.
D if non-cash assets were available from the
partnership, none of the partners would be personally
liable.

In a general partnership, partners are jointly and


severally liable, along with the partnership, for
partnership obligations. As among the partners,
Stiles would be liable for $12,000 and Hoffman and
Golden for $6,000 each. But as against the third
party seeking recovery, any partner could be held
liable for 100% of the claim.

Business Environment & Concepts > Partnerships > Estoppel

Estoppel
Question 1

Answer 1

At a dinner party to honor Blade for his general philanthropy,


Wilson represented to several of the guests, in Blades
presence, that a new real estate development venture -- Jett
Landing -- will be a certain success with my friend Blade,
here, as one of our partners. Blade, who had heard such
representations before, smiled and said nothing even though
he had no involvement in the Jett Landing development. Bragg
Banks president, who had been present when Wilson made
the statement, authorized a significant loan to the developers
of Jett Landing based, in part, on his belief that Blade was a
partner in the venture. Blade

Correct answer is D

A will have no liability in the loan with Bragg Bank.


B will be liable on the Bragg Bank loan only if he
fails to renounce liability when he learns of the banks
reliance.
C will be liable on the Bragg Bank loan only if
Wilsons representation could be considered a public
statement.
D will be liable on the loan with Bragg Bank.

The (Revised) Uniform Partnership Act continues


the basic principles of partnership by estoppel from
prior law. Estoppel is now more accurately entitled
Liability of Purported Partner. (RUPA continues
the distinction between representations made to
specific persons and those made in a public
manner, but this is not at issue in the question.)
There is no duty of denial, and thus a person held
out by another as a partner is not liable unless he
actually consents to the representation. Blades
repeated failure to object to the representation
would be evidence of his consent.

Business Environment & Concepts > Partnerships > partner distribution in kind

partner distribution in kind


Question 1

Answer 1

Parke, a partner in the general partnership of Parke, Stewart &


Hammond, has decided that working for the partnership from
her home would be suitable to her needs and lifestyle. In this
regard, Parke made arrangements for all the office equipment
in her office to moved to her home. When the remaining
partners protested, Parke asserted her right to a distribution "in
kind" under general partnership law. Which of the following
assertions made by one or more of partners would be correct
under the Uniform Partnership Act?

Correct answer is B

A A partner has the right to receive a distribution in


kind prior to dissolution.
B A partner may NOT be required to accept a
distribution in kind.
C A partner may demand a distribution in kind only
upon dissolution of the partnership.
D A partner has no right to receive, but may be
required to accept, a distribution in kind.

SECTION 402 of the Uniform Partnership Act


provides as follows: A partner has no right to
receive, and may not be required to accept, a
distribution in kind.

Business Environment & Concepts > Partnerships > liability of P for other P's contracts

liability of P for other P's contracts


Question 1

Answer 1

Kosh Training Centers, a general partnership consisting of


Larry Kosh, Peter Kosh and Ralph Inge, was engaged in the
business of management training. Kosh Training Centers was
awash in cash due to a successful year of business operations.
Peter Kosh invested most of the firms funds in a municipal
bond fund. When the fund took a significant drop in value, the
remaining partners wished to take action to recover the losses.
Which statement is correct?

Correct answer is A

A Peter Kosh may be liable to his partners if he


exceeded his authority in investing the firm's cash.
B In a general partnership, only partners who are
granted authority to invest partnership assets will have
such authority.
C In a general partnership, only the managing
general partner has authorith to invest partnership
assets.
D The broker who accepted the partnership funds
will be liable if Peter Kosh did not have express
authority to invest the partnership's funds.

In a general partnership, each partner has actual


authority to conduct business on behalf of the
partnership. Generally, this would include
investment of partnership assets. The partnership
could agree, however, that only certain partners
have such authority. Thus, Peter Kosh may have
violated such an agreement.

Potrebbero piacerti anche